Вы находитесь на странице: 1из 632
“Mark this question => Question Id : 22672 Question 1 of 30 During an outbreale of influenza A, which of the following may provide appropriate prophylaic for healthcare workers? @) Amantadine ) Oseltamivie ©) Cidofovir Lamimeine ©) Didanosine Question Explanation: Oseltamivir (Tarriflu) may be used in the prophylactic treatment of healthcare workers during fu epidemics. However, long tern. treatment does run a risk of resistance jon Report An Error “Mark this question => Question Id : 22672 Question 1 of 30 During an outbreak ofinfiuenza A, which of the following may provide appropsiate prophylasic for ealthoare woekcere? 2) Amantacine JY © b) Oseltamivir ©) Cidofovir Lamimeine ©) Didanosine Question Explanation: Oseltamivir (Tarriflu) may be used in the prophylactic treatment of healthcare workers during fu epidemics. However, long tern. treatment does run a risk of resistance jon Report An Error Mark this cuestion ez Question Id : 42649 Question 2 of 30 “The least likely cause of HIV infection is which of the following? a) Homosesality b) Blood transfusion c) Heterosexuality 4) Prostitution e) Drug abuse Answer (Bortanation Other User's Explanation Report An Error Question Explanation: ‘When HIV first started appearing in large numbers in North America in the early 1980's is was called the disease of the 4 H's Heroin users, Homosexuals, Hepatitis patieats and Haitians). Since then the mode of transmission of the virus has extensively studied and the following are the primary modes of transmission of HIV are: 1) Blood-to-blood transmission-through transfision of, or direst contact with, HIV-infected blood, exposure to HIV-contaminated needles syringes, and other equipment. 2) Sexual contact - unprotected vaginal, oral, or anal intercourse ditect contact with HIV-infected body fiuids such as semen and cervical and vaginal secretions 3) Mother to newbom - during pregnancy, labor and delivery. Since 1985 all blood producte all blood products are routinely tected for HIV before being given during transfusions. Mark this cuestion ez Question Id : 42649 Question 2 of 30 ‘The least likely cause of HIV infection is which of the following? a) Homasesnality Y © b) Blood transfusion c) Heterosexuality 4) Prostiution e) Drug abuse Answer (Bortanation Other User's Explanation Report An Error Question Explanation: ‘When HIV first started appearing in large numbers in North America in the early 1980's is was called the disease of the 4 H's Heroin users, Homosexuals, Hepatitis patieats and Haitians). Since then the mode of transmission of the virus has extensively studied and the following are the primary modes of transmission of HIV are: 1) Blood-to-blood transmission-through transfision of, or direst contact with, HIV-infected blood, exposure to HIV-contaminated needles syringes, and other equipment. 2) Sexual contact - unprotected vaginal, oral, or anal intercourse ditect contact with HIV-infected body fiuids such as semen and cervical and vaginal secretions 3) Mother to newbom - during pregnancy, labor and delivery. Since 1985 all blood producte all blood products are routinely tected for HIV before being given during transfusions. ‘Mark this question & => Question Td : 64407 Question 3 of 30 A.26 year old male patient ic experiencing inability to swallow and has speech dlffiulty. The patient was in pesfect health prior to the consumption of home canned green beans. The most appropriate treatment for this condition is a) Fenicilin administration +) Aduiristration of antitoxin ©) Adtainistration of tetanus toxoid 4) Placement of the patient in a hyperbaric oxygen chamber Anover (RESIRIRNAN) other Users Exptanat Question Explanation: Botuliem is newromuscular poisoning from Clostridium botulimmm toxin, Home canned foods are the most common sources, but commercially prepared foods have been implicated in about 10% of outbreaks. Vegetables, fish, fuits, and condiments are the most common vehicles, but beef, mile products Pork, poultry, and other foods have been involved. Symptoms of bulbar paresis (eg, dysarthria, dysphagia, dysphonia, a flaccid facial expression) develop. If diagnosed early, food-bome and wound botulism can be treated with an antitoxin that blocks the action of neurotoxin circulating in the blood. The trivalent antitovin is effective against three neurotoxins: A, B, aid E, Report An Error ‘Mark this question & => Question Td : 64407 Question 3 of 30 A.26 year old male patient ic experiencing inability to swallow and has speech dlffiulty. The patient was in pesfect health prior to the consumption of home canned green beans. The most appropriate treatment for this condition is a) Fenicilin administration J © b) Administration of antitoxin ©) Adtainistration of tetanus toxoid 4) Placement of the patient in a hyperbaric oxygen chamber Anover (RESIRIRNAN) other Users Exptanat Question Explanation: Botuliem is newromuscular poisoning from Clostridium botulimmm toxin, Home canned foods are the most common sources, but commercially prepared foods have been implicated in about 10% of outbreaks. Vegetables, fish, fuits, and condiments are the most common vehicles, but beef, mile products Pork, poultry, and other foods have been involved. Symptoms of bulbar paresis (eg, dysarthria, dysphagia, dysphonia, a flaccid facial expression) develop. If diagnosed early, food-bome and wound botulism can be treated with an antitoxin that blocks the action of neurotoxin circulating in the blood. The trivalent antitovin is effective against three neurotoxins: A, B, aid E, Report An Error Mark ths question = => Question Td: 65934 Question 4 of 30 A 57-year-old gardener presents to his physician with complaints of a rash on the upper forearm ofhis left arm. He has no other complaints. He had been working in his backyard for the last few weeks. On exam, he was found to have a non-tender red pepule on the left forearm with proximal extension along the lymphatics. What is the most likely dlagnosis? 2) Sporotrichosis 1b) Cryptococens ©) Mucormycosis 6) Actinomycosis ©) Lesihmaniasis Question Explanation: This patient has sporotrichosis, which is on infection caused by Sporothrix schenckii Its common in florists, gardeners, and mursery workers. Treatment is sarurated potassium iodide. Cryptococcus is a fungus which causes meningitis and pneumonitis in jinmminocompromised hosts, Mucormycosis is a very debilitating disease caused by the fungus Mucor or Rhizopus. It occurs in Gabetics, leukemic patients, and uremic patients. Treatment is extensive debridement and amphotericin. Actinomycosis is an infection caused by the anaerobic Gram-postiive branching bactena. It occurs by a brealc in mucosa or by aspiration Sulfa gramules are seen cu nmicroscopy. Lesihmaniasis is a vecior borne disease, Report An Error Mark ths question = => Question Td: 65934 Question 4 of 30 A 57-year-old gardener presents to his physician with complaints of a rash on the upper forearm ofhis left arm. He has no other complaints. He had been working in his backyard for the last few weeks. On exam, he was found to have ancn-tender red papule on the left forearm with proximal extension along the lymphatics. Whatis the most likely dlagnosis? Y © & Sporotrichosis ) Cryptococcus ©) Mucomycosis 6) Actinomycosis ©) Lesihmaniasis Question Explanation: This patient has sporotrichosis, which is on infection caused by Sporothrix schenckii Its common in florists, gardeners, and mursery workers. Treatment is sarurated potassium iodide. Cryptococcus is a fungus which causes meningitis and pneumonitis in jinmminocompromised hosts, Mucormycosis is a very debilitating disease caused by the fungus Mucor or Rhizopus. It occurs in Gabetics, leukemic patients, and uremic patients. Treatment is extensive debridement and amphotericin. Actinomycosis is an infection caused by the anaerobic Gram-postiive branching bactena. It occurs by a brealc in mucosa or by aspiration Sulfa gramules are seen cu nmicroscopy. Lesihmaniasis is a vecior borne disease, Report An Error ‘Mark this question = => Question Td : 75930 Question 5 of 30 A.49-year-old male presents with cutaneous ulcers and hyperpiamentation Physical examination reveals hyperpigmentation. Which one oftthe following parasitic infections does the patient most likely have? a) Bubonic Plague ) Toxoplasmosis ©) Trypanosomniasis 6) Leishmaniasis 8) Fileviasis Question Explanation: Leishmaniasis, or kala-azar (translated, "black sickness’), is a disease caused by Leishmania donovani, Ibis transmitted via the phlebotamine sand fly and causes cutaneous ulcers and hyperpigmentation None of the other answer choices are consistent with, these clinical findings Report An Error ‘Mark this question = => Question Td : 75930 Question 5 of 30 A.49-year-old male presents with cutaneous ulcers and hyperpiamentation Physical examination reveals hyperpigmentation. Which one of the fellowing parasitic infections does the patient most likely have? a) Bubonic Plague +) Toxoplasmesie ©) Trypanosomniasis YM © O Leishmaniasis ©) Filariass Question Explanation: Leishmaniasis, or kala-azar (translated, "black sickness’), is a disease caused by Leishmania donovani, Ibis transmitted via the phlebotamine sand fly and causes cutaneous ulcers and hyperpigmentation None of the other answer choices are consistent with, these clinical findings Report An Error Mark this question = => Question Id : 80240 Question 6 of 30 ‘Worldwide, Amebiasis isthe third cause of death from parasitic disease. Which one of the following is true regarding infection caused ‘by Entameba histolytica? 2) Foodbome infection is prevalent due to cyst ingestion. +) Motile trophozoites, released from cysts in the smal intestine, almost always cause invasive disease ©) Following encystation, infectious cysts which are shed in the stool, rarely survive more than 24 hours 4) Trophozoites play a vital role in causing infection, ©) Production ofintracellular proteinases and sensitivity to complerment-mediated lysis are two means by which virulence is increased. Question Explanation: “Amebiasis is the third cause of death from parasitic disease worlduade, Death rates are exceeded only by malaria and Schistosomiasis. Infection follows ingestion of vizble cysts fom fecally contaminated water, food, or hands. Foodborne infection is prevalent because food handlers shed cysts Foodbome infection may also occur because food is grown in fecally contaminated soil or produced using fecally contaminated fertilizer. Motile trophozcites, following release from cysts in the small intestine, remain as harmless commensals in the bowel of most persons. After encystation, infectious cysts are shed in the stool, where they can survive for several weeks in a moist environment Trophozoites, which are rapidly killed by exposure to either air or stomach acid, do not produce infection. The wide spectrum of disease results from the virulence of the infeciing strain. Several factors may affect virulence, jncliding the production of extracellular proteinases and resistance to complement-mediated lysis Mark this question = => Question Id : 80240 Question 6 of 30 ‘Worldwide, Amebiasis isthe third cause of death from parasitic disease. Which one of the following is true regarding infection caused ‘by Entameba histolytica? Y © a) Foodbome infection is prevalent due to cyst ingestion. +) Motile trophozoites, released from cysts in the smal intestine, almost always cause invasive disease ©) Following encystation, infectious cysts which are shed in the stool, rarely survive more than 24 hours 4) Trophozoites play a vital role in causing infection, ©) Production ofintracellular proteinases and sensitivity to complerment-mediated lysis are two means by which virulence is increased. Question Explanation: “Amebiasis is the third cause of death from parasitic disease worlduade, Death rates are exceeded only by malaria and Schistosomiasis. Infection follows ingestion of vizble cysts fom fecally contaminated water, food, or hands. Foodborne infection is prevalent because food handlers shed cysts Foodbome infection may also occur because food is grown in fecally contaminated soil or produced using fecally contaminated fertilizer. Motile trophozcites, following release from cysts in the small intestine, remain as harmless commensals in the bowel of most persons. After encystation, infectious cysts are shed in the stool, where they can survive for several weeks in a moist environment Trophozoites, which are rapidly killed by exposure to either air or stomach acid, do not produce infection. The wide spectrum of disease results from the virulence of the infeciing strain. Several factors may affect virulence, jncliding the production of extracellular proteinases and resistance to complement-mediated lysis Merk this question = => Question Td : 81391 Question 7 of 30 A.55 year old male pet store owner, who received anew shioment of lizards and birds about 12 days eatlier, develops a high fever, a severe headache, and 2 hacking cough. The most appropriate cause of his illness is a) Tuleremia +) Paittacosis ©) Rabies 6) Trichinosis ©) Sporotrichosis Question Explanation: Psittacosis, caused by Chlamydia psittaci and often transtritted by contact with infected birds, is the likeliest cause of this man’s ilness, Tulatemia (A) does not present in this manner and is usually associated with exposure to sick rabbits or other rodents. Rabies (C) does not manifest in this fashion and is usually associated with an animal bite, Trichinosis (D) is a parastic infection most commonly resulting from ingestion of inadequately cooled meat. Sporotrichosis (E) ic a fungal infection that can be transmitted via thom pricks from plants such as roses. Merk this question = => Question Td : 81391 Question 7 of 30 A.55 year old male pet store owner, who received anew shioment of lizards and birds about 12 days eatlier, develops a high fever, a severe headache, and 2 hacking cough. The most appropriate cause of his illness is a) Tuleremia oY © b) Psittacosis ©) Rabies 6) Trichinosis ©) Sporotrichosis Question Explanation: Psittacosis, caused by Chlamydia psittaci and often transtritted by contact with infected birds, is the likeliest cause of this man’s ilness, Tulatemia (A) does not present in this manner and is usually associated with exposure to sick rabbits or other rodents. Rabies (C) does not manifest in this fashion and is usually associated with an animal bite, Trichinosis (D) is a parastic infection most commonly resulting from ingestion of inadequately cooled meat. Sporotrichosis (E) ic a fungal infection that can be transmitted via thom pricks from plants such as roses. Mark this cuestion ez Question Id: 86914 Question 8 of 30 An immigrant ftom Brazil presents with difficulty swallowing, and is referred to a gastroenterologist for evaluation of his dysphegia. Studies demonstrate massive dlatation of the esophagus. Other organ that may also be sericusly affected by the patients disease is a) Bladder b) Bran ©) Heart ) Lungs e) Small intestine Question Explanation: “The disease is Chagas disease, which is due to the intracellular protozoan parasite, Trypanosoma cruzi. Chagas cisease is an important cause of cardiac failure in some Latin American countries, and Braziian cases can be associated with megaesoohagus and megacolon. The bladder parasite that you should remember is Schistosoma haeratobiutn The brain parasites you should remember ace Neegleria fowlesi, Acanthoamocba , and Echinococcus granulosis. The jung parasites you should remember is Paragonismus ‘westermenni, Small intestinal parasites are yew numerous, and notably include Ascaris Taenia species and Strongyloides. Mark this cuestion ez Question Id: 86914 Question 8 of 30 An immigrant ftom Brazil presents with difficulty swallowing, and is referred to a gastroenterologist for evaluation of his dysphegia. Studies demonstrate massive dlatation of the esophagus. Other organ that may also be sericusly affected by the patients disease is a) Bladder b) Bran V © c) Heat 4) Lungs e) Small intestine Question Explanation: “The disease is Chagas disease, which is due to the intracellular protozoan parasite, Trypanosoma cruzi. Chagas cisease is an important cause of cardiac failure in some Latin American countries, and Braziian cases can be associated with megaesoohagus and megacolon. The bladder parasite that you should remember is Schistosoma haeratobiutn The brain parasites you should remember ace Neegleria fowlesi, Acanthoamocba , and Echinococcus granulosis. The jung parasites you should remember is Paragonismus ‘westermenni, Small intestinal parasites are yew numerous, and notably include Ascaris Taenia species and Strongyloides. ‘Mark this question —& => Question Td : 94353 Question 9 of 30 A 29 year old menstruating weman presents at the Emergency Department with a high fever, vomiting and ciarrhea of 24 hours duration, Her blood pressure is 90/50 mm He and there is a diffuse ervthematous rash. Which of the following laboratory resut does not keep with a dlagosis of toxic shock syndrome? a) Creatine kinase 290 UHL QV 20-170 U/L) ) Calcium (total) 1. 66mmolfL (NT 2.2.2 74mmoliL) ©) White blood cell count 18 x 10°9/L QV 3.8-98 x 109. ©) Aspartate aminotransferase (AST) 100 U/L QN 11-47 U/L) 8) Glucose 2.2 mmol/L (Y 3.6-6.3 mmovL) Question Explanation: Toxic shock syndrome is caused by staphylococcal or streptococcal exotoxins. Symptoms include high fever, hypotension, diffuse erythematous rash, and multiple organ involvement thar may rapidly progress to severe and intractable shock, Diagnosis is made clinically and by isolating the orgenism. Treatment includes antibiotics, intensive support, and immunoglobulin, Creatine kinase levels may indicate rhabdomyolysis. Electrolste levels may indicate hyponatremia, hypokalemia, hypocalcemia out of proportion to hypoalbuninemia, hypophosphatemia, and hypomagnesemia. The CBC may reveal leulcocytosis. Liver fiction test results may reveal an elevated aspartate aminotransferase (AST/SGOT) level ‘Mark this question —& => Question Td : 94353 Question 9 of 30 A 29 year old menstruating weman presents at the Emergency Department with a high fever, vomiting and ciarrhea of 24 hours duration, Her blood pressure is 90/50 mm He and there is a diffuse ervthematous rash. Which of the following laboratory resut does not keep with a dlagosis of toxic shock syndrome? a) Creatine kinase 290 UHL QV 20-170 U/L) ) Calcium (total) 1. 66mmolfL (NT 2.2.2 74mmoliL) ©) White blood cell count 18 x 10°9/L QV 3.8-98 x 109. ©) Aspartate aminotransferase (AST) 100 U/L QN 11-47 U/L) Y © &) Gtucose 2.2 mmol ( 3.6-6.3 minoVL) Question Explanation: Toxic shock syndrome is caused by staphylococcal or streptococcal exotoxins. Symptoms include high fever, hypotension, diffuse erythematous rash, and multiple organ involvement thar may rapidly progress to severe and intractable shock, Diagnosis is made clinically and by isolating the orgenism. Treatment includes antibiotics, intensive support, and immunoglobulin, Creatine kinase levels may indicate rhabdomyolysis. Electrolste levels may indicate hyponatremia, hypokalemia, hypocalcemia out of proportion to hypoalbuninemia, hypophosphatemia, and hypomagnesemia. The CBC may reveal leulcocytosis. Liver fiction test results may reveal an elevated aspartate aminotransferase (AST/SGOT) level Mark this question ez Question Id : 95321 Question 10 of 30 Aman contracts HTLV 1 infection through sexual contact. He lives in southern Japan. Twenty five year later he develops generalized lymphadenopathy with hepztosplenomegaly, a skin rash, hypercalcemia, and an elevated white blood count. Which of the following has mastlikely occurred inthis patient? 2) AIDS 6) Autoimmmarity ©) Delayed hypersensitivity reaction 4) Levkemia «) Recurrent infection Anower [UERSNGEN) otter ueors Explanation Report An Error Question Explanation: HTLV-1, or human T-cell lymphotrophic virus 1 is an enveloped, single-stranded, RMA retrovirus endemic to scuthem Japan and the Caribbean basin, but sporadically seen in the United States, Infection by HTLV-1 can lead to adu T-cell leukemia/lymphora (ATLL) 20-30 years after the infection. The HTLV-associated T-cell leuicernia generally presents as described above, and is very aggressive, progressing to death in less than one year. AIDS acanired immunodeficiency syndrome, is transmitted through the HIV virus. Although both wiuses are RNA retroviruses and both infect CD4+T cells, the diseases they produce are distinct Antoimmane diseases produced after infections arc typified by rheumatic fever afer infection with Streptococcus. There is speculation that EBV virus Epstein-Barr virus) may produce the autoimmune reactions in rheumatoid arthritis, but autoimmunity plays no role in leukernia alter HTLV-1 infection. Delayed hypersensitivity reactions are T-cell-mediated immune responses typified by the subcutaneous reaction to tuberculin in sensitized incivicuals. It characteristielly occurs 2-3 days after an exposure, andis a local lymphocytic response. Recurrent infections occur when latent viruses are reactivated in the host at the site of primary infection Recurrent infections are most common with the herpesviruses, especially HSV-1, HSV-V2, and VZV (varicella zoster virus). Mark this question ez Question Id : 95321 Question 10 of 30 Aman contracts HTLV | infection through sexual contact. He lives in southern Japan. Twenty five year later he develops generalized Ismphadenopethy with hepetosplenomegaly, a skin rash, hypercalcemia, and an elevated white blood count, Which of the following has most likely occurred in this patient? a) AIDS 6) Autoimmmarity c) Delayed hypersensitivity reaction Y¥ © @ Leukemia e) Recurrent infection Anower [UERSNGEN) otter ueors Explanation Report An Error Question Explanation: HTLV-1, or human T-cell lymphotrophic virus 1 is an enveloped, single-stranded, RMA retrovirus endemic to scuthem Japan and the Caribbean basin, but sporadically seen in the United States, Infection by HTLV-1 can lead to adu T-cell leukemia/lymphora (ATLL) 20-30 years after the infection. The HTLV-associated T-cell leuicernia generally presents as described above, and is very aggressive, progressing to death in less than one year. AIDS acanired immunodeficiency syndrome, is transmitted through the HIV virus. Although both wiuses are RNA retroviruses and both infect CD4+T cells, the diseases they produce are distinct Antoimmane diseases produced after infections arc typified by rheumatic fever afer infection with Streptococcus. There is speculation that EBV virus Epstein-Barr virus) may produce the autoimmune reactions in rheumatoid arthritis, but autoimmunity plays no role in leukernia alter HTLV-1 infection. Delayed hypersensitivity reactions are T-cell-mediated immune responses typified by the subcutaneous reaction to tuberculin in sensitized incivicuals. It characteristielly occurs 2-3 days after an exposure, andis a local lymphocytic response. Recurrent infections occur when latent viruses are reactivated in the host at the site of primary infection Recurrent infections are most common with the herpesviruses, especially HSV-1, HSV-V2, and VZV (varicella zoster virus). Mark this question & => Question Td ; 95885 Question 11 of 30 A woman suffered a sore throat during her third trimester of pregnancy. She becomes febrile and hypotensive after delivery. She most likely has which of the following? a) Paeumocysts cannit +) Streptococcal sepsis. c) Miliary tuberculosis. ) Staphylococcal sepsis. ¢) Pseudomonas sepsis Answer | Bolanation Other User's Explanation Report An Error Question Explanation The patient probably has puerperal sepsis, which is caused by streptococcal infection. Pneumocystis carinii does not cause sore throats or other disease in healthy individuals Similarly miliary tuberculosis is not associated with such a clinical pattern Both staphylococcal and pseudomonas sepsis ere associated with acute fulminant infections Mark this question & => Question Td ; 95885 Question 11 of 30 A woman suffered a sore throat during her third trimester of pregnancy. She becomes febrile and hypotensive after delivery. She most likely has which of the following? a) Paeumocysts cannit WV © b) Streptococcal sepsis. c) Miliary tuberculosis. ) Staphylococcal sepsis. ¢) Pseudomonas sepsis Answer | Bolanation Other User's Explanation Report An Error Question Explanation The patient probably has puerperal sepsis, which is caused by streptococcal infection. Pneumocystis carinii does not cause sore throats or other disease in healthy individuals Similarly miliary tuberculosis is not associated with such a clinical pattern Both staphylococcal and pseudomonas sepsis ere associated with acute fulminant infections ‘Mark this question ez Question Id : 97333 Question 12 of 30 “Which ofthe following viruses can NOT be transmitted transplacentally? a) CMV ) Rubella c) EBV juSV ey HIV Question Explanation: CMV, rubella, HBV, and HIV can be transmitted transplacentally. However, HSV infection in perinates is contracted via delivery through the vaginal canal and conzact with the vaginal epithelium. ‘Mark this question ez Question Id : 97333 Question 12 of 30 ‘Which of the following viruses can NCT be transmitted transplacentaly? a) CMV +) Rubella c) EBV vO aousV e) HIV Question Explanation: CMV, rubella, HBV, and HIV can be transmitted transplacentally. However, HSV infection in perinates is contracted via delivery through the vaginal canal and conzact with the vaginal epithelium. ‘Mark this question & => Question Td : 100067 Question 13 of 30 Infection thet is associated with draining sinuses overlying affected joints? 8) Candidiasis b) Detmatophytosis ©) Staphylococci 6) Sporotrichosis 6) Histoplasmosis Question Explanati Joint infection with Sporethrix schenckii can resuit in draining sinus tracts along lymphatics over the involved joint. Candida, Staphylococci, and, rarely, histoplasmosis can involve joints, but do not cause sims tracts to drain, Dermatophytosis describes superficial infections, such as tinea, that do not involve joints ‘Mark this question & => Question Td : 100067 Question 13 of 30 Infection thet is associated with draining sinuses overlying affected joints? 8) Candidiasis +b) Detmatophytosis 6) Staphglocacci Y © & Sporotrichosis 6) Histoplasmosis Question Explanati Joint infection with Sporethrix schenckii can resuit in draining sinus tracts along lymphatics over the involved joint. Candida, Staphylococci, and, rarely, histoplasmosis can involve joints, but do not cause sims tracts to drain, Dermatophytosis describes superficial infections, such as tinea, that do not involve joints “Mark this question => Question Td : 119920 Question 14 of 30 Clinical feature that has NOT been associated with HIV infection include 2) Folymyositis +) Reiter's syndrone ©) Bsotiatic arthritis 4) Spemdyltie ©) Osteoarthritis Question Explanation: HIV/AIDS has not been specifically associated with osteoarthritis, but increased incidence of polymyositis, sometimes related to treaiment, Reiter's syndrome, psoriatic arthritis, and various spordylitides have all bea described in these patients. “Mark this question => Question Td : 119920 Question 14 of 30 Clinical feature that has NOT been associated with HIV infection include 2) Folymyositis +) Reiter's syndrone ©) Bsotiatic arthritis 4) Spemdyltie YM © 2&) Osteoarthritis Question Explanation: HIV/AIDS has not been specifically associated with osteoarthritis, but increased incidence of polymyositis, sometimes related to treaiment, Reiter's syndrome, psoriatic arthritis, and various spordylitides have all bea described in these patients. ‘yr interface. adu.ck/') % CL wow, interface.edu.pk/ned Apps [Google A Settings [Signin g oe wsdl diye Ofte ge sigsoll JS) Free Hatmal Gi other bookmar ‘Matk this question ert Question Td : 141502 Question 15 of 30 As EB donor call strain having genes for resistance to ceftazidime, empicilis, vancomycin, gentamicin, and tetracycline is cultures with a bacterial strain that is sensitive to all of these drugs. On growth of resultant recombinants in antibictic containing medium, 95% of the isclates grew in the presence of amapicilin, 80% in the presence of gentamicin, 75% with tetracycline. 60% with ceftazidime, and 45% ‘with Vancomycin, A characteristic ofthe location of these dmg resistance genes is which one of the following? a) The drug resistance genes are in a bacteriophage genome 'b) The drug resistance genes are in a conjugative plasmid, c) The drug resistance genes are in an episome. 4) The gene in sequence closest to Orit encodes Vancomycin resistance, ©) The gene in the sequence furthest from the tra operon encodes ampiciin resistance Avowor [EQNS omer Usors Exot Question Explanation: Hi donor bacteria are capable of transferring chromosomal genes to F- recipient cells. The order of the transfer is from ort towards the tra operon although the tra operon is never successfully transferred to the recipient in this case. Thus the likethood of successful recombination is a fimction of the placement of genes on the chromosome. Those closest to Orit are most licely to be successfully trensferred (in this case ampicillin resistance): and those farthest away are least likely (n this case, vancomycin resistance). The drug resistance genes are in a bacteriophage genome is not true. In the Hi cell, the drug resistance genes are in the chromosome of a cell that has a stably integrated episome (a plasmid that has been insested in the chromosome by site-specific recombination). Traits given. to bacteria by stably integrated bacteriophages include the antigen of Salmonella, the botulinum exotoxin, the eryihrogenic exotoxins of Streptococcus pyogenes and the diphtheria toxin, The drug resistance genes are in a conjugative plasmid is not tue, Conjugative plasmids exist in F+ bacteria, not HIF strans When plasmid DNA is transferred in this type of conjugation, all plasmid genes are trensferred and there is no time dependent acquisition of recombinant traits as is described here. The drug resistance genes arc in episcme is not true. The genes thet are trensferred in an Eft conjugal cross are chromosomal genes, not episome genes. The episome (integrated plasmid) causes the formation ofthe conjugal bridge and contains the tra operon and Orit, but the recombined genes are all from the chromosome because at the end of the transfer process, homologous recombination is required to stabilize the donated DNA. Homologous recombination requires homology between the exegenote and chromosomal DMA, and the F-recipient does not have any DIVA homologous to the DINA thet originates in the episome. The gene in sequence closest to Orit encodes vancomycin resistance is not trae Since this gene is transferred to the smallest number of recombinant strains, it means that this is the least likely to male, it across the conjugal bridge: t is fixthest from otit and closest in sequence to the ta operon, jon Report An Error Total Questions fs KO Heo Fa do es BRE Is ke ke Sk SR BEREERBE cKKXKRKXKKKKKKAKRKXKKAKRAKRKAKRAKRMAKRAKRRKRXRK ‘yr interface. adu.ck/') % CL wow, interface.edu.pk/ned Apps [Google A Settings [Signin g oe wsdl diye Ofte ge sigsoll JS) Free Hatmal Gi other bookmar ‘Matk this question ert Question Td : 141502 Question 15 of 30 As EB donor call strain having genes for resistance to ceftazidime, empicilis, vancomycin, gentamicin, and tetracycline is cultures with a bacterial strain that is sensitive to all of these drugs. On growth of resultant recombinants in antibictic containing medium, 95% of the isclates grew in the presence of amapicilin, 80% in the presence of gentamicin, 75% with tetracycline. 60% with ceftazidime, and 45% with Vancomycin. A characteristic ofthe location of these drug resistance genes is which one of the following? a) The drug resistance genes are in a bacteriophage genome b) The drug resistance genes are in a conjugative plasmid c) The drug resistance genes are in an episome. 4) The gene in sequence closest to Orit encodes Vancomycin resistance, Y © €)The gene in the sequence furthest from the ra operon encodes ampicillin resistance. Avowor [EQNS omer Usors Exot Question Explanation: Hi donor bacteria are capable of transferring chromosomal genes to F- recipient cells. The order of the transfer is from ort towards the tra operon although the tra operon is never successfully transferred to the recipient in this case. Thus the likethood of successful recombination is a fimction of the placement of genes on the chromosome. Those closest to Orit are most licely to be successfully trensferred (in this case ampicillin resistance): and those farthest away are least likely (n this case, vancomycin resistance). The drug resistance genes are in a bacteriophage genome is not true. In the Hi cell, the drug resistance genes are in the chromosome of a cell that has a stably integrated episome (a plasmid that has been insested in the chromosome by site-specific recombination). Traits given. to bacteria by stably integrated bacteriophages include the antigen of Salmonella, the botulinum exotoxin, the eryihrogenic exotoxins of Streptococcus pyogenes and the diphtheria toxin, The drug resistance genes are in a conjugative plasmid is not tue, Conjugative plasmids exist in F+ bacteria, not HIF strans When plasmid DNA is transferred in this type of conjugation, all plasmid genes are trensferred and there is no time dependent acquisition of recombinant traits as is described here. The drug resistance genes arc in episcme is not true. The genes thet are trensferred in an Eft conjugal cross are chromosomal genes, not episome genes. The episome (integrated plasmid) causes the formation ofthe conjugal bridge and contains the tra operon and Orit, but the recombined genes are all from the chromosome because at the end of the transfer process, homologous recombination is required to stabilize the donated DNA. Homologous recombination requires homology between the exegenote and chromosomal DMA, and the F-recipient does not have any DIVA homologous to the DINA thet originates in the episome. The gene in sequence closest to Orit encodes vancomycin resistance is not trae Since this gene is transferred to the smallest number of recombinant strains, it means that this is the least likely to male, it across the conjugal bridge: t is fixthest from otit and closest in sequence to the ta operon, jon Report An Error Total Questions fs KO Heo Fa do es BRE Is ke ke Sk SR BEREERBE cKKXKRKXKKKKKKAKRKXKKAKRAKRKAKRAKRMAKRAKRRKRXRK Mark ths question e& => Question Td = 141574 Question 16 of 30 A.66 year old African immigrant developed thick erythematous nodules on her ears and nose with significant associated sensory loss The nodules have grown slowly over the course of many years. Biopsy reveals dermal granulomas with giant cells but no acid fast bacteria. No growth is seen on blood agar and Lowenstein Jensen medium. These findings are consistent with which diagnosis? 2) Cutaneous leishmaniasis ‘b) Lepromatous leprosy ©) Onchocerciasis 4d) Rhinoscleroma 6) Tuberculoid leprosy Answer | Explanation Other User's Explanation Report An Error Question Explanation: Mycobacterium leprae is endlemic to parts of Aftica, Asia, and South America. Tuberculoid leprosy is an indolent cisease typically affecting cooler parts of the body (such as the nose and ears). producing dermal granulomas with very rare, acid fast bacilli and damage to peripheral nerves. This icin marleed contrast to leprematous leprosy, which is progressive, invasive, and generally characterized by the presence of aumerous acid fast bacteria in a histiocytic but non-granulomatous tissue resporse. M. leprae hes not been grown in any culture medium, Cutaneous leishmaniasis is clue to Leishinania species thet show a worldwide distribution. The infection is transmitted through a sand fly bite, and the skin lesion is typically ulcerated. Histology shows intracellular parasites within the dermis and epidermis. Granulomas are not formed. Lepromatous leprosy is the form of disease when the wrong T-helper (TE) cellis activated, Because TH2 cells stimulate antibody production, they have no value against an intravelluker pathogen, and the patient develops disfiguring disseminated disease. Onchocerciasis, or river blindness, is a roundworm infecton transmitted by black fies of Aftica and South America The microfilarias grow atthe site of inoculation and cause an inflamed subcutaneous nodule, The «organism is seen onticcue sections. Rhinoscleroma is a destructive granulomatous infection of the nacepharynst and is caused by Klebsiella rhinoscleromatis. Gram-negative rods can be-cultured fiom the lesions. Mark ths question e& => Question Td = 141574 Question 16 of 30 A.66 year old African immigrant developed thick erythematous nodules on her ears and nose with significant associated sensory loss The nodules have grown slowly over the course of many years. Biopsy reveals dermal granulomas with giant cells but no acid fast bacteria. No growth is seen on blood agar and Lowenstein Jensen medium. These findings are consistent with which diagnosis? 2) Cutaneous leishmaniasis ‘b) Lepromatous leprosy ©) Onchocerciasis 4d) Rhinoscleroma Y © 8) Tubercuboid leorosy Answer | Explanation Other User's Explanation Report An Error Question Explanation: Mycobacterium leprae is endlemic to parts of Aftica, Asia, and South America. Tuberculoid leprosy is an indolent cisease typically affecting cooler parts of the body (such as the nose and ears). producing dermal granulomas with very rare, acid fast bacilli and damage to peripheral nerves. This icin marleed contrast to leprematous leprosy, which is progressive, invasive, and generally characterized by the presence of aumerous acid fast bacteria in a histiocytic but non-granulomatous tissue resporse. M. leprae hes not been grown in any culture medium, Cutaneous leishmaniasis is clue to Leishinania species thet show a worldwide distribution. The infection is transmitted through a sand fly bite, and the skin lesion is typically ulcerated. Histology shows intracellular parasites within the dermis and epidermis. Granulomas are not formed. Lepromatous leprosy is the form of disease when the wrong T-helper (TE) cellis activated, Because TH2 cells stimulate antibody production, they have no value against an intravelluker pathogen, and the patient develops disfiguring disseminated disease. Onchocerciasis, or river blindness, is a roundworm infecton transmitted by black fies of Aftica and South America The microfilarias grow atthe site of inoculation and cause an inflamed subcutaneous nodule, The «organism is seen onticcue sections. Rhinoscleroma is a destructive granulomatous infection of the nacepharynst and is caused by Klebsiella rhinoscleromatis. Gram-negative rods can be-cultured fiom the lesions. Marke this question & => Question Td: 141772 Question 17 of 30 ‘A 35 year old woman presents to the ER complaining of double vision Examination demonstrates a dry mouth, crooping eyelids, markedly diminished papillary light reflex, a flaccid facial expression, and difficulty opening her mouth, This condition was acquired by what method? a) Fecal oral contamination +) Ingestion of preformed toxin ©) Ingestion of spores, 4) Semual transmission ¢) Traumatic implantation Question Explanation: This is a typical presentation of botulism, which is cansed by the preformed toxin of Clostridinin botalinam. Patients develop a progressive flaccid paralysis that can lead to death ithe respiratory system is not adequately supposted, Mentation is intectiin these patients, and they do not exhibit sensory changes except for those, such as the double vision related to failure of extra and intraocular muscles, Botulism can follow ingestion of home canned vegetables, fish, fruts, and condiments Other vehicles that have been the source of outbreaks inchide commercially prepared canned foods, poultry, and dairy products, and even restanrant prepared non camed foods, including seafood, oil wrapped baked potatoes, chopped gadic in oil, and paltry melt sandwiches. For this reason, the absence of a classic history of ingestion of home canned food should not be used to exclude the diagnosis of botulism, Treatment is suppottive, An antitoxin is available, but may not be obtained early enough to be helpful. Most patients recover from food borne ‘botulism without sequelae. Fecal oral transmission is important in several types of food poisoning such as Salmonella, Escherichia coli, and Campylobacter, but is notinvolved in adu botulism. Ingestion of spores is the means of acquisition of infant botulism or floppy baby syndrome. In these cases, the microenvironment of the infant's digestive tract is suitable for germination of the ingested spores of C. botalimam, and a toxic infection occurs in which the toxin is made invivo, Sexual transmission has not been impliceted in cases of food poisoning although anal sex has been shown to transmit Giardia lambla and Entemoeba histolytica, Traumatic implantation is the means of infection with the spores of C. botulinum in cases of wound botulism. Report An Error Marke this question & => Question Td: 141772 Question 17 of 30 ‘A 35 year old woman presents to the ER complaining of double vision Examination demonstrates a dry mouth, crooping eyelids, markedly diminished papillary light reflex, a flaccid facial expression, and difficulty opening her mouth, This condition was acquired by what method? a) Fecal oral contamination Y © b) Ingestion of preforined toxin ©) Ingestion of spores, 4) Semual transmission ¢) Traumatic implantation Question Explanation: This is a typical presentation of botulism, which is cansed by the preformed toxin of Clostridinin botalinam. Patients develop a progressive flaccid paralysis that can lead to death ithe respiratory system is not adequately supposted, Mentation is intectiin these patients, and they do not exhibit sensory changes except for those, such as the double vision related to failure of extra and intraocular muscles, Botulism can follow ingestion of home canned vegetables, fish, fruts, and condiments Other vehicles that have been the source of outbreaks inchide commercially prepared canned foods, poultry, and dairy products, and even restanrant prepared non camed foods, including seafood, oil wrapped baked potatoes, chopped gadic in oil, and paltry melt sandwiches. For this reason, the absence of a classic history of ingestion of home canned food should not be used to exclude the diagnosis of botulism, Treatment is suppottive, An antitoxin is available, but may not be obtained early enough to be helpful. Most patients recover from food borne ‘botulism without sequelae. Fecal oral transmission is important in several types of food poisoning such as Salmonella, Escherichia coli, and Campylobacter, but is notinvolved in adu botulism. Ingestion of spores is the means of acquisition of infant botulism or floppy baby syndrome. In these cases, the microenvironment of the infant's digestive tract is suitable for germination of the ingested spores of C. botalimam, and a toxic infection occurs in which the toxin is made invivo, Sexual transmission has not been impliceted in cases of food poisoning although anal sex has been shown to transmit Giardia lambla and Entemoeba histolytica, Traumatic implantation is the means of infection with the spores of C. botulinum in cases of wound botulism. Report An Error 3/1/2014 6:24:56 PM ‘Mark this question & => Question Td : 141823 Question 18 of 30 A 36 year old female presents with vaginal tchiness and discharge. Pelvic examination shows abundant white, curdy material in the vagina, Microscopy revealss fungal hyphae and yeast forms Systemic discase that can predispose for this condition is a) Crohn disease b) Diabetes melitus ©) Disseminated gonocorcal infection ) Rheumatoid arthritis ) Systemic lupus erythematoms Anower (FERRER) Other Users Explanation — Report An E01 Question Explanation: ‘The patient has vulvovaginitis secondary to Candida infection, Predisposing factors include a high vaginal pH, diabetes, anduse of antibiotics. The increased vulnerabiity in diabetes may reflect increased glucose concentrations in vaginal secretions and relative itnrmmosuppression, Crohn disease can predispose for fistulas involving the vagina, rather than vulvovaginitis. Dissemination of Neisseria gonorrhoeae can canse septic arthritis, Neither rheumatoid arthritis nor systemic Iupus erythematosus is specifically associated with Candida vulvovegiits. 3/1/2014 6:24:56 PM ‘Mark this question & => Question Td : 141823 Question 18 of 30 A 36 year old female presents with vaginal tchiness and discharge. Pelvic examination shows abundant white, curdy material in the vagina. Microscopy revealss fungal hyphae and yeast forms. Systemic disease that can precispose for this condition is a) Crohn disease Y © b) Diabetes mellitus ©) Disseminated gonocorcal infection 4) Rheumatoid ertaits «) Systemic ypu erythemstosus Anower (FERRER) Other Users Explanation — Report An E01 Question Explanation: ‘The patient has vulvovaginitis secondary to Candida infection, Predisposing factors include a high vaginal pH, diabetes, anduse of antibiotics. The increased vulnerabiity in diabetes may reflect increased glucose concentrations in vaginal secretions and relative itnrmmosuppression, Crohn disease can predispose for fistulas involving the vagina, rather than vulvovaginitis. Dissemination of Neisseria gonorrhoeae can canse septic arthritis, Neither rheumatoid arthritis nor systemic Iupus erythematosus is specifically associated with Candida vulvovegiits. ‘Marke this question €& => Question Td : 141854 Question 19 of 30 A4 year old gitl developed a fever with a cough, runny nose, and pirke eyes for a few days, but this went away. Today she presents with red lesions beginning on the back of her aeck and now spreading to the front and progressing to her trunk. On examination an erythematous, macular rash invoWing the face, neck, thorax and upper arms is seen. Irregular hypopigmented areas on the anterior ‘buccal macosa are also observed. Another organism in the same family as this infective organism is a known cause of which condition? 2) Bronchiotitis +) California encephalitis ©) Ebola 4) Lymphocytic chosiomeringitie «) Rabies Answer (Explonation | Other User's Explanation Report An Error Question Explanation: ‘The child in the question has measles. It starts with the classic prodrome of cough cotyza, and conjunctivitis, usually with a high fever. [As this dissipates over afew days and prior to the onset of the exanthem is the development of the enanthem, ie. hypopigmented, jsregular oral lesions on the anterior buccal mucosa, These are known as Koplik spots and are shortlived Next, is the erythematous ‘macular rash that begins at the posterior hair ine and behind the ears, spreads anteriorly to the face and then spreads downward to the trunk and extremities, The rash lasts up to ten days. The measles virus is a paramyxovirus, which is a single stranded negative sense RNA vis Other vinses in this group are mumps, parainflienza (infections croup) and respiratory syncytial virus, the major cause of bronchiolitis. California encephalitis is 2 bunyavirus, which is also a single stranded RNA virus The majer illnesses in this group are Calfomia encephaltis, La Crosse encephalitis, and Hantavirus infection Ebola is a flovirus, which is a single stranded ENA virus. The other major virus in this group is the Marburg virus. Lymphocytic choriomeningitis is a member of the arenavirus group, which ica single-stranded RMA virus. The other major illness is Lassa fever. Rabies is a chabdovirus, which is a single stranded RIA virus, The other mnajor illaess in this group is vesicular stomatiis ‘Marke this question €& => Question Td : 141854 Question 19 of 30 A4 year old gitl developed a fever with a cough, runny nose, and pirke eyes for a few days, but this went away. Today she presents with red lesions beginning on the back of her aeck and now spreading to the front and progressing to her trunk. On examination an erythematous, macular rash invoWing the face, neck, thorax and upper arms is seen. Irregular hypopigmented areas on the anterior ‘buccal macosa are also observed. Another organism in the same family as this infective organism is a known cause of which condition? ¥ © a) Bronchiolitis +) California encephalitis ©) Ebola 4) Lymphocytic chosiomeringitie «) Rabies Answer (Explonation | Other User's Explanation Report An Error Question Explanation: ‘The child in the question has measles. It starts with the classic prodrome of cough cotyza, and conjunctivitis, usually with a high fever. [As this dissipates over afew days and prior to the onset of the exanthem is the development of the enanthem, ie. hypopigmented, jsregular oral lesions on the anterior buccal mucosa, These are known as Koplik spots and are shortlived Next, is the erythematous ‘macular rash that begins at the posterior hair ine and behind the ears, spreads anteriorly to the face and then spreads downward to the trunk and extremities, The rash lasts up to ten days. The measles virus is a paramyxovirus, which is a single stranded negative sense RNA vis Other vinses in this group are mumps, parainflienza (infections croup) and respiratory syncytial virus, the major cause of bronchiolitis. California encephalitis is 2 bunyavirus, which is also a single stranded RNA virus The majer illnesses in this group are Calfomia encephaltis, La Crosse encephalitis, and Hantavirus infection Ebola is a flovirus, which is a single stranded ENA virus. The other major virus in this group is the Marburg virus. Lymphocytic choriomeningitis is a member of the arenavirus group, which ica single-stranded RMA virus. The other major illness is Lassa fever. Rabies is a chabdovirus, which is a single stranded RIA virus, The other mnajor illaess in this group is vesicular stomatiis Mark this question <=> Question Id : 144614 Question 20 of 30 A patientis brought to a psychiatsic hospital after having been picked up for making inappropnate sexual advances. A detailed psychiatric history indicates deficits in memory, insight, judgement, personal appearance, and social behavior. The patient is noticed experiencing a possible epileptic seizure, Over a period of several years, motor finding also develop, including relaxed, but expressionless facies, tremor, dysarthria, and pupillarty abnormalities. Test performed on his CSF that would most lkely be dagnostic is @) CSF elucose t) FTA-ABS ¢) Gram stain ) Lymphocyte count ©) Neutrophil count Answer | Explanation Other User's Explanation Report An Error Question Explanati ‘The patient has neurosyphilis, specifically, general paresis, a term that means "general paralysis of the insene." In this late sequela of syphilitic infection, which occurs 5 to 20 years after infection, patients develop mental deterioration, which precedes motor system deterioration, leading eventually to "general pacalysic" with muticm aad incoxtinence. The abnormalities may be conveniently recalled using paresis as amnemenic for personality, alfect, hyperaciive reflexes, Argyl Robertson pupils in the eyes, defects in the sensoxiumn, intelectual dectine and deficient speech. Specific antitreponemal tests such as FTA-ABS are usually positive on both serum and cerebrospinal fuid. In addition there are another three clinical types ofnearosyphilis. The first being acute syphilitic meningitis, which occurs within the first year of infection, and patients present with headache meningeal initation, and cranial nerve abnormalites The second is meningovascular syphilis, of which a petient may present with unilateral numbness paresthesias, vertigo, upper or lower extremity weakness and personality changes. The last is tabes dorsalis, which affects the posterior columns. Symptoms include severe stabbing pain loss of vibration sensation loss of reflexes, and Argyll Robertson pupils (pupils does not sespond to light but contracts with accommodation). The cerebrospinal (CSF) glucose in aeurosyphilis is usually normal. Gram stain of CSF will not demonstrate spirochetes in aeurosyphilc. The CSF lymphocyte comnt is typically elevated in neurocyphilis, but this ir a nonepesifc finding. The CSF neuirophi couat is usually normal in neurosyptilis, Mark this question <=> Question Id : 144614 Question 20 of 30 A patientis brought to a psychiatric hospital afer heving been picked up for making inappropriate sexual advances. A detailed psychiatric history indicates deficits in memory, insight, judgement, personal appearance, and social behavior. The patient is noticed experiencing a possible epileptic seizure. Over a period of several years, moter finding alro develop, including relaxed, but expressionless facies, tremor, dysarthria, and pupillerty abnormalities. Test perfomed on his CSF that would most lkely be diagnostic is a) CSF glucose wv © b)FTA-ABS ¢) Gram stain 6) Lymphocyte count ©) Neutrophil count Answer | Explanation Other User's Explanation Report An Error Question Explanati ‘The patient has neurosyphilis, specifically, general paresis, a term that means "general paralysis of the insene." In this late sequela of syphilitic infection, which occurs 5 to 20 years after infection, patients develop mental deterioration, which precedes motor system deterioration, leading eventually to "general pacalysic" with muticm aad incoxtinence. The abnormalities may be conveniently recalled using paresis as amnemenic for personality, alfect, hyperaciive reflexes, Argyl Robertson pupils in the eyes, defects in the sensoxiumn, intelectual dectine and deficient speech. Specific antitreponemal tests such as FTA-ABS are usually positive on both serum and cerebrospinal fuid. In addition there are another three clinical types ofnearosyphilis. The first being acute syphilitic meningitis, which occurs within the first year of infection, and patients present with headache meningeal initation, and cranial nerve abnormalites The second is meningovascular syphilis, of which a petient may present with unilateral numbness paresthesias, vertigo, upper or lower extremity weakness and personality changes. The last is tabes dorsalis, which affects the posterior columns. Symptoms include severe stabbing pain loss of vibration sensation loss of reflexes, and Argyll Robertson pupils (pupils does not sespond to light but contracts with accommodation). The cerebrospinal (CSF) glucose in aeurosyphilis is usually normal. Gram stain of CSF will not demonstrate spirochetes in aeurosyphilc. The CSF lymphocyte comnt is typically elevated in neurocyphilis, but this ir a nonepesifc finding. The CSF neuirophi couat is usually normal in neurosyptilis, ‘Merk this question & => Question Td : 149458 Question 21 of 30 A person with a history of TV crug abuse has been treated for HIV for the past 4 years. His condition hes been deteriorating over the past 6 months despite varying treatment combination His CDA + counthas been steadily cropping, and he has now developed prolonged ciarhea with fever. Consistent with the diagnosis of full-blown ATDS is which one of the following? 8) Bacillary angiomstosis ) Hairy leukoplakia o) Listeriosis 6) Mycobactesiuin avium complex infection 6) Thrush Anewor (UEQVIRISEN) ter Usors Explanation Report An Error Question Explanation: AIDS can be defined as a CD 4+ count less than 200 cellsfmm3 and a viral load of >55,000 copies/ml or any ofthe AIDS defining conuitions. Pneumocystis pneumonia, candidiasis of the esophagus (trachea, lang), cryptococcosis, Mycobacterium avium intracellulare (MLAI) toxoplasmosis, CMV infection, Keposi sarcoma, and HIV encephelopatiy. In ths question the patient became symptomatic with prolonged diarthea, which i the most commen presenting symptom of MAL. other menifestations are fever and wasting, Bacillary angiomatosis caused by Bartonella hensolae, Sts into the category of a disease that is associated with the easly symptomatic period of HIV infection but not the diagnosis of full-blown AIDS. Hairy leukoplakia is a concttion that is associated with the early syraptomatic period of HIV infection, but not the diagnosis of fill-blown AIDS. Listeriosis is a disease that is associated with the early cymptomatic period of HIV infection, but net the ciggnocie of fill-blown AIDS. Thruch, of oral candidiasis, ie associated with the eauly symptomatic period of HIV infection, but not the diagnosis of fill-blown AIDS. ‘Merk this question & => Question Td : 149458 Question 21 of 30 A person with a history of TV drug abuse has been treated for HIV for the past 4 years. His condition hes been deteriorating over the pact 6 monthe despite varying treatment combination, His CDA + count has beea steadily cropping, and he has now developed prolonged clarshea with fever. Consistent with the diagnosis of fll-blown AIDS is which one ofthe following? 8) Bacillary angiomstasis 'b) Hairy leukoplakia 6) Listeriosis Y¥ © @ Mycobacterium avium complex infection ©) Thrush Anewor (UEQVIRISEN) ter Usors Explanation Report An Error Question Explanation: AIDS can be defined as a CD 4+ count less than 200 cellsfmm3 and a viral load of >55,000 copies/ml or any ofthe AIDS defining conuitions. Pneumocystis pneumonia, candidiasis of the esophagus (trachea, lang), cryptococcosis, Mycobacterium avium intracellulare (MLAI) toxoplasmosis, CMV infection, Keposi sarcoma, and HIV encephelopatiy. In ths question the patient became symptomatic with prolonged diarthea, which i the most commen presenting symptom of MAL. other menifestations are fever and wasting, Bacillary angiomatosis caused by Bartonella hensolae, Sts into the category of a disease that is associated with the easly symptomatic period of HIV infection but not the diagnosis of full-blown AIDS. Hairy leukoplakia is a concttion that is associated with the early syraptomatic period of HIV infection, but not the diagnosis of fill-blown AIDS. Listeriosis is a disease that is associated with the early cymptomatic period of HIV infection, but net the ciggnocie of fill-blown AIDS. Thruch, of oral candidiasis, ie associated with the eauly symptomatic period of HIV infection, but not the diagnosis of fill-blown AIDS. Marl this question = => Question Td: 149611 Question 22 of 30 A 35 year old female presents with fever. vorniting, severe inttative voiding syinptoms. and pronounced costovertebral angle tendemess. Investigation reveals leuicozytosis with a lef chif; blood cultures reveal bacteremia Urinalysis shows pyuria, mild hematuria, and gram-negative bacteria Medication that would best reat this patient's infection include a) Ampicillin and gentamicin b) Exythromycin ©) Gentamicin and vancomycin 6) Phenazopyridine and nitrofurantein 6) Tetracycline Anewor [UEXNSHR) other tsor'sExptan Question Explanation: Acute pyelonephits ican infectious disease involving the kidney pacenchyma and the renal pelvis. Gram-negative bacteria, such as Escherichia coli, Proteus, Klebsiella, and Enterobacter, are the most common causative organisms in acute pyclonephtis Laboratory evaluation wil often reveal leukocytosis with a left shit, and urinalysis typically shows pyutia, varying degrees of hematuria, and white cell casts. Since bacteremia is present, the patient should be hospitalized and empirically started on IV ampicilin and gentamicin. This regimen may aved to be chenged, however, oxce the sensitivity results are available. Erythromycin and tetracycline are both bacteriostatic antibiotics and would not be recommended in a patient with a severe infection, such as acute pyelonephritis with bacteremia Vancomycin is primarily used in the treatment of severe gram positive infections. Phenazopyricine is a urinary analgesic, and nitrafirantain is a urinary tract anti-infective. Although nitvofurantoin is indicated far the treatmert of "mild" cases of pyelonephritis, as well as cystitis, this patient's contltionis severe and should be treated with appropriate antibiotics jon Report An Error Marl this question = => Question Td: 149611 Question 22 of 30 ‘A.35 year old female presents with fever. vomting, severe iitative voiding symptoms, and pronourced costovertebral angle tenderness. Investigation reveals leukocytosis with a let chif; blood cultures reveal bacteremia Urinalysis shows pyuria, mild hematuria, and gram-negative bacteria Medication that would best creat this patient's infection include Y © a Ampiciin and gentamicin b) Erythromycin ©) Gentamicin and vancomycin @ Phenazopyridine and nitrofurantoin @) Tetracycline Anewor [UEXNSHR) other tsor'sExptan Question Explanation: Acute pyelonephits ican infectious disease involving the kidney pacenchyma and the renal pelvis. Gram-negative bacteria, such as Escherichia coli, Proteus, Klebsiella, and Enterobacter, are the most common causative organisms in acute pyclonephtis Laboratory evaluation wil often reveal leukocytosis with a left shit, and urinalysis typically shows pyutia, varying degrees of hematuria, and white cell casts. Since bacteremia is present, the patient should be hospitalized and empirically started on IV ampicilin and gentamicin. This regimen may aved to be chenged, however, oxce the sensitivity results are available. Erythromycin and tetracycline are both bacteriostatic antibiotics and would not be recommended in a patient with a severe infection, such as acute pyelonephritis with bacteremia Vancomycin is primarily used in the treatment of severe gram positive infections. Phenazopyricine is a urinary analgesic, and nitrafirantain is a urinary tract anti-infective. Although nitvofurantoin is indicated far the treatmert of "mild" cases of pyelonephritis, as well as cystitis, this patient's contltionis severe and should be treated with appropriate antibiotics jon Report An Error Mark this question & => Question Td : 201435 Question 23 of 30 Antiretroviral medication that is licely to cause increased pigmentation of the skin in a black Aftican patient is which one of the following? a) Didanosne ) Bfevirens ©) Emtricitebine a) Nevirapine «) Stavudine Answer | Explanation Other User's Explanation Report An Error Question Explanation: Emiricitabine causes hyper-pigmentation of skin, including palmar creases in 8% of black patients. Didanosine and stavudine cause tnitochandrial toxicity, hence peripheral neuropathy, pancreatitis and hyperlactataersia, Bfavireaz causes CNS toxicity. Nevirapine causes acute hepattis and skin rash Mark this question & => Question Td : 201435 Question 23 of 30 Antiretroviral medication that is licely to cause increased pigmentation of the skin in a black Aftican patient is which one of the following? a) Didanosne ) Bfevirens Y © 0) Eniricitebine a) Nevirapine «) Stavudine Answer | Explanation Other User's Explanation Report An Error Question Explanation: Emiricitabine causes hyper-pigmentation of skin, including palmar creases in 8% of black patients. Didanosine and stavudine cause tnitochandrial toxicity, hence peripheral neuropathy, pancreatitis and hyperlactataersia, Bfavireaz causes CNS toxicity. Nevirapine causes acute hepattis and skin rash Mark this question —& => Question Td : 203422 Question 24 of 30 A 36 year oldman presents 6 months after a cadaveric renal allograft. He receives azathicprine and prednisolone. For the past week he has fever of 38.6°C, anoresia and a cough productive of thick green sputum, CXR shows aleft lower lobe nodule of approximately 5 cm diameter with central cavitation. Sputum analysis reveals long, crocked, branching and beaded grem positive flaments. What is the most appropriate initial therapy for this patient? 2) Ceftasichme 6) Co-ameniclay ©) Co-tinoxazcle ) Erythromycin, ) Rifampicin and isoniazid Question Explanation: The likely diagnosis is nocardiosis. Nocardia are aerobic, Gram-positive branching flamentous bacteria which often appear beaded on staining. Nocardiosis can be diagnosed rapidly by examination of sputum or pus with the Gram stain and a modified acid-fast stain, Paeumoniais typically found in the irerainecoraproreiced, as inthis case and may be a single lesion or exelensive paeumonic consolidation. The drug of choice is trimethoprim-sulfamethoxazole. Mark this question —& => Question Td : 203422 Question 24 of 30 A 36 year old man presents 6 months afer a cadaveric renal allograft. He receives azathioprine and prednisolone, For the past week hae has fover of 38.6°C, ancresia and a cough productive of thick green sputum. CR shows alett lower lobe aodule of approximately 5 cm diameter with central cavitation. Spunan analysis reveals long, crocked, branching and beaded gram positive laments. What is the most appropriate initial therapy for this patient? 2) Ceftazidime 6) Co-ameniclay VW © ©) Co-trimoxazole ) Erythromycin: ©) Riftrmpicin and isoniazid Question Explanation: The likely diagnosis is nocardiosis. Nocardia are aerobic, Gram-positive branching flamentous bacteria which often appear beaded on staining. Nocardiosis can be diagnosed rapidly by examination of sputum or pus with the Gram stain and a modified acid-fast stain, Paeumoniais typically found in the irerainecoraproreiced, as inthis case and may be a single lesion or exelensive paeumonic consolidation. The drug of choice is trimethoprim-sulfamethoxazole. Mark this question e& => estion Td : 203442 Question 25 of 30 A woman aged 23 years presents with a vaginal discharge. She is in her 16th week of preanancy Farther investigation confirms infection with Chlamydia trachomatis. The most appropriate treatment for her is which one of the following? a) Azithromycin ) Ciprofloxacin ©) Co-trimoxazole & Doxycycline #) Metronidazole Question Explanation: C. trachomatis infection is common in pregnancy and is associated with adverse fetal outrome incinding + spontaneous miscarriage © premature rupture of membranes and © intrauterine growth retardation (UGR) “Appropriate treatment is with macrolides such as erythromycin or azithromycin Both doxycycline and ciprofloxacin are conraindivated in pregnancy. ‘Marke this question <>=> Question 25 of 30 A woman aged 23 years presents with a vaginal discharge. She is in her 16th week of preanancy Farther investigation confirms infection with Chlamydia trachomatis. The most appropriate treatment for her is which one of the following? Y © a) Anthromycin ) Ciprofloxacin ©) Co-trimoxazole & Doxycycline #) Metronidazole Question Explanation: C. trachomatis infection is common in pregnancy and is associated with adverse fetal outrome incinding Report An Error + spontaneous miscarriage © premature rupture of membranes and © intrauterine growth retardation (UGR) “Appropriate treatment is with macrolides such as erythromycin or azithromycin Both doxycycline and ciprofloxacin are conraindivated in pregnancy. ‘Mark this question & => Question Td : 206274 Question 26 of 30 ‘True about Giardia lamblia infection is which one of the following? a) Ibis offen symptomatic ) Ibis usually spread by contaminated meats ©) Itis eradicated by mebendazole ) d #8) Itis diagnosed by stool culture Question Explanation: It causes steatorrhea Usually acquired by the facco-oral routs; Many individuals excreting cysts are asymptomatic and are thus carriers. Others have diarthoea, steatorrhoea, abdominal pain and nausea. Diagnosed by stool mictoscopy-if negatwe, paresite found in duodenal aspirates of biopsy. Testing of serum antbodies against G lamblia trophozoites is not usefil in diagnesing current infection, Eradicete with metronidazole (or quinacrine, tinidazole, omidazole, furazolidone, paromomycin). Mebendazole is used in treating hookeworm infections eg ascavis, whipworm and threadworm, ‘Mark this question & => Question Td : 206274 Question 26 of 30 ‘True about Giardia lamblia infection is which one of the following? a) Ibis offen symptomatic ) Ibis usually spread by contaminated meats 0) Ibis eradicated by mebendazole Y © d) It causes steatorrhea #8) Itis diagnosed by stool culture Question Explanation: Usually acquired by the facco-oral routs; Many individuals excreting cysts are asymptomatic and are thus carriers. Others have diarthoea, steatorrhoea, abdominal pain and nausea. Diagnosed by stool mictoscopy-if negatwe, paresite found in duodenal aspirates of biopsy. Testing of serum antbodies against G lamblia trophozoites is not usefil in diagnesing current infection, Eradicete with metronidazole (or quinacrine, tinidazole, omidazole, furazolidone, paromomycin). Mebendazole is used in treating hookeworm infections eg ascavis, whipworm and threadworm, Mark this question & => Question Td : 206324 Question 27 of 30 A 15 year old boy has a2 week history of general malaise, fatigue and pharyngitis. On exam muluple smell lymph nodes were palpable in the neck, axillae and groins. Labs show Hb of 12 SgfdL., WBC count of 16 0 210%, platelet court of 160 x1 0°AT. and blood film shows lymphocytosis. The likely diagnosis is a) Acute lymphoblastic levicaemia ) Cytomegalovirus infection ©) Epstein-Barr virus infection 4) Hodgkin's diseese €) Toxoplasmosis Question Explanation: “Acute EBV typically presents with a history of 1-2 weeks offatigue a malaise, fever, pharyngitis, and symmetrical, bilateral igmmphadenopathy. Heterophil antibody tesis are usually positive. Mild transient thtombocytopaenia is not uncommon in EBV infectious mononucleosis, CMV monomucleosis has a lower incidence of pharyngitis and cervical adenopathy, Primary toxoplasmosis is acquired vie ingestion of undercooked meat containing toxoplasma cysts, of ingestion of ech food contaminated toxoplasma excreted in cats faeces. The infection is asymptomatic in 80-90% of immunocompetent patients. Highly characteristic of toxoplasmosis is asyrametrical lymphadenopathy limited to an isolated lymph nede group. Patients with toxoplasmosis have little or a0 fever. fatigue, or pharyngitis. CMV infectious mononucleosis may be indistinguishable in clinical presentation from EBV but s usvally not accompanied by posterior cervical adenopathy, non-exudative pharynsitic ir minimal or abcent. The cagnosis of ALL and HD is made by a combination of blood film examination, bone marrow aspiration and biopsy and lymph nede biopsy Mark this question & => Question Td : 206324 Question 27 of 30 A 15 year old boy has a2 week history of general malaise, fatigue and pharyngitis. On exam muluple smell lymph nodes were palpable in the neck, axillae and groins. Labs show Hb of 12 SgfdL., WBC count of 16 0 210%, platelet court of 160 x1 0°AT. and blood film shows lymphocytosis. The likely diagnosis is a) Acute lymphoblastic levicaemia ) Cytomegalovirus infection Y © 0) Epstein-Barr virus infection 4) Hodgkin's diseese €) Toxoplasmosis Question Explanation: “Acute EBV typically presents with a history of 1-2 weeks offatigue a malaise, fever, pharyngitis, and symmetrical, bilateral igmmphadenopathy. Heterophil antibody tesis are usually positive. Mild transient thtombocytopaenia is not uncommon in EBV infectious mononucleosis, CMV monomucleosis has a lower incidence of pharyngitis and cervical adenopathy, Primary toxoplasmosis is acquired vie ingestion of undercooked meat containing toxoplasma cysts, of ingestion of ech food contaminated toxoplasma excreted in cats faeces. The infection is asymptomatic in 80-90% of immunocompetent patients. Highly characteristic of toxoplasmosis is asyrametrical lymphadenopathy limited to an isolated lymph nede group. Patients with toxoplasmosis have little or a0 fever. fatigue, or pharyngitis. CMV infectious mononucleosis may be indistinguishable in clinical presentation from EBV but s usvally not accompanied by posterior cervical adenopathy, non-exudative pharynsitic ir minimal or abcent. The cagnosis of ALL and HD is made by a combination of blood film examination, bone marrow aspiration and biopsy and lymph nede biopsy Mark this question & => Question Td : 206817 Question 28 of 30 A.31 year old diabetic male has pain in his left hand, fever and a rash for 2 days that began after pricking his finger in the garden. His HbAlc is 7.3%. Physical exam shows a painful, red and swellen middle finger with the redness exttanding to the metacarpophalangeal joint. He is diagnosed with cellulitis and is prescribed antibiotics. Tae most likely infective organism is a) Escherichia coi b) Elebsiella o) MRSA & Pseudomonas aeruginosa ¢) Streptococens pyogenes Question Explanation: “This patient has a community acquired celultis which is mostlikely due to either Strep pyogenes or Staph aureus, but not methicillin resistant Staphylococcus aureus (MRSA) which is more of a hospital pathogen and tends notto cause cellulitis. Understanding the likely infective organism can dictate the appropriate antibiotic therapy. Ta this case, oral fiuclozacilin and penicillin V are appropriate Tthe cellulitis does not improve quickly then intravenous antibiotics may be required, Mark this question & => Question Td : 206817 Question 28 of 30 A.31 year old diabetic male has pain in his left hand, fever and a rash for 2 days that began after pricking his finger in the garden. His HbAlc is 7.3%. Physical exam shows a painful, red and swellen middle finger with the redness exttanding to the metacarpophalangeal joint. He is diagnosed with cellulitis and is prescribed antibiotics. Tae most likely infective organism is a) Escherichia coi b) Elebsiella o) MRSA & Pseudomonas aeruginosa VY © 2) Streptococcus pyogenes Question Explanation: “This patient has a community acquired celultis which is mostlikely due to either Strep pyogenes or Staph aureus, but not methicillin resistant Staphylococcus aureus (MRSA) which is more of a hospital pathogen and tends notto cause cellulitis. Understanding the likely infective organism can dictate the appropriate antibiotic therapy. Ta this case, oral fiuclozacilin and penicillin V are appropriate Tthe cellulitis does not improve quickly then intravenous antibiotics may be required, 3/1/2014 6:28:05 PM. € wovw.interfaceedu.ck/i CL www. interface.edu.pk/medi xams/t inalysis pho utid= 14759 apps [J cosgle S settings FiSonin 2. ube dul © yo ueilléaye Glutio ue gosoll JS [1 Free Hotmall * 8 Ci other bookmar 16867 ‘Mark this question er- Question Td: Question 29 of 30 A.21- year old male has fever, hemoptysis, wheesing, and dificubyy breathing, Respiratory rate =30fmin, There is urticaria on his trunk. CXR shows several bilateral patchy infltrates more in the nat lung, CBC shows 20% eosinophils. In addition to testing for the etiology, what should be prescribed now? a) Albendazole 400 mg as a single dose +b) Ivermestia, 200 mgfeg orally repeated every 6 monthe c) Levamisole, 150 mg orally as a single dose ) Metendazole, 500 orally ac a single doce ¢) Pyrantel pamoate 11 ghkg as a single dose 8) Thiabendazole, 25 mg/kg orally twice daily for 3 days Avmwor (UEIUIRNMIY) other Users Explanation Report AnEnor Question Explanation: The causative organism in this cace is Ascarie lnmbriooides. Ite the moct common iatectinal helminthic infection and occurs on ‘worldwide basis. There is a high prevalence in the southeastern United States or anywhere that standards of hygiene end sanitation are poor, or where human feces is used as a fertizer. Lung involvement presents as low-grade fever, nonproductive cough, hemoptysis. wheezing. dyspnea, and pain. A hypersensitivity reaction may result in urticaria, With large numbers of worms in the intestine, abdominal discomfort may be scen. Worms may also cause eymptoms by migrating to the ccmmon bile duct, pancreatic duct, appendix, diverticula, and other sites, including brain, kidney, eye, and spinal cord. Masses of worms may cause intestinal obstruction, intussusception, and death. Chest radiographs may show transient, patchy, asymmetric infiltrates. During the pulmonary phase, peripheral eosinophils may be as high as 30 to 50% and sputum analysis may show eosinophils as well as Charcot-Leyden, crystals (composed of an eosinophil-derived enzyme) as with any other parasitic nrg infections Asingle dose of mebendazole is the treatment of choice for eradicating Ascaris in non-pregnant patients. Ibis also a first line drug in Enterobins vermsiculeris, Trichuris tricuria and Ancylastoma duodenale infestations. This patient should also receive corticosteroid for his pulmonary symptoms. However, eradication of Ascaris should be done promptly to avoid very serious GI manifestations of the disease Albendazole has high activity against Ascaris but is not approved in the United States for this indication. Itis indicated as treatment for neurocysiicercosis (Taena solum) and cystic hydatid disease (Echinococcus granulosus) Ivermectin like albendazole has shown to be effective against Ascaris however itis not approved as the treatment of choice and is not used very widely. Levamnisole hias also been shown as an effective form of therapy against Ascaris but mebendazole is considered the first ine drug Levamnisole is available in the United States but its role is stl under investgetion, Pyrantel pamtoate is an alternative therapy in preanant women and can be used as a sinale dose. However, in non-pregnant patients mebendazole is preferred over pyrantel pamoate asi has high efficacy against Asceris Thiabendezole has a wide spectrum of anthelminthic activty, andis used for enterobiasis, Strongyloides, cutaneous larva migrans, and thooleworm. and dunne the invasive stage of tichinosis. Total Questions 1 No RO mh F bbHaGbEBBRELB BERBRBREBRBBE XKXXKKXKRKKKKRKAKRKARKAKRKAKRAKKMAKRKAKRRKKER 3/1/2014 6:28:05 PM. € wovw.interfaceedu.ck/i CL www. interface.edu.pk/medi xams/t inalysis pho utid= 14759 apps [J cosgle S settings FiSonin 2. ube dul © yo ueilléaye Glutio ue gosoll JS [1 Free Hotmall * 8 Ci other bookmar 16867 ‘Mark this question er- Question Td: Question 29 of 30 A.21- year old male has fever, hemoptysis, wheesing, and dificubyy breathing, Respiratory rate =30fmin, There is urticaria on his trunk. CXR shows several bilateral patchy infltrates more in the nat lung, CBC shows 20% eosinophils. In addition to testing for the etiology, what should be prescribed now? a) Albendazole 400 mg as a single dose +b) Ivermestia, 200 mgfeg orally repeated every 6 monthe c) Levamisole, 150 mg orally as a single dose Y © d) Mebendazole, 500 orally as a single dose ¢) Pyrantel pamoate 11 ghkg as a single dose 8) Thiabendazole, 25 mg/kg orally twice daily for 3 days Avmwor (UEIUIRNMIY) other Users Explanation Report AnEnor Question Explanation: The causative organism in this cace is Ascarie lnmbriooides. Ite the moct common iatectinal helminthic infection and occurs on ‘worldwide basis. There is a high prevalence in the southeastern United States or anywhere that standards of hygiene end sanitation are poor, or where human feces is used as a fertizer. Lung involvement presents as low-grade fever, nonproductive cough, hemoptysis. wheezing. dyspnea, and pain. A hypersensitivity reaction may result in urticaria, With large numbers of worms in the intestine, abdominal discomfort may be scen. Worms may also cause eymptoms by migrating to the ccmmon bile duct, pancreatic duct, appendix, diverticula, and other sites, including brain, kidney, eye, and spinal cord. Masses of worms may cause intestinal obstruction, intussusception, and death. Chest radiographs may show transient, patchy, asymmetric infiltrates. During the pulmonary phase, peripheral eosinophils may be as high as 30 to 50% and sputum analysis may show eosinophils as well as Charcot-Leyden, crystals (composed of an eosinophil-derived enzyme) as with any other parasitic nrg infections Asingle dose of mebendazole is the treatment of choice for eradicating Ascaris in non-pregnant patients. Ibis also a first line drug in Enterobins vermsiculeris, Trichuris tricuria and Ancylastoma duodenale infestations. This patient should also receive corticosteroid for his pulmonary symptoms. However, eradication of Ascaris should be done promptly to avoid very serious GI manifestations of the disease Albendazole has high activity against Ascaris but is not approved in the United States for this indication. Itis indicated as treatment for neurocysiicercosis (Taena solum) and cystic hydatid disease (Echinococcus granulosus) Ivermectin like albendazole has shown to be effective against Ascaris however itis not approved as the treatment of choice and is not used very widely. Levamnisole hias also been shown as an effective form of therapy against Ascaris but mebendazole is considered the first ine drug Levamnisole is available in the United States but its role is stl under investgetion, Pyrantel pamtoate is an alternative therapy in preanant women and can be used as a sinale dose. However, in non-pregnant patients mebendazole is preferred over pyrantel pamoate asi has high efficacy against Asceris Thiabendezole has a wide spectrum of anthelminthic activty, andis used for enterobiasis, Strongyloides, cutaneous larva migrans, and thooleworm. and dunne the invasive stage of tichinosis. Total Questions 1 No RO mh F bbHaGbEBBRELB BERBRBREBRBBE XKXXKKXKRKKKKRKAKRKARKAKRKAKRAKKMAKRKAKRRKKER 3/1/2014 6:28:38 PM € wovw.interfaceedu.ck/i CL www. interface.edu.pk/medical-exarns /test-analysis.pho ?utd=14759 apps El.coogle Asetings [)Sgnin Zou woe duly oo wsdl dips Ofte ge sgsoll JS) Free Hatmall * 8 Gi other bookmar ‘Mark this question & Question Td : 216933 Question 30 of 30 A. 39-year-old man has a skin rash for 5 days starting with en initial red "bump" after retuming from a camping trip to mountains. It hhas been spreading since. He has headache and sight joint pain and tenderness but no impaired range ofmotion, There is an erythematous annular plaque 15 om n diameter on his sight flark. Tis centered by a resolving petechial macule. Which vectoris the mostlikely? a) Amblyomma americanum b) Dentnacentor andersori c) Lxodes scapularis 4d) Loxosceles recluse €) Pediculas humanus conporis Answer (Boisnation) Other User's Explanation Report An Error Question Explanation: This patient has erythema miarans, a manifestation of Lyme disease. Erythema migrans develops 3 to 32 days after the bite of Ixodes scapularis (a deer tick) infected by Borrelia burgdorfen. The rash begins as a papule that expands to form an erythematous arnular lesion with central clearing I:may be accompenied by malaise, lethargy, fever, and arthralgias. Ticks are found in grass, shrubs, vines, and bushes, from which they attach themselves to dogs, deer, cattle, and humans. Prompt removal of the attached tick within the first 8 to 12 hours minimizes the chance of transmission of the disease Appropriate follow-up blood werk should be done to evaluate farti- Borrelia antibody titers are increasing and treatment with anibiotics is necessary. In the United States, the most common vector of Lyme disease is Irodes scapularis Txodes pacificus is also a possible transmitter. Txodes ticks are also vectors of babesiosis and Ehrlichia phagocytophila Amblyomma americanum or the Lone Start tick is the vector for ehrlichiosis, a disease produced by Ehrlichia chaffeensis. Tis similar clinically to Rocky Mountain spotted fever but generally without the rash and without serious complications. Amblyomma americanum can also transmit tularemia and Southern tick-associated rash ilness (STARD). ‘Dermacentor andersom or the wood tick is a vector for Rocky Mountain spotted fever, tularemia, and Colorado tick fever It also may cause tick paralysis. The other Dermacentor species that act as vectors of these diseases is Dermacentor variebilis Loxosceles rechisa or the brown recluse spider is a hunting spider found throughout the United States. It is not a vector for any skin disceses, however, it inicts painfil bite that evolves into a necrotic eschar within days, which is accompanied by systemic symptoms ofintomcation with spider venom, Pediculus hummus corporis or the body louse has been implicated as the vector of epidemic typhus, trench fever, and louse-bome relapsing fever. Itis found in seams of clothes and descends to the skin for blood meals. Total Questions SBBARBEBSBEBRERBREKEKBBEBY PR eee eee XxxXXXKXXKXKXKKXKXKXKKKKXKKXKKXKKXKKXKKKKXKK 3/1/2014 6:28:38 PM € wovw.interfaceedu.ck/i CL www. interface.edu.pk/medical-exarns /test-analysis.pho ?utd=14759 apps El.coogle Asetings [)Sgnin Zou woe duly oo wsdl dips Ofte ge sgsoll JS) Free Hatmall * 8 Gi other bookmar ‘Mark this question & Question Td : 216933 Question 30 of 30 A. 39-year-old man has a skin rash for 5 days starting with an initial red "bump" after retuming from a camping trip to mountains. It fhas been spreading since. He has headache and sight joint pain and tenderness but no impaired range oftmotion. There is an erythematous annular plaque 15 cm in diameter on his right flank. It is centered by a resolving petechial macule. Which vector is the most likely? a) Amblyornma americanum b) Dermacentor andersori Y © oc) Lzodes scapularis d) Loxosceles recluse €) Pediculas hnmanus corporis Answer (Boisnation) Other User's Explanation Report An Error Question Explanation: This patient has erythema miarans, a manifestation of Lyme disease. Erythema migrans develops 3 to 32 days after the bite of Ixodes scapularis (a deer tick) infected by Borrelia burgdorfen. The rash begins as a papule that expands to form an erythematous arnular lesion with central clearing I:may be accompenied by malaise, lethargy, fever, and arthralgias. Ticks are found in grass, shrubs, vines, and bushes, from which they attach themselves to dogs, deer, cattle, and humans. Prompt removal of the attached tick within the first 8 to 12 hours minimizes the chance of transmission of the disease Appropriate follow-up blood werk should be done to evaluate farti- Borrelia antibody titers are increasing and treatment with anibiotics is necessary. In the United States, the most common vector of Lyme disease is Irodes scapularis Txodes pacificus is also a possible transmitter. Txodes ticks are also vectors of babesiosis and Ehrlichia phagocytophila Amblyomma americanum or the Lone Start tick is the vector for ehrlichiosis, a disease produced by Ehrlichia chaffeensis. Tis similar clinically to Rocky Mountain spotted fever but generally without the rash and without serious complications. Amblyomma americanum can also transmit tularemia and Southern tick-associated rash ilness (STARD). ‘Dermacentor andersom or the wood tick is a vector for Rocky Mountain spotted fever, tularemia, and Colorado tick fever It also may cause tick paralysis. The other Dermacentor species that act as vectors of these diseases is Dermacentor variebilis Loxosceles rechisa or the brown recluse spider is a hunting spider found throughout the United States. It is not a vector for any skin disceses, however, it inicts painfil bite that evolves into a necrotic eschar within days, which is accompanied by systemic symptoms ofintomcation with spider venom, Pediculus hummus corporis or the body louse has been implicated as the vector of epidemic typhus, trench fever, and louse-bome relapsing fever. Itis found in seams of clothes and descends to the skin for blood meals. Total Questions SBBARBEBSBEBRERBREKEKBBEBY PR eee eee XxxXXXKXXKXKXKKXKXKXKKKKXKKXKKXKKXKKXKKKKXKK ‘Marke this question => Question Ta : 30955 Question 1 of 30 An 80 year old female with dementia presents from a nursing home with vaginal discharge. High vaginal swabs confinm gonortheal infection and she has been commenced on appropriate treatment. What else would you like to do for her’? 2) Contact tracing ) Inform police ©) None of the above ¢) Inform family ©) Non-officia enquiry Answer (Exvionaton | Other User's Explanation Report An Error Question Explanation: This appears to be case of elder sexual abuse For those who are legally competent, voluntary referrals to social services, victims’ services the criminal justice eystem or any other appropriate resource agencies are encourage. Doctors should not report the incidence without the patient's permission Here this cemented lady is not legally competent. In this case one should discuss this case with the next of kin first to establish if there truly is sexual abuse or whether in fact thic patient has heen having a relationship with a third party. The facts should be established rst before involving the police. IFin any doubt, contact your medical defence oragnisation. ‘Marke this question => Question Ta : 30955 Question 1 of 30 An 80 year old female with dementia presents from a nursing home with vaginal discharge. High vaginal swabs confinm gonortheal infection and she has beea commenced on appropriate treatment. What else wanld you like ta do for her? 2) Contact tracing +) Inform police ©) None of the above Y¥ © @) Inform family ©) Non-officia enquiry Answer (Exvionaton | Other User's Explanation Report An Error Question Explanation: This appears to be case of elder sexual abuse For those who are legally competent, voluntary referrals to social services, victims’ services the criminal justice eystem or any other appropriate resource agencies are encourage. Doctors should not report the incidence without the patient's permission Here this cemented lady is not legally competent. In this case one should discuss this case with the next of kin first to establish if there truly is sexual abuse or whether in fact thic patient has heen having a relationship with a third party. The facts should be established rst before involving the police. IFin any doubt, contact your medical defence oragnisation. 3/1/2014 6:30:11 PM Mark this question <= => Question Td : 48943 Question 2 of 30 A vroman attacked by a bee comes to emergency room with hypotension, tachycarcia and visible allergic reaction with swelling The most appropriate initial management is which of the following? a) Antihistamine ) Epinephrine injection ©) Saline perfusion 4) Intubation 6) IVF steroid Anowor (UBIGIRNBTNY otner Users Exton Question Explanation: “Anaphylactic reactions (anaphylaxis) are sudden, widespread, potentially severe and life-threatening allergic reactions. Anaphylactic reactions begin within 1 to 15 mines of exposure to the allergen. Rarely, reactions begin after 1 hour. The heart beats quickly The persoa may feel uneasy and become agitated. Blood pressure may fall, causing fainting, Other symptoms include tingling (pixs-and- needles) sensations, itchy and flushed skin, throbbingin the ears, coughing, sneezing, hives, and eweling (angioedema). IFan anaphylactic reaction occurs an epinephrine injection should be given immediately. Report An Error 3/1/2014 6:30:11 PM Mark this question <= => Question Td : 48943 Question 2 of 30 A. woman attacked by a hee comes to emergency room with hypotension, tachycardia and visible allergic reaction with ewelling The most appropriate initial management is which ofthe following? a) Antihistamine Y © >) Epinephrine injection 6) Saline perfusion 4) Intubation 6) IV stercid Anowor (UBIGIRNBTNY otner Users Exton Question Explanation: “Anaphylactic reactions (anaphylaxis) are sudden, widespread, potentially severe and life-threatening allergic reactions. Anaphylactic reactions begin within 1 to 15 mines of exposure to the allergen. Rarely, reactions begin after 1 hour. The heart beats quickly The persoa may feel uneasy and become agitated. Blood pressure may fall, causing fainting, Other symptoms include tingling (pixs-and- needles) sensations, itchy and flushed skin, throbbingin the ears, coughing, sneezing, hives, and eweling (angioedema). IFan anaphylactic reaction occurs an epinephrine injection should be given immediately. Report An Error ‘Marke this question & => Question Td : 60238 Question 3 of 30 A 44-year-old male with acute onset of symptoms relating to meningitis is brought to the emergency medicine department After lumbar puncture end appropriate laboratory testing, it is fond that he is infected by Streptococcus Pneumonia. Which one ofthe following represents the first-line parenteral drug therapy in this individual? a) Penicilin b) Cefotatime ©) Ceftriaxone 4) Cefandime @) None of the above Anaver [RESRIREHBA) other Users Explanation Report An Esror Question Explanation: Penicilin is the first-line drug therapy for Streptococcus Preumeniae meningitis. For meringitides associated with H. Influenzae infection or Neisseria Meningitis infection, cefotaxime, ceftriaxone, or another thrd- generation cephalosporin is indicated. ‘Marke this question & => Question Td : 60238 Question 3 of 30 A 44-year-old male with acute onset of symptoms relating to meningitis is brought to the emergency medicine department After lumbar puncture end appropriate Inboratory testing, it is Found that he is infocted by Streptococcus Paoumoniae. Which one ofthe following represents the first-line parenteral drug therapy in this individual? VY © a) Penicilin ) Cefotaximae ©) Ceftriazone 4) Cefiandime ©) None of the above Anaver [RESRIREHBA) other Users Explanation Report An Esror Question Explanation: Penicilin is the first-line drug therapy for Streptococcus Preumeniae meningitis. For meringitides associated with H. Influenzae infection or Neisseria Meningitis infection, cefotaxime, ceftriaxone, or another thrd- generation cephalosporin is indicated. Marke this question << => Question Td = 63394 Question 4 of 30 Hepatitis B infection is characterized by all of the following EXCEPT a) Hepatitis B is transmitted primarily via the parenteral route ) pre-SI and pre-S2 are important in virion attachment to and penetration of the hepatocyte. ) Pre-core mutants have been associated with more aggressive acute or chronic clinical courses 4) Tehas a high mortality rate among pregnant women, €) HBeAg positivity confers increased infectivty to HBsAg positive serutn Question Explanation: ‘A high mortality rate among pregnant women is true about Hepatitis E. Hepatitis B is a DNA virus which is transmitted predominartly by the parenteral route. The major antigenic determinant HBsAg consists of three co-terminal protein components: S, pre-S1, and pre-S2. Of these, the pre $1 and pre-S2 are important in vition attachment and penetration of the hepatocy:e, HBeAg positivity confers increased infectivity to persons with HBsAg-positive serum “Although anti-HBe may indicate acute infection, it may also persist in the circulation for years. Particularly in Europe, pre-core mutants have been described and are associated with both a more agoressive acute and chronic clinical picture. Marke this question << => Question Td = 63394 Question 4 of 30 Hepatic B infection is characterized by all of the following EXCEPT 2) Hepatitis B is transmitted primaniy via the parenteral route +) pre-S1 and pre-$2 are important in virion attachment to and penetration of the hepatocyte. ¢) Pre-core mutants have been associated with more aggressive acute or chronic clinical courses ¥ © d)Kthas a high mortality rate among pregnant women ©) HBeAg positivity confers increased infectivity to HBsAg positive serum. Question Explanation: ‘A high mortality rate among pregnant women is true about Hepatitis E. Hepatitis B is a DNA virus which is transmitted predominartly by the parenteral route. The major antigenic determinant HBsAg consists of three co-terminal protein components: S, pre-S1, and pre-S2. Of these, the pre $1 and pre-S2 are important in vition attachment and penetration of the hepatocy:e, HBeAg positivity confers increased infectivity to persons with HBsAg-positive serum “Although anti-HBe may indicate acute infection, it may also persist in the circulation for years. Particularly in Europe, pre-core mutants have been described and are associated with both a more agoressive acute and chronic clinical picture. Mark this question & => Question Id : 67376 Question S of 30 Patients suffering from acqured immunodeficiency syndrome (AIDS) can have chontoretnitis with blindness, enteritis with intractable darshea, interstitial pneumonitis, and adrenalitis, all caused by infection with which of the following? 8) Cryptosporidinm ) Heapes zoster ©) Cytomegalovirus 6) Toxoplasma 6) Pneumocystis Question Explanation: Anyone can become infected with CMV. Almost all people have been exposed-to CMV by the time they are adults, but the virus usually does not make otherwise healthy people sick. However, some people are at increased rise for active infection and serious complications: Babies born to women who have a first-time CMY infection during pregnancy, pregnant women who work with infants and children, Persons with weakened immune systents, including cancer patients on chemotherapy, organ transplant recipients, and persons with HIV infection Active infection in otherwise healthy chilcren and aduits can cause prolonged high fever, chils, severe tiredness, a generally sill feeling, headache, and an enlarged spleen. Most infected newborns have no syreptoms at birth, but, in some cases, symptoms will appear over the aczt several years. These inclads mental and developmental problems and vision or hearing problems Tn rare cases, a newbom can have a life- threatening infection at birth. Infants and children who get CMV infection after birth have few, any, symptoms or complications. When symptoms do appear, they include hng problems, poor weight gain, swollen dlands, rash, liver problems, and blood problems People with weakened immane systems can have more serious, potentially life-threatening illnesses, with fever, pneumonia, liver infection, and anemia, Inesses can last for weeles or months and can be fatal. In persons with HIV infection, CMY can infect the retina of the eye (CMV retinitis) and cause blindness, ion Report An Error Mark this question & => Question Id : 67376 Question S of 30 Patients suffering from acqured immunodeficiency syndrome (AIDS) can have chontoretnitis with blindness, enteritis with intractable darshea, interstitial pneumonitis, and adrenalitis, all caused by infection with which of the following? 8) Cryptosporidinm ) Heapes zoster V © 0) Cytomegalovirus 6) Toxoplasma 6) Pneumocystis Question Explanation: Anyone can become infected with CMV. Almost all people have been exposed-to CMV by the time they are adults, but the virus usually does not make otherwise healthy people sick. However, some people are at increased rise for active infection and serious complications: Babies born to women who have a first-time CMY infection during pregnancy, pregnant women who work with infants and children, Persons with weakened immune systents, including cancer patients on chemotherapy, organ transplant recipients, and persons with HIV infection Active infection in otherwise healthy chilcren and aduits can cause prolonged high fever, chils, severe tiredness, a generally sill feeling, headache, and an enlarged spleen. Most infected newborns have no syreptoms at birth, but, in some cases, symptoms will appear over the aczt several years. These inclads mental and developmental problems and vision or hearing problems Tn rare cases, a newbom can have a life- threatening infection at birth. Infants and children who get CMV infection after birth have few, any, symptoms or complications. When symptoms do appear, they include hng problems, poor weight gain, swollen dlands, rash, liver problems, and blood problems People with weakened immane systems can have more serious, potentially life-threatening illnesses, with fever, pneumonia, liver infection, and anemia, Inesses can last for weeles or months and can be fatal. In persons with HIV infection, CMY can infect the retina of the eye (CMV retinitis) and cause blindness, ion Report An Error ‘Mark this question & => Question Td : 72643 Question 6 of 30 A 29-year-old HIV-positive patient presents with a pneumonia suspected to be caused by Pneumocystis carinii, Which one of the following antitricrobial agents would be most applicable in the treatment ofthis disease? 8) Cefotaxime 6) Streptomycin ©) Penicillin G 6) Exyhromycin ©) Co-trimoxazole Answer [MEIRIBWBHBN) othe: User's Explanation Repost An Exor Question Explanation Co-Trimoxazole ot Trmethoprim-sulfamethoxazole is important in the treatment of pneumocystis carinii pneumonia, otitis media in patients with penicillin allergies, and in severe worsening in COPD patients. None of the other agents has significant activity against Pneumocystis carini, ‘Mark this question & => Question Td : 72643 Question 6 of 30 A 29-year-old HIV-positive patient presents with a pneumonia suspected to be caused by Pneumocystis carinil, Which one of the following antitricrobial agents would be most applicable in the treatment ofthis disease? 8) Cefotexime +) Streptomycin 6) Penicilin G 6) Exythromycin SV © 8) Co-trimoxazole Answer [BEWBHBM) othe: User's Explanation Repost An Exor Question Explanation Co-Trimoxazole ot Trmethoprim-sulfamethoxazole is important in the treatment of pneumocystis carinii pneumonia, otitis media in patients with penicillin allergies, and in severe worsening in COPD patients. None of the other agents has significant activity against Pneumocystis carini, Mark this question & => Question Id : 75952 Question 7 of 30 ‘All of the following are true of infection with Coccidioides Iinmitis EXCEPT a) Infection results from inhalation of Arthroconidia ) T-cell inmmunity arrests fisngal proliferation. ©) Causes meningitis in inmmunocompromised aduts. 4) Following the initial puimonery infection, the organism spreads hematogenously ©) Both IM and IgG antibodies, induced by infection, confer protection. Question Explanation: Coccidicidemycosis almost always results from inhalation of the arthroconidia of Coctidioides Inmitis, Several weeks alter the onset ofinfection, T-cell immunty normally arrests the fungal proliferation, In a small percentage of cases, following the intial pulmonary infection, the organism spreads hematogenously. Both IgM and IgG antbodies are induced by infection, but neither is protective High titers of IgG herald a poor prognosis. Reactivation of dormant lesions may result in persons whose cell-mediated ireraunity becomes impaired at a later time. n Report An Error Mark this question & => Question Id : 75952 Question 7 of 30 ‘All of the following are true of infection with Coccidioides Iinmitis EXCEPT a) Infection results from inhalation of Arthroconidia ) T-cell inmmunity arrests fisngal proliferation. ©) Causes meningitis in inmmunocompromised aduts. 4) Following the initial puimonery infection, the organism spreads hematogenously Y © &) Both IgM and IgG antibodies, induced by infection, confer protection Question Explanation: Coccidicidemycosis almost always results from inhalation of the arthroconidia of Coctidioides Inmitis, Several weeks alter the onset ofinfection, T-cell immunty normally arrests the fungal proliferation, In a small percentage of cases, following the intial pulmonary infection, the organism spreads hematogenously. Both IgM and IgG antbodies are induced by infection, but neither is protective High titers of IgG herald a poor prognosis. Reactivation of dormant lesions may result in persons whose cell-mediated ireraunity becomes impaired at a later time. n Report An Error ‘Marke this question <= => Question Td : 78499 Question 8 of 30 After noticing the appearance of a blister on a patient, the physician peeforms a Tzanke's smear, which is most usefil in the diagnosis of a) HSV infection. ») CMV infection ©) RSV infection 4) EBV infection. @) Mycobacterum Tuberculosis Infection Question Explanation: The Tzanck’s smearis a cytological test that utlzes scrapings of a blister’s base and roof Tris particularly helpfil for the diagnosis of herpes infections. ‘Marke this question <= => Question Td : 78499 Question 8 of 30 After noticing the appearance of a blister on a patient, the physician performs a Tzank's smear, which is most usefil in the diagnosis of ¥ © a) HSV infection, b) CMV infection. c) RSV infection. d) EBV infection. e) Mycobacterum Tuberculosis Infection. Question Explanation: The Tzanck’s smearis a cytological test that utlzes scrapings of a blister’s base and roof Tris particularly helpfil for the diagnosis of herpes infections. ‘Mark this question & => Question 9 of 30 ‘Which ofthe following statements regarding Kaposi's sarcoma is NOT wus? 8) Lymph nodes are often involved. ) his often alate manifestation of ELV infection 6) Tt can cause pleural effusions 6) Lesions can occur in regions of previous trauma. ©) Kaposi's sarcoma is more common in males with HIV/AIDS, newer (RESIN) otter User's Explanation Report An Error Question Explanation: Kaposi's sarcoma is often an early manifestation of HIV infection, All ofthe other statements ace true Question Td : 80422 ‘Mark this question & => Question 9 of 30 “Which of the following statements regarding Kaposi's sarcoma is NOT tue? a) Lymph nodes are often involved. Y © b) Bis often a late manifestation of HIV infection. ¢) Itcan cause pleural effusions. 6) Lesions can occur in regions of previous trauma e) Kaposi's sarcoma is more common in males with HIV/AIDS. newer (RESIN) otter User's Explanation Report An Error Question Explanation: Kaposi's sarcoma is often an early manifestation of HIV infection, All ofthe other statements ace true Question Td : 80422 ‘Mark this question —& => Question Id : 81698 Question 10 of 30 ‘Which ofthe following maternal condition during pregnancy is NOT associated with major congenital anomalies? 2) Rubella +) Toxoplasmosis ©) Viral gastroenteritis, 4) Cytomegalovirus. ©) Syphilis. newer [UEIRERIBNY Other Users Explanation Report An Exes Question Explanation: ‘The other four diseases represent different members of the TORCHS group of'infections that can cause severe congerital malformations when they occur in women during pregnancy. ‘Mark this question —& => Question Id : 81698 Question 10 of 30 ‘Which ofthe following maternal condition during pregnancy is NOT associated with major congenital anomalies? «) Rubel +) Toxoplasmosis Y © ©) Viral gastroenteritis, 4) Cytomegalovirus. «) Syphiis newer [UEIRERIBNY Other Users Explanation Report An Exes Question Explanation: ‘The other four diseases represent different members of the TORCHS group of'infections that can cause severe congerital malformations when they occur in women during pregnancy. ‘Mark this question = Question Id : 87675 Question 11 of 30 An 8 year old gil suffers from a viral infection, characterized by crops of wesicular lesions that begin on the trunk. Five days after the onset of the rash, she begins vomiting and becomes lethargic. She is brought to the emergency room and admitted to the hospital ater which she becomes comatose. Lab investigations show an elevated liver enzymes and anmonia. The most lixely diagnosis is a) Crigler-Majjar syndrome 6) Dubin-Ichnson syndrome ©) Gilbert syndrome 6) Reye syndrome ©) Rotor syndrome Anewor (REISE) othe: Users Explanation — Report An Evor Question Explanatis The vial disease is chickenpox (varicella). Reye syndrome (fatty liver with encephalopathy) is an acute (and potentially fatal) postviral injury that is characterized by severe mitochondrial damage affecting the liver, brain, skeletal muscle, heast, and kidneys. Most patients are children, although adult cases have been described, Varicella and influenza A and B are the most common precipitating ilnesses. Aspirin use has been linked to the development of this dsorder, but cases occur in the absence of salicylate ingestion Crigler-Najjar syndrome is a rare, mild to severe form of inherited unconjugated hyperbiliubinemia Dubin-Tohnson syncrome is an inherited conjugated hyperbilirbinemia associated with a darkly pigmented liver. Gilbert syndrome is 2 relatively common, benign form of inherited unconjugated hyperblirubinemia, Rotor syadrome resembles Dubin-Johnson syndrome but is associated with a normal colored liver. ‘Mark this question = Question Id : 87675 Question 11 of 30 An 8 year old gil suffers from a viral infection, characterized by crops of wesicular lesions that begin on the trunk. Five days after the onset of the rash, she begins vomiting and becomes lethargic. She is brought to the emergency room and admitted to the hospital ater which she becomes comatose. Lab investigations show an elevated liver enzymes and anmonia. The most lixely diagnosis is a) Crigler-Majjar syndrome 6) Dubin-Ichnson syndrome ©) Gilbert syndrome Y © 6 Reye syndrome ©) Rotor syndrome Anewor (REISE) othe: Users Explanation — Report An Evor Question Explanatis The vial disease is chickenpox (varicella). Reye syndrome (fatty liver with encephalopathy) is an acute (and potentially fatal) postviral injury that is characterized by severe mitochondrial damage affecting the liver, brain, skeletal muscle, heast, and kidneys. Most patients are children, although adult cases have been described, Varicella and influenza A and B are the most common precipitating ilnesses. Aspirin use has been linked to the development of this dsorder, but cases occur in the absence of salicylate ingestion Crigler-Najjar syndrome is a rare, mild to severe form of inherited unconjugated hyperbiliubinemia Dubin-Tohnson syncrome is an inherited conjugated hyperbilirbinemia associated with a darkly pigmented liver. Gilbert syndrome is 2 relatively common, benign form of inherited unconjugated hyperblirubinemia, Rotor syadrome resembles Dubin-Johnson syndrome but is associated with a normal colored liver. ‘Marle this question <-> Question Td : 89862 Question 12 of 30 Infection with Chlamydia trachomatis causes which of the following? a) lymphogranuloma venereum, ) Chancroid. ©) Syphilis 6) Bacterial vaginosis, ©) Granvioma inguinale Anewor [UEXIRISVER) otnerueorsExplanation Report An Error Question Explanation: Infection with Colymmatobacterium granulometis causes painful beefy red grarmlomatous lesions, Ttis a sexually transmitted disease. 1H. ducreyi causes chancroid, which presents with a painfil ulcer and secondary inguinal edenopathy. Treponema pallidum causes syphilis. Gardenerela spp. is a factor in bacterial vaginosis. Chlamydia trachomats subtypes L1-L3 may cause lymphogranuloma venereum, which may produce a vesicular lesion followed by inguinal lymphadenitis and, eventually, bubs inguinale ‘Marle this question <-> Question Td : 89862 Question 12 of 30 Infection with Chlamydia trachomatis causes which of the following? ¥ © a) lmphogranuloma venereum. ) Chancroid. ©) Syphilis 6) Bacterial vaginosis, ©) Granvioma inguinale Anewor [UEXIRISVER) otnerueorsExplanation Report An Error Question Explanation: Infection with Colymmatobacterium granulometis causes painful beefy red grarmlomatous lesions, Ttis a sexually transmitted disease. 1H. ducreyi causes chancroid, which presents with a painfil ulcer and secondary inguinal edenopathy. Treponema pallidum causes syphilis. Gardenerela spp. is a factor in bacterial vaginosis. Chlamydia trachomats subtypes L1-L3 may cause lymphogranuloma venereum, which may produce a vesicular lesion followed by inguinal lymphadenitis and, eventually, bubs inguinale Marke this question e& => Question Ta : 91324 Question 13 of 30 ‘Treating HIV infected cells with an inhibitor ofthe FTV protease will cause which one of the fallowring? a) An inabiity of the virus to perform reverse transcription. ') Aninability of the viral genome to integrate into cellular DA, ¢) Formation of an aberrant tat protein Synthesis of only multiply spliced transcripts e) Synthesis of non-infectious viral progeny. Answer (Explanation | Other User's Explanation Report An Error Question Explanation: ‘The HIV protease functions mainly as a maturation protein, so non-infectious progeny vimus is produced in the presence of protease inhibitors, The virus is assembled with precursor gag and env proteins, but these must be cleaved by the viral protease during or after egress from the cell to complete the maturation process and render the virus infectious, The reverse transcriptase and integrase proteins are brought into the infected cell in the virus particle and are not directly affected by protease inhibition, The viral tat protein is made in infected cells and is not acted on by the protease ‘The wiral rev protein is responsible for suppressing synthesis of multiple spliced transcripts, and itis not influenced by the protease Marke this question e& => Question Ta : 91324 Question 13 of 30 ‘Treating HIV infected cells with an inhibitor ofthe FTV protease will cause which one of the fallowring? a) An inabiity of the virus to perform reverse transcription. ') Aninability of the viral genome to integrate into cellular DA, ¢) Formation of an aberrant tat protein Synthesis of only multiply spliced transcripts ¥ © ¢) Synthesis of non-infectious viral progeny, Answer (Explanation | Other User's Explanation Report An Error Question Explanation: ‘The HIV protease functions mainly as a maturation protein, so non-infectious progeny vimus is produced in the presence of protease inhibitors, The virus is assembled with precursor gag and env proteins, but these must be cleaved by the viral protease during or after egress from the cell to complete the maturation process and render the virus infectious, The reverse transcriptase and integrase proteins are brought into the infected cell in the virus particle and are not directly affected by protease inhibition, The viral tat protein is made in infected cells and is not acted on by the protease ‘The wiral rev protein is responsible for suppressing synthesis of multiple spliced transcripts, and itis not influenced by the protease ‘Maze this question & => Question Ti : 96185 Question 14 of 30 A235 year old African man from the Nile valley presents with dysaria, hematatia, anc proteinuria, Urine samples revecled parasite eggs. His physician suspects schistoscmiasis. The type of schistosomiases that can cause these eymptoms is a) Schistosomiasis mansoni +) Schistesomiasis japonica c) Schistosomiasis haematobia 4) Schistosomiasis intercalatum ¢) Schistosomiasis mekongi Answer (Bavienenon) Other User's Explanation Report An Error Question Explanation: S haematobium is endemic in Africa and the eges form gramulomas in the urinary bladder, leading to obstructive uropathy, bacteriuria, hematuria, and even bladder cancer. Schistosomiasis mansoni is endemic in several Claribbean countries and southwest ‘Asia. Symptoms include fever, abdominal pain, enorezia, and headache, which were not present in this patent. Hepatospleaomegaly and eosinophilia, also lacking in this patient, are common, Schistosomiasis japonica is endemic in Asia. Syraptoms inchide fever, weakness, abdominal pain, darthea, and hepatosplenomegaly, which were absent in this patient. Schistosomiasis intercalatum lay their eggs in the mesenteric veins and thus produce gastrointestinal symptoms, which were absent in this patient. Schistosomiasis mekongi also lay their eggs in the mesenieric veins and causc gastrointestinal symptoms once the cercariae have penetratzd the skin. ‘Maze this question & => Question Ti : 96185 Question 14 of 30 A235 year old African man from the Nile valley presents with dysaria, hematatia, anc proteinuria, Urine samples revecled parasite eggs. His physician suspects schistoscmiasis. The type of schistosomiases that can cause these eymptoms is a) Schistosomiasis mansoni +) Schistesomiasis japonica Y © 0) Schistosomiasis heematobia 4) Schistosomiasis intercalatum ¢) Schistosomiasis mekongi Answer (Bavienenon) Other User's Explanation Report An Error Question Explanation: S haematobium is endemic in Africa and the eges form gramulomas in the urinary bladder, leading to obstructive uropathy, bacteriuria, hematuria, and even bladder cancer. Schistosomiasis mansoni is endemic in several Claribbean countries and southwest ‘Asia. Symptoms include fever, abdominal pain, enorezia, and headache, which were not present in this patent. Hepatospleaomegaly and eosinophilia, also lacking in this patient, are common, Schistosomiasis japonica is endemic in Asia. Syraptoms inchide fever, weakness, abdominal pain, darthea, and hepatosplenomegaly, which were absent in this patient. Schistosomiasis intercalatum lay their eggs in the mesenteric veins and thus produce gastrointestinal symptoms, which were absent in this patient. Schistosomiasis mekongi also lay their eggs in the mesenieric veins and causc gastrointestinal symptoms once the cercariae have penetratzd the skin. ‘Marke this question & => Question Td : 105060 Question 15 of 30 Spread of methicilin resistant staphslocaccal infection in mursing home residents is best prevented by which one of the following? 2) Prophylactic antibictic treatment of all residents +) Ieolation of an infected resident. c) Hand washing by an infected resident 4) Mot sharing utenzile among residents 6) Transfer of an infected resident to an acute care facility Question Explanation: Tsolation of an infected person until he or she has been adequately treated for MRSA is the best way of preventing the infection of thers. None of the other choices by itself would be beneficial, Cption A might resuit in an increased incidence of MRSA in the population, Option 3 is medically unnecessary. Report An Error ‘Marke this question & => Question Td : 105060 Question 15 of 30 Spread of methicilin resistant staphslocaccal infection in mursing home residents is best prevented by which one of the following? 2) Prophylactic antibictic treatment of all residents Y © 6) Isolation of an infected resident. c) Hand washing by an infected resident 4) Mot sharing utenzile among residents 6) Transfer of an infected resident to an acute care facility Question Explanation: Tsolation of an infected person until he or she has been adequately treated for MRSA is the best way of preventing the infection of thers. None of the other choices by itself would be beneficial, Cption A might resuit in an increased incidence of MRSA in the population, Option 3 is medically unnecessary. Report An Error Mark this question <=> Question Id : 118340 Question 16 of 30 ‘True statement regarding the use of antibiotics in the neutropenic patient who is febrile? 8) Antibiotics should be avoided unless a source of infection is isolated. b) Antibiotics should be withheld until an organism is isolated. ©) These patients frequently get low grads fevers and rarely require antibiotics, 6) The patient should be empirically started on cefazolin as Staph. aureus coverage is most important 8) The patient should be empirically started on two drugs with activity against Psuedomonas Question Explanation Neutropenic patients are at high risk for Hie threatening bacterial infections and should be started on antibiotics empirically whenever they are febrile. These patients do fairly well with Gram postive infections, but the outcome with Gram negative infections is poor and empiric therapy with two antibiotics with activiy against Pouedomonas is essential. The physician chould not wait for culture resube or the identiication of a source of infection, as an established infection may be life threatening in these patients. Report An Error Mark this question <=> Question Id : 118340 Question 16 of 30 ‘True statement regarding the use of antibiotics in the neutropenic patient who is febrile? a) Antbiotics should be avoided unless a source of infection is isolated. ‘b) Antibiotics should be withheld until an organism is isolated. c) These patients frequertly get low grade fevers and rarely require antibiotics The patient should be empirically started on cefazolin as Staph. aureus coverage is most important. Y © 2&) The patient should be errpitically started on two drugs with activity against Psuedotonas Question Explanation Neutropenic patients are at high risk for Hie threatening bacterial infections and should be started on antibiotics empirically whenever they are febrile. These patients do fairly well with Gram postive infections, but the outcome with Gram negative infections is poor and empiric therapy with two antibiotics with activiy against Pouedomonas is essential. The physician chould not wait for culture resube or the identiication of a source of infection, as an established infection may be life threatening in these patients. Report An Error ‘Marke this question <= => Question Td : 132204 Question 17 of 30 All oF the following are common symptoms in patients with acute HIV infection, EXCEPT a) Fever +) Myalga c) Thnish 4) Lymphadenopathy ©) Photophobia Answer (Bortatation Other User's Explanation Report An Error Question Explanatiot ‘Thrush is often seen during the middle stages of AIDS development in HIV infected patients, However, such symptoms as fever, myalgia, fever, lymphadenopatay, photophobia, and maculopapular rashes are common in the easly stages of acute infecton by EIV (e., two to six weeks post infection) ‘Marke this question <= => Question Td : 132204 Question 17 of 30 All oF the following are common symptoms in patients with acute HIV infection, EXCEPT a) Fever b) Myalga Y © c) Thrush d Lymphadenopathy €) Photophobia Answer (Bortatation Other User's Explanation Report An Error Question Explanatiot ‘Thrush is often seen during the middle stages of AIDS development in HIV infected patients, However, such symptoms as fever, myalgia, fever, lymphadenopatay, photophobia, and maculopapular rashes are common in the easly stages of acute infecton by EIV (e., two to six weeks post infection) ‘Mark this question & => Question Td : 141356 Question 18 of 30 An HIY positive 24 year old man has headache, fever, voting, and general feeling of malaise. The patient has a brief seizure during examination. CSE analysis is as follows: 150 cells, mostly lymphocytes, protein 60 mg/dL, and glucose 35 mg/dL. The mecharisin of action of the likely therapeutic agent is a) Binding to the sterols in the cell membrane b) Inhibition of B (1, 3) D ghacan syathesis ©) Inhibition of steroidogenesis 4) Inhibition of squelence epoxidase ¢) Inhibition of the synthesis of exgosterol Anowor [RESRIRISHGN) othe: User's Explanation Repost An vor Question Explanation: Cryptococcus neoformans is a common cause of meningitis in the imminocampromised patients. The CSF findings in this case are consistent with a fungal infection in the ceatral nervous system, Amphotericin B is a polyene antibiotic produced by a sirain of Streptomyces nodosus that is fungistatic or fungicidal, depending on its concentration in the body fluids. The drug acts by binding to sterols in the fingal cel membrane, thereby increasing cell membrane permeability, and leakage of intracellular components to the periphery, causing subsequent cell death, Amphotericin B is indicated for the treatment of potentially life threatening invasive fingal irfections. Ttis important to note that nephrotozicity is the limiting factor of amphctericin B usage. Caspofinginis an antifingal that inhibits the synthesis of B (1, 3) D glacan in fungal cell walls, Tkis indicated for the treatment ofinvasive Aspergillus infections in patients unresponsive to other therapies, including ketoconazole and amphotericin B. Ketoconazole is an azole antifungal thet exerts its mechanism of action through the mhibition of steroidogenesis. Ketoconazole is also effective in decreasing cortisol levels in most Cushing disease patients. Ttis used for the treatment of serious systemic mucocutaneous fungal infections. Terbinafine is an antifungal that exerts its mechanism of action by inhibiting squalene epoxiiiace thereby inhibiting the synthesic of ergosterol. Terbinafine is, indicated for the weatment of onychomycosis. ‘Mark this question & => Question Td : 141356 Question 18 of 30 An HIY positive 24 year old man has headache, fever, voting, and general feeling of malaise. The patient has a brief seizure during examination. CSE analysis is as follows: 150 cells, mostly lymphocytes, protein 60 mg/dL, and glucose 35 mg/dL. The mecharisin of action of the likely therapeutic agent is YW © a) Binding to the sterols in the cell membrane b) Inhibition of B (1, 3) D ghacan syathesis ©) Inhibition of steroidogenesis 4) Inhibition of squelence epoxidase ¢) Inhibition of the synthesis of exgosterol Anowor [RESRIRISHGN) othe: User's Explanation Repost An vor Question Explanation: Cryptococcus neoformans is a common cause of meningitis in the imminocampromised patients. The CSF findings in this case are consistent with a fungal infection in the ceatral nervous system, Amphotericin B is a polyene antibiotic produced by a sirain of Streptomyces nodosus that is fungistatic or fungicidal, depending on its concentration in the body fluids. The drug acts by binding to sterols in the fingal cel membrane, thereby increasing cell membrane permeability, and leakage of intracellular components to the periphery, causing subsequent cell death, Amphotericin B is indicated for the treatment of potentially life threatening invasive fingal irfections. Ttis important to note that nephrotozicity is the limiting factor of amphctericin B usage. Caspofinginis an antifingal that inhibits the synthesis of B (1, 3) D glacan in fungal cell walls, Tkis indicated for the treatment ofinvasive Aspergillus infections in patients unresponsive to other therapies, including ketoconazole and amphotericin B. Ketoconazole is an azole antifungal thet exerts its mechanism of action through the mhibition of steroidogenesis. Ketoconazole is also effective in decreasing cortisol levels in most Cushing disease patients. Ttis used for the treatment of serious systemic mucocutaneous fungal infections. Terbinafine is an antifungal that exerts its mechanism of action by inhibiting squalene epoxiiiace thereby inhibiting the synthesic of ergosterol. Terbinafine is, indicated for the weatment of onychomycosis. € Ss pile wl AleX pogle xy - Jad SX 3/1/2014 6:43:37 PM. 7% wow interface ec ki % php dutid=14759 oo wsdl dine Ofte ge sgsoll JS) Free Hatmall CL www. interface.edu.pk/med apps Eljcoogle Asetings [)Sgnin Zou wos dul xarns ftest-analy' * 8 Gi other bookmar ‘Marte this question <= => Question Td : 141680 Question 19 of 30 A.15 year old boy atending summer camp is admitted to the ER after suffering a seizure. After completing a high diving class in the camp lake he developed a fever 10 days later and began to complain of a severe prefrontal headache and an altered sense of smell with a profise green discharge from his nose. CSF analysis shows 100 neutrophils, 50 lymphocytes, 30 mg/dl. atacose, and 60 mg/dL protein. The child dies on the second hospital day in spite the appropriste therapy Postmortem specimen from CMS would show a) Gram-negative, encapsulated diplococci b) Gram-positive, catalase negative diplococei ©) Motle Alagellated trophozoites 4) Multinucleated giant cells and intranuclear inclusion bodies ¢) Ribbon like hyphae branching at 90° angles Question Explanation: This child has acqnired primary amebic meningoencephalitis cansed by Naealeria fowleri, This organism is a fiee living ameba which is found in warm, fresh water lelees. During the act of diving into such lakes, changes in pressure can force the organism through the cribriform plate and result in the production of necrotic lesions spreading from the olfactory lobes, The form of the organist that is found in human tissue is a flagellated trophozoite. Gram negative, encapsulated diplococci describes Neisseria meningtidis. This is a common cause of meningitis in young aduits, but the case history would have to describe the presence of petechial hemorrhages from endotoxin toxicity. N. meningitidis orerproduces outer- membrane fragments and thus produces very catly signs of endotoxin shock compared.to other gram negative organisins. Gram positive, catalase negative diplococei describes Streptococcus pneumoniae, This is the most common cause of typical pneumonia. Multinucleated giant cells anc intranuclear inclusion bockes describes cells infected with hempes simplex virns, This is the most common canse of viral encephalitis. Ttis not associated with swimming or diving Although the CSF cell, protsin, and glucose values could have indicated a viral etiology, herpes simplex encephalitis is also associated with a high opening pressure end the presence of many red blood cells. Encephalitis is also generally associated with confusion or change in mental status. Ribbon-lke hyphae branching at 90° angles describes the Zygomycoohyta bread meld fimal in the genera Rhizopys, Absidia, and Mucor. These agents can cause fatal rhinocerebral infections in ketoacidotic clabetics and adults with cancer. The symptoms would be quite similar to those described in this case history, but these are oppostunisiic pathogens, which do not cause disease in otherwise healthy individuals Total Questions BREKERRBEBY eRe eRe SBBERBREBBEB XxXxXXKXKXKXKXKXKXKXKXKKXKKXKKKKKKKKKKKKKK € Ss pile wl AleX pogle xy - Jad SX 3/1/2014 6:43:37 PM. 7% wow interface ec ki % php dutid=14759 oo wsdl dine Ofte ge sgsoll JS) Free Hatmall CL www. interface.edu.pk/med apps Eljcoogle Asetings [)Sgnin Zou wos dul xarns ftest-analy' * 8 Gi other bookmar ‘Marte this question <= => Question Td : 141680 Question 19 of 30 A 15 year old boy attending summer camp is admitted to the ER after suffering a seize. Atter completing a high civing class in the camp lake he developed a fever 10 days later and began to complain of a severe prefiontal headache and an altered sense of smell with a profise green discharge from his nose. CSF analysis shows 100 neutrophils, 50 lymphocytes, 30 mg/dl. atacose, and 60 mg/dL. protein. Tho child dies on the second hospital day in spite the approgriete therapy. Postmortsm specimen from CNS would show 8) Gram-negative, encapsulated diplacocsi +b) Gram-positive, catalase negative diplococci JY © ©) Motle flagellated trophozoites 4) Multinucleated giant cells and intranuclear inclusion bodies ¢) Ribbon like hyphae branching at 90° angles Question Explanation: This child has acqnired primary amebic meningoencephalitis cansed by Naealeria fowleri, This organism is a fiee living ameba which is found in warm, fresh water lelees. During the act of diving into such lakes, changes in pressure can force the organism through the cribriform plate and result in the production of necrotic lesions spreading from the olfactory lobes, The form of the organist that is found in human tissue is a flagellated trophozoite. Gram negative, encapsulated diplococci describes Neisseria meningtidis. This is a common cause of meningitis in young aduits, but the case history would have to describe the presence of petechial hemorrhages from endotoxin toxicity. N. meningitidis orerproduces outer- membrane fragments and thus produces very catly signs of endotoxin shock compared.to other gram negative organisins. Gram positive, catalase negative diplococei describes Streptococcus pneumoniae, This is the most common cause of typical pneumonia. Multinucleated giant cells anc intranuclear inclusion bockes describes cells infected with hempes simplex virns, This is the most common canse of viral encephalitis. Ttis not associated with swimming or diving Although the CSF cell, protsin, and glucose values could have indicated a viral etiology, herpes simplex encephalitis is also associated with a high opening pressure end the presence of many red blood cells. Encephalitis is also generally associated with confusion or change in mental status. Ribbon-lke hyphae branching at 90° angles describes the Zygomycoohyta bread meld fimal in the genera Rhizopys, Absidia, and Mucor. These agents can cause fatal rhinocerebral infections in ketoacidotic clabetics and adults with cancer. The symptoms would be quite similar to those described in this case history, but these are oppostunisiic pathogens, which do not cause disease in otherwise healthy individuals Total Questions BREKERRBEBY eRe eRe SBBERBREBBEB XxXxXXKXKXKXKXKXKXKXKXKKXKKXKKKKKKKKKKKKKK ‘Mark this question = => Question Td : 142014 Question 20 of 30 A34 year old female recently retumed from several years in the Peace Corps in Guatemala. She presents with severe, acute, sight upper quadrant abdominal pain Before coming back. she had several months of bloody diarrhea. Liver CT scan shows lesions that ate interpreted to be abscesses. The mostllieely cause is a) Ascaris lumbricoides b) Entamoeba histolytica ¢) Enterobius &) Salmonella typhi ©) Shigella species Question Explanation: ‘The patient probably has hepatic amebiasis, which is a ife threatening complication of intestinal infection with Entamoeba histolytica E histolytica is transmitted via the fecal oral route. Intestinal colonization (which may be asymptomatic) always precedes Infection of the liver. The abscesses generally contain necrotic debris, with amoebae located along the edges ofthe abscess. Treatment for liver abscesses of this origins iodoquinol, while metroridazcle is usefal to destroy the luminal stages of the parasite. Ascaris humbricoides can cause intestinal obstruction, but does not usually cause hepatic abscesses. Enterobius vermicularis is the pinworm, which inhabits the rectura, Bloody diathea can also be seen with some strains of Salmonella and Shigella, but these organisins do aot usually cause hepatic abscesses. Bacterial causes of hepatic abscesses include E. cali, Klebsiella, Streptoceccus, Staphylococcus, Bacteroides, aad Pseudomonas ‘Mark this question = => Question Td : 142014 Question 20 of 30 A34 year old female recently retumed from several years in the Peace Corps in Guatemala. She presents with severe, acute, sight upper quadrant abdominal pain Before coming back. she had several months of bloody diarrhea. Liver CT scan shows lesions that are interpreted to be abscesses, The mostlikely cause ie a) Ascaris lumbricoides Y © b) Entamoeba histolytica ¢) Enterobius 4) Salmonella typhi ©) Shigella species Question Explanation: ‘The patient probably has hepatic amebiasis, which is a ife threatening complication of intestinal infection with Entamoeba histolytica E histolytica is transmitted via the fecal oral route. Intestinal colonization (which may be asymptomatic) always precedes Infection of the liver. The abscesses generally contain necrotic debris, with amoebae located along the edges ofthe abscess. Treatment for liver abscesses of this origins iodoquinol, while metroridazcle is usefal to destroy the luminal stages of the parasite. Ascaris humbricoides can cause intestinal obstruction, but does not usually cause hepatic abscesses. Enterobius vermicularis is the pinworm, which inhabits the rectura, Bloody diathea can also be seen with some strains of Salmonella and Shigella, but these organisins do aot usually cause hepatic abscesses. Bacterial causes of hepatic abscesses include E. cali, Klebsiella, Streptoceccus, Staphylococcus, Bacteroides, aad Pseudomonas ‘Marle this question <=> Question Td : 145375 Question 21 of 30 A 36 year old sailor prevents because of painful, fuctant masses in bic groin, Examination shows multiple erlarged, abscessed lymph. nodes draining through the skin via indolent sinuses. He previously had 2 small papular lesion on his penis that spontaneously resolved, ‘The most likely diagnosis is 2) Condylome acuminatum ') Granuloma inguinale ©) Herpes virus infection ©) Lymphogranloma venereum 8) Syphilis Question Explanation: This is lyinphogranvioma venereum The etlarged, abscessed lymph nodes ate termed "buboes" (which can also occur in bubonic plague). The only other commonly discussed sexually transmitted disease in which they can occur is chancroid, Histologically. the tbuboes of lymphogranuloma venereum are enlarged lymph nodes with stellate abscesses. The primary lesion is usually a self-healing papule or shallow ulcer. The causative organism is Chlamydia trachomatis Condsloma acuminatum causes a papillary lesion and does not cause buboes. Gramuloma inguinale causes a spreading ulcer and does not canse buboes. Herpes virus infection causes tiny vesicles and ulcers and does not cause buboes. Syphilis causes a painless nodule and does not cause buboes ‘Marle this question <=> Question Td : 145375 Question 21 of 30 A 36 year old sailor prevents because of painful, fuctant masses in bic groin, Examination shows multiple erlarged, abscessed lymph. nodes draining through the skin via indolent sinuses. He previously had 2 small papular lesion on his penis that spontaneously resolved, ‘The most likely diagnosis is 2) Condylome acuminatum ') Granuloma inguinale ©) Herpes virus infection Y © & Lymphogrannioma venereum 8) Syphilis Question Explanation: This is lyinphogranvioma venereum The etlarged, abscessed lymph nodes ate termed "buboes" (which can also occur in bubonic plague). The only other commonly discussed sexually transmitted disease in which they can occur is chancroid, Histologically. the tbuboes of lymphogranuloma venereum are enlarged lymph nodes with stellate abscesses. The primary lesion is usually a self-healing papule or shallow ulcer. The causative organism is Chlamydia trachomatis Condsloma acuminatum causes a papillary lesion and does not cause buboes. Gramuloma inguinale causes a spreading ulcer and does not canse buboes. Herpes virus infection causes tiny vesicles and ulcers and does not cause buboes. Syphilis causes a painless nodule and does not cause buboes Mark this question ez Question Id : 145469 Question 22 of 30 A 29 year old HIV+ man has pain on swallowing, Examination is remarkable for white plaque like material on his tongue and buccal ‘mucosa, which is scraped and sent to the laboratory. Test thatis most likely to confirm the identity of the infectious agent is which one of the following? 2) CAME test b) Elek test ©) Germ tube test 6) Heterophile antibody test ©) Weil-Felix test Question Explanation: Candida albicans produces oral thrush which is common in acute HIV disease, and besomes increasingly common as the CD4 cell count fall The lesions are usually painless. Diagnosis is by demonstration of pseudohypha formation in animal serum, the basis of the germ tube test. Tae cAMP testis used to identify a partial hemolytic agent produced by Streptococcus agalactiae (group B streptococous). This testis used to distinguish between group A and group B streptococri, along with testing for bacitracin sensitivity Group B. streptococci cause ecnaéal meningiic and septicemia, not the oral lesione deccribed here, The Elekc tect ie ured te identify toxin producing strains of organisms in the oropharynx that resemble Corynebacterium diphiheriae, Ibis used to differentiate between the pathogen C. diphtheriae and aomal fora diphtheroids. C. diphtheriae causes a gray fbrnous exudate to form oa the uvula and upper airways. It would not have an association with HIV infection. Heterophile antibody tests are corarmon in medicine in cases in which a causative agent ic dificult to identify or culture. There teste depend onthe detection of accidental cross- reactions between antibodies formed during an infection with an unrelated substance. Key examples ofheterophile antibody tests include the monospot test, the Weil Felix test, end the VDRL for syphilis. None of these tests would be used to identify the cause of'a white oral exudate in AIDS patients. The Weil-Felis testis a heterophile antibody test used in the diagnosis of several tickettsial diseases. It depends on the accidental cross reaction between antibodies produced during rickettsial infections and the OX strain of Proteus vulgaris. Rickettsial diseases are arthropod-bome diseases that are all characterized by rashes. None of them would be consistent with the signs in this case. Report An Error Mark this question ez Question Id : 145469 Question 22 of 30 A 29 year old HIV+ man has pain on swallowing, Examination is remarkable for white plaque like material on his tongue and buccal ‘mucosa, which is scraped and sent to the laboratory. Test thatis most likely to confirm the identity of the infectious agent is which one of the following? 2) CAMP test ) Blek test Y¥ © ©) Germ be test ©) Heterophile antibody test ©) Weil-Felx test Question Explanation: Candida albicans produces oral thrush which is common in acute HIV disease, and besomes increasingly common as the CD4 cell count fall The lesions are usually painless. Diagnosis is by demonstration of pseudohypha formation in animal serum, the basis of the germ tube test. Tae cAMP testis used to identify a partial hemolytic agent produced by Streptococcus agalactiae (group B streptococous). This testis used to distinguish between group A and group B streptococri, along with testing for bacitracin sensitivity Group B. streptococci cause ecnaéal meningiic and septicemia, not the oral lesione deccribed here, The Elekc tect ie ured te identify toxin producing strains of organisms in the oropharynx that resemble Corynebacterium diphiheriae, Ibis used to differentiate between the pathogen C. diphtheriae and aomal fora diphtheroids. C. diphtheriae causes a gray fbrnous exudate to form oa the uvula and upper airways. It would not have an association with HIV infection. Heterophile antibody tests are corarmon in medicine in cases in which a causative agent ic dificult to identify or culture. There teste depend onthe detection of accidental cross- reactions between antibodies formed during an infection with an unrelated substance. Key examples ofheterophile antibody tests include the monospot test, the Weil Felix test, end the VDRL for syphilis. None of these tests would be used to identify the cause of'a white oral exudate in AIDS patients. The Weil-Felis testis a heterophile antibody test used in the diagnosis of several tickettsial diseases. It depends on the accidental cross reaction between antibodies produced during rickettsial infections and the OX strain of Proteus vulgaris. Rickettsial diseases are arthropod-bome diseases that are all characterized by rashes. None of them would be consistent with the signs in this case. Report An Error ‘Mark this question €& => 146861 Question 23 of 30 ‘During investigation of pneumonia outbreak in elderly living in nursing home, cultares of water from bathroom drains and shower nozzles are positive for Legionella pneumophila when grovin on medium enriched for cysteine and iron. What procedure would allow the visualization of the causative organism in sputum saruples? a) Direct fluorescent antibody test b) Enzymne-tnked immunosorbent assay ©) Indirect faorescent antbody test 4) Racioinmmnoessay 8) Westem blot assay Question Explanation: ‘The only test on the list that would alow youto visualize organisms in a sample from the patient is the direct fuorescent antibody test. Itis performed by treating a tissue sample from the patient with Tuorecceinated antbodies againct a caucative agent. Ifthe tissue sample uoresces under a luorescence microscope the organista is present and the result is positive ‘The enzyme linked immunosorbent assay (ELISA) is a widely used test for antibodies in a patient. It depends on linking antigens to a ticrotiter plate well treating them with serum from the patient, and then adding an enzyme-inked antibody conjugate. When the substrate for the enzyme is added, a color change reflects the fact thet antibody to that antigen was present in the patient. The indirect Auorescent antibody testis a test for the presence of antibodies in a patient Ibis performed by taking a laboratory raised sample of a pathogen treating it with the serum from the patient, and then addirg fucrescently-labeled antibodies against human immmmnoglobulins. the anti-human immunoglobufins are bound, and the sample becomes fhiorescent, itreflects the fact thet the patient had specific aatibodies in his sorum, Radioimmmnoassay is a test for antibodies in a patient. Ituses the same basic technique as the ELISA, but instead of using an enzyme-labeled conjugate this test uses a radioactive label Westem blot assay is used to detect antibodies in a patient Tris best known for ts use in diagnosis of HIV infection, Ibis a cumbersome expensive test involving electrophoresis of test antigens followed by treatment with patient serum and visualization of positive reactions using either an enzyme or a radicisotype system, Report An Error ‘Mark this question €& => 146861 Question 23 of 30 During investigation of pneumonia outbreak in elderiy living in musing home, cultures of water from bathroom drains and shower nozzles are positive for Legionella pneumophila when grown on medium enriched for cysteine and iron. What procedure would allow the visualization of the causative organism in sputum samples? V © a) Direct fluorescent antibody test +b) Enzyme-linked irsmunosorbent assay ¢) Indirect fnorescent antbody test Radioimmunoassay ©) Westem blot assay Question Explanation: ‘The only test on the list that would alow youto visualize organisms in a sample from the patient is the direct fuorescent antibody test. Itis performed by treating a tissue sample from the patient with Tuorecceinated antbodies againct a caucative agent. Ifthe tissue sample uoresces under a luorescence microscope the organista is present and the result is positive ‘The enzyme linked immunosorbent assay (ELISA) is a widely used test for antibodies in a patient. It depends on linking antigens to a ticrotiter plate well treating them with serum from the patient, and then adding an enzyme-inked antibody conjugate. When the substrate for the enzyme is added, a color change reflects the fact thet antibody to that antigen was present in the patient. The indirect Auorescent antibody testis a test for the presence of antibodies in a patient Ibis performed by taking a laboratory raised sample of a pathogen treating it with the serum from the patient, and then addirg fucrescently-labeled antibodies against human immmmnoglobulins. the anti-human immunoglobufins are bound, and the sample becomes fhiorescent, itreflects the fact thet the patient had specific aatibodies in his sorum, Radioimmmnoassay is a test for antibodies in a patient. Ituses the same basic technique as the ELISA, but instead of using an enzyme-labeled conjugate this test uses a radioactive label Westem blot assay is used to detect antibodies in a patient Tris best known for ts use in diagnosis of HIV infection, Ibis a cumbersome expensive test involving electrophoresis of test antigens followed by treatment with patient serum and visualization of positive reactions using either an enzyme or a radicisotype system, Report An Error ‘Mark this question & => Question Td : 149416 Question 24 of 30 A 29 year old IV drug abuser presents with a febrile ilhess that has lasted 2 weeks. He also complains of chill, weakness, dyspnea, cough, arthralga, diarrhea, and abdominal pain. On examinetion, a heart muemnur is preseat, and small tender nodules are found on the finger and to pads, along with small hemorhages oa the pame and soles. While awaiting blood culture result, an empiric antibiotic regimen should primariy be directed at which of the folowing organisms? a) Enterococci ) Pseudomonas aeruginosa ¢) Staphylococcus aureus 4) Streptococcus pneumoniae e} Streptococcus pyogenes Answer | Explanation | Other User's Explanation Report An Error Question Explanation: ‘This patient is presenting with signs and symptoms of acute infective endocardtis (IE), which is most commonly caused by Staphylococcus aureus, Although the clinical characteristics of IE can vary. most patients present with a febrile ilness lasting several days to 2 weeks. This illness is offen accompanied by a variety of nonspecific signs and symptoms such as chills, weakness, dyspnea, cough, arthralgia, diarrhea, and abdominal pain. Heart murmurs occur in approximately 90% of all patients, bus may be absent in patients with right sided infections. Other clinical signs include Osler nodules (purplish or erythematous subcutaneous papules or nodules on the pads of the fingers and toes), Jane way lesions (hemorrhagic painless plaques on the palms and soles), petechiae (mall erythematous painless hemorrhagic lesions that may appear anywhere), and splinter hemorshages (thin linear hemorrhages found under the nail beds of fingers and toes). Ths diagnosis of IE is dependent on positive blood cultures and echocardiographic evidence of “valvular vegetation’ and/or valvular injury, with echocardiography the preferred method of dagnosis. A past medical history of intravenous drug abuse contributes to the diagnosis. Enterococci cause a minority of cases of native valve endocarditis, but ace not usually involved in endocarditis associated with intravenous drug abuse Gram-negative organisms such as Pseudomonas acruginosa are rarely the cause of TE. Streptococcus pneumoniae and Streptococcus pyogenes are nol common causes of TE, ‘Mark this question & => Question Td : 149416 Question 24 of 30 A.29 year old IV drug abuser presents with a febrile ilness that has lasted 2 weeks, He also complains of chills, weakness, dyspnea, cough, arthralga, diarrhea, and abdominal pain. On examinetion, a heart muemnur is preseat, and small tender nodules are found on the finger and to pads, along with small hemorrhages oa the palms and soles. While awaiting blood culture result, an empiric antibiotic regimen should primariy be direcied at which of the folowing organisms? a) Enterococci b) Fscudomonas acruginosa ¥ © 2) Staphylococcus aureus oD) Streptococcus pncumoniac e} Streptococcus pyogenes Answer | Explanation | Other User's Explanation Report An Error Question Explanation: ‘This patient is presenting with signs and symptoms of acute infective endocardtis (IE), which is most commonly caused by Staphylococcus aureus, Although the clinical characteristics of IE can vary. most patients present with a febrile ilness lasting several days to 2 weeks. This illness is offen accompanied by a variety of nonspecific signs and symptoms such as chills, weakness, dyspnea, cough, arthralgia, diarrhea, and abdominal pain. Heart murmurs occur in approximately 90% of all patients, bus may be absent in patients with right sided infections. Other clinical signs include Osler nodules (purplish or erythematous subcutaneous papules or nodules on the pads of the fingers and toes), Jane way lesions (hemorrhagic painless plaques on the palms and soles), petechiae (mall erythematous painless hemorrhagic lesions that may appear anywhere), and splinter hemorshages (thin linear hemorrhages found under the nail beds of fingers and toes). Ths diagnosis of IE is dependent on positive blood cultures and echocardiographic evidence of “valvular vegetation’ and/or valvular injury, with echocardiography the preferred method of dagnosis. A past medical history of intravenous drug abuse contributes to the diagnosis. Enterococci cause a minority of cases of native valve endocarditis, but ace not usually involved in endocarditis associated with intravenous drug abuse Gram-negative organisms such as Pseudomonas acruginosa are rarely the cause of TE. Streptococcus pneumoniae and Streptococcus pyogenes are nol common causes of TE, Question Td : 14% Mark this question ¢<>=> Question 25 of 30 ‘Many executives come unexpectedly to the physician over a two day period complaining of 1-2 days with tmubiple episodes of watery diarthea, Some have had blood wisible in their stool, some have had profound ahdominal cramping and fever. All affected individaals ettended the same buffet style dinner party several days eavlicr. Stool cultures at 42°C grow microacrophilic oxidase- postive gram-negative curved rods with polar flagella. The most iely source of this infection is, 2) Custard and potato salad dishes b) "Carrier" food hanclers ©) Milk. and cream. 46) Poorly canned green vegetables ©) Poultry dishes Question Explanation: ‘This case history describes a typical gastroenteritis due te Campylobacter jejuni, the most common bacterial gastroenteritis n the U.S. Contaminated poultry or poultxy products are recponsible fer more than 50 percent of all infectione. Custard and petato salad ie the most likely source ifthe agent had been Staphylococcus aureus but this would have had an onset of 2 hours and a gram-positive coccus would have been the cause, Camers are aot known to occur with Campylobacter, and ate best known with Salmonella typhi Milk and cream might contein Listeria or Mycobacterium bovis and act to neutralize the stomach acid and thereby decrease the infectious doce of Salmonella and Shigella and others, but would not be the cource of the contamination. Poorly canned green ‘vegetables is the usual source of Clostridium botulinum , which, of course, would have a different disease presentation, Question Td : 14% Mark this question ¢<>=> Question 25 of 30 Many executives come unexpectedly to the physician over a two day period complaining of 1-2 days with mubiple episodes of watery diarthea. Some have had blood wisitle in their stool, some have had profound abdominal cramping and fever. All affected individuals attended the same buffet style dinner party several days easier. Stool cultures at 42°C grow microacrophilic oxidase- postive gram-negative curved rods with polar flagella. The most likely source of this infection is 2) Custard and potato salad dishes +) "Cartier" food handlers 6) Mille end cream 4) Poorly canned green vegetables Y © & Poultry dishes Question Explanation: ‘This case history describes a typical gastroenteritis due te Campylobacter jejuni, the most common bacterial gastroenteritis n the U.S. Contaminated poultry or poultxy products are recponsible fer more than 50 percent of all infectione. Custard and petato salad ie the most likely source ifthe agent had been Staphylococcus aureus but this would have had an onset of 2 hours and a gram-positive coccus would have been the cause, Camers are aot known to occur with Campylobacter, and ate best known with Salmonella typhi Milk and cream might contein Listeria or Mycobacterium bovis and act to neutralize the stomach acid and thereby decrease the infectious doce of Salmonella and Shigella and others, but would not be the cource of the contamination. Poorly canned green ‘vegetables is the usual source of Clostridium botulinum , which, of course, would have a different disease presentation, 3/4/2014 6: “Mark this question => Question Id : 149601 Question 26 of 30 43 PM A 33 year old man presents to the emergency room with a severe headache, Examination shows nuchal rigidity. Lumbar puncture reveals cerebrospinal fluid with markedly increased Immphocstes. Other cell populations are not increased. The most licely cause of his symptoms is a) Escherichia col b) Haemophilus ©) Herpesvirus 4) Mycobacterium tuberculosis €) Treponema pallidum Question Explanatio: ‘The clinically suspected diagnosis is meningitis, which is confimed by the abnormal cerebrospinal fuid. The markedly increased gmmphocytes suggest acute Ipmphocytic meningtis which is distinguished from acute pyogenic meningitis (increased neutrophils as well as iymphocytes). Acute Iymphocytic meningtis is usually viral in origin. Among the meny viruses thet have been implicated, mumps, herpes, Epstein-Barr, echovirus, and Coxsackie virus are the most commen. Escherichia coli and Haemophilus influenzae cause acute pyogenic meningitis. Mycobacterium tuberculosis and Treponema pallidum cause chronic meningitis. 3/4/2014 6: “Mark this question => Question Id : 149601 Question 26 of 30 43 PM A. 33 year old man presents to the emesgency room with a severe headache. Examination shows michal sigiity. Lumbar puncture reveals cerebrospinal fluid with markedly increased Immphocstes. Other cell populations are not increased. The most licely cause of hic cyraptorns is a) Escherichia coi 'b) Haemophilus V © 0) Herpesvins 4) Mycobacteriuin tuberculesie ) Treponema pallidum Question Explanatio: ‘The clinically suspected diagnosis is meningitis, which is confimed by the abnormal cerebrospinal fuid. The markedly increased gmmphocytes suggest acute Ipmphocytic meningtis which is distinguished from acute pyogenic meningitis (increased neutrophils as well as iymphocytes). Acute Iymphocytic meningtis is usually viral in origin. Among the meny viruses thet have been implicated, mumps, herpes, Epstein-Barr, echovirus, and Coxsackie virus are the most commen. Escherichia coli and Haemophilus influenzae cause acute pyogenic meningitis. Mycobacterium tuberculosis and Treponema pallidum cause chronic meningitis. ‘Marke this question —& => Question Td : 203 Question 27 of 30 A.17 year old student developed profuse watery Garrhea within 24 hours of returning from India. Initially he cid not have any nausea, vomiting or stomach cramps, but these developed within a day. Diarrhea is like cloudy water. Stool culture revealed Vibrio cholerae The appropriate antibiotic to administer is a) Ceftriaxone b) Doxycycline c) Meropenem 4) Metronidazele e) Pperacillia plus gentamicin Answer | Bavienaton | Other User's Explanation Report An Error Question Explanation: Cholera has a short incubation petiod of 24-48 hours. The illtess begins with the sudden onset of painless, watery diathoea The diarrhea may be accompanied later by abdominal cramps, nausea and voriting, Patients are usually afebrile, The diarrhoea is typically described as having the appearance ofrice waler and a feintly fshy smell, The diathoea may be copious and resul: in hypovolaemic shock unless fuids are administered ‘The primary aim of treatment is to restore fluid balance; antibiotcs have a secondary role, However, antibiotics have been shown to reduce fiuid loss and hasten clearance (of the organism from the gut Appropriate antibiotics include tetracycline, doxycycline, ciproflozacia, erythromycin, and co-trimoxazole, Tetracycline is usually the firstine drug of choice although resistance is emerging in cerlain parts of the world, ‘Marke this question —& => Question Td : 203 Question 27 of 30 A.17 year old student developed profuse watery Garrhea within 24 hours of returning from India. Initially he cid not have any nausea, vomiting ce stomach cramps, but these developed within a day. Diarrhea is like cloudy water. Stool culture revealed Vibrio cholerae The appropriate antibiotic to administer is a) Ceftriaxone Y © b) Doxycycline c) Meropenem 4) Metronidazole e) Pperacillia plus gentamicin Answer | Bavienaton | Other User's Explanation Report An Error Question Explanation: Cholera has a short incubation petiod of 24-48 hours. The illtess begins with the sudden onset of painless, watery diathoea The diarrhea may be accompanied later by abdominal cramps, nausea and voriting, Patients are usually afebrile, The diarrhoea is typically described as having the appearance ofrice waler and a feintly fshy smell, The diathoea may be copious and resul: in hypovolaemic shock unless fuids are administered ‘The primary aim of treatment is to restore fluid balance; antibiotcs have a secondary role, However, antibiotics have been shown to reduce fiuid loss and hasten clearance (of the organism from the gut Appropriate antibiotics include tetracycline, doxycycline, ciproflozacia, erythromycin, and co-trimoxazole, Tetracycline is usually the firstine drug of choice although resistance is emerging in cerlain parts of the world, ‘Marke this question & => Question Ta : 206462 Question 28 of 30 A 54 year old female has a chest infection, X-ray shows consolidation in the right base and urinary Legionella antigen is found to be postive. The most appropriete treatment for this woman is which one of the following? a) Cefotaxime ) Clarithromycin ©) Co-amoxiclay 4d) Minocycine e) Vancomycin Answer | Bolanation Other User's Explanation Report An Error Question Explanation: ‘The most appropsiate treatment for legionellosis is clarithromycin Ciprofloxacin is also a usefil drug in combination with clarithromycin for severe infections or alone in those intolerant of macrolides. ‘Marke this question & => Question Ta : 206462 Question 28 of 30 A 54 year old female has a chest infection, X-ray shows consolidation in the right base and urinary Legionella antigen is found to be postive. The most appropriete treatment for this woman is which one of the following? a) Cefotaxime ¢ © b) Clarithromycin ©) Co-amoxiclay ) Minocycine e) Vancomycin Answer | Bolanation Other User's Explanation Report An Error Question Explanation: ‘The most appropsiate treatment for legionellosis is clarithromycin Ciprofloxacin is also a usefil drug in combination with clarithromycin for severe infections or alone in those intolerant of macrolides. ‘wow. interface.2du.pk/ Sy ileal AleX CL wow. interface.edu.pk/ned apes Blcoogle A settings Cisiynin Bun woke du € xams ftest-analy Free Hatmall #38 Gi other bookmar 06715 ‘Marte this question = => Question Ta: Question 29 of 30 Correct statement regarding human varicella zoster immunoglobin (WZIG) is which one of the following? a) Is invariably protective against severe vaticella 'b) Is recommended for al patients with eczema exposed to chickerp ox. ) Is used to treat severe chicken pox infection 4) Should be given to a 6 week old baby whose mother has developed chickenpox ¢) Should be given to an 18 week pregnant non-inmaune female who has been exposed to a case of chicken pox. Question Explanation: ‘Varicella kas a secondary infection rate in household contacts of 90%. Iris commonest in spring time, and the incubation period is 14-21 days, It shares the herpes virus family properties of latency and reactivation (zoster) Risks to the fetus and neonate relate to the time of infection: + Lece than 20 wesles pregnancy: congenital varicella (Imb hypoplasia, microcephaly, cataracts, growth retardation, skin scarring) High mortality © Second to third trimester: herpes zoster in an otherwise healthy mfant + Minus seven days to plus seven days after delivery. severe and even fatal disease (30% mortality). Although a lve attermated vaccine is available, iti not licensed for use in the UE. Varicella zoster immunoglobulin is prepared from pooled plasma of blood donors with a history of recent chickenpox or herpes zoster, Being an immunoglobulin, ¢ is a protein concentrate, and should be stored between 2 and 8°C. Donors are screened for HIV, hepatitis B and hepatitis C. VZIG prophylaxis is recommended for patients who fall all the folowing criteria 4 clinical condition that increases the risk of severe varicella, (or exemple, immunosuppression, neonates, pregnant women) + No antibodies to varicella zoster © Significant exposure to chickenpox or herpes Severe or fatal varicella can occur despite VZIG prophylaxis. Active immurisction should therefore be used for susceptible immunosuppressed patients at long term risk. Clinical chickenpox occurs in 50% of those who receive VZIG prophylaxis, and 10% more will be affected sub-clinically. Total Questions a. i [os fe be ks Ek ke ke IS kes BEBEEBBE cKKXKRKXKKKKKKAKRKXKKAKRAKRKAKRAKRMAKRAKRRKRXRK ‘wow. interface.2du.pk/ Sy ileal AleX CL wow. interface.edu.pk/ned apes Blcoogle A settings Cisiynin Bun woke du € xams ftest-analy Free Hatmall #38 Gi other bookmar 06715 ‘Marte this question = => Question Ta: Question 29 of 30 Correct statement regarding human varicella zoster immunoglobin (WZIG) is which one of the following? a) Is invariably protective against severe vaticella 'b) Is recommended for al patients with eczema exposed to chickerp ox. ) Is used to treat severe chicken pox infection 4) Should be given to a 6 week old baby whose mother has developed chickenpox Y © e)Should be given to an 18 week pregnant non-inmune female who has been exposed to a case of chicken pox. Question Explanation: ‘Varicella kas a secondary infection rate in household contacts of 90%. Iris commonest in spring time, and the incubation period is 14-21 days, It shares the herpes virus family properties of latency and reactivation (zoster) Risks to the fetus and neonate relate to the time of infection: + Lece than 20 wesles pregnancy: congenital varicella (Imb hypoplasia, microcephaly, cataracts, growth retardation, skin scarring) High mortality © Second to third trimester: herpes zoster in an otherwise healthy mfant + Minus seven days to plus seven days after delivery. severe and even fatal disease (30% mortality). Although a lve attermated vaccine is available, iti not licensed for use in the UE. Varicella zoster immunoglobulin is prepared from pooled plasma of blood donors with a history of recent chickenpox or herpes zoster, Being an immunoglobulin, ¢ is a protein concentrate, and should be stored between 2 and 8°C. Donors are screened for HIV, hepatitis B and hepatitis C. VZIG prophylaxis is recommended for patients who fall all the folowing criteria 4 clinical condition that increases the risk of severe varicella, (or exemple, immunosuppression, neonates, pregnant women) + No antibodies to varicella zoster © Significant exposure to chickenpox or herpes Severe or fatal varicella can occur despite VZIG prophylaxis. Active immurisction should therefore be used for susceptible immunosuppressed patients at long term risk. Clinical chickenpox occurs in 50% of those who receive VZIG prophylaxis, and 10% more will be affected sub-clinically. Total Questions a. i [os fe be ks Ek ke ke IS kes BEBEEBBE cKKXKRKXKKKKKKAKRKXKKAKRAKRKAKRAKRMAKRAKRRKRXRK ‘Mark this question & Question Td : 206849 Question 30 of 30 A 44 year old man type | diabetic has an ulcer on his sight foot On exam he has a small ulcer of approximetely 2 cm diameter on the outer aspect of his sight big toe. His peripheral pulses are all palpabie but he has a peripheral neuropathy to the mid shins. The ulcer has an erythematous margin and is covered by pus. The most lkely infective organism is which one of the following? ) Escherichia ccli +) MESA, ©) Pseudomonas aeruginosa 6) Staphylococcus aureus ©) Streptococcus pyogenes Question Explanation: “Although infection in diabetic foor ulcers is usually mixed, the most likely infective organism in this case is Staph aureus. Methicillin resistant Staphylococcus aureus (MRSA) would be less ikely but might be suspected ifthe patient developed an infection in hospital or kad recurrent admissions. The most appropriate antibiotic therapy is flacloxacillin, plus another broad spectrum agent with advice that the patient should not weight bear and should wear appropriate footwear. ‘Mark this question & Question Td : 206849 Question 30 of 30 A 44 year old man type | diabetic has an ulcer on his sight foot On exam he has a small ulcer of approximetely 2 cm diameter on the outer aspect of his sight big toe. His peripheral pulses are all palpabie but he has a peripheral neuropathy to the mid shins. The ulcer has an erythematous margin and is covered by pus. The most lkely infective organism is which one of the following? ) Escherichia ccli +) MESA, ©) Pseudomonas aeruginosa ¥ © 4) Staphylococcus aureus ©) Streptococcus pyogenes Question Explanation: “Although infection in diabetic foor ulcers is usually mixed, the most likely infective organism in this case is Staph aureus. Methicillin resistant Staphylococcus aureus (MRSA) would be less ikely but might be suspected ifthe patient developed an infection in hospital or kad recurrent admissions. The most appropriate antibiotic therapy is flacloxacillin, plus another broad spectrum agent with advice that the patient should not weight bear and should wear appropriate footwear. Mark this question => Question Td : 23001 Question 1 of 30 A.23 year old paticat, who underwent splenectomy 2 years ago for trauma, presents to the ER with confision, cough and fever which have developed over the past 12 hours. On atvival, his vital signs are Temp: 41°C Respiratory rate 28iminute Blood pressure 90/70 man Hg Pulse 130/minate ‘Which one of the following investigetions would best confirm the diggnosis? 2) Chests: cay ) Blood culture EKG 4) Urine cuiture «) Sputum culnare Question Explanation: Because splenectomy causes an increased risk of overwhelming sepsis due to encapsulated organisms (such as S. pneumoniae and Haemophilus iafinenzas) the patient chould be immized, ifpossible, price to removal oftthe spleen. ‘An increase in blood leukocytes can occur following a splenectomy. Failure to do so later puts the patient at risk of overwhelming post-splenectomy infection (OPSI), a potentially rapidly fatal septicemia. These bacteria often cause a sore throat under normal circumstances but after splenectomy, when infecting bacteria cannot be adequately opsonized, the infection becomes more severe Tis inpertant to do blood culturee in a post-eplenectcmy patient who precents with fever and signe of infection, to isolate the n Report An Error organism: Mark this question => Question Td : 23001 Question 1 of 30 A.23 year old paticat, who underwent splenectomy 2 years ago for trauma, presents to the ER with confision, cough and fever which have developed over the past 12 hours. On atvival, his vital signs are Temp: 41°C Respiratory rate 28iminute Blood pressure 90/70 man Hg Pulse 130/minate ‘Which one of the following investigetions would best confirm the diggnosis? 2) Chests: cay Y © b) Blood culture EKG 4) Urine cuiture «) Sputum culnare Question Explanation: Because splenectomy causes an increased risk of overwhelming sepsis due to encapsulated organisms (such as S. pneumoniae and Haemophilus iafinenzas) the patient chould be immized, ifpossible, price to removal oftthe spleen. ‘An increase in blood leukocytes can occur following a splenectomy. Failure to do so later puts the patient at risk of overwhelming post-splenectomy infection (OPSI), a potentially rapidly fatal septicemia. These bacteria often cause a sore throat under normal circumstances but after splenectomy, when infecting bacteria cannot be adequately opsonized, the infection becomes more severe Tis inpertant to do blood culturee in a post-eplenectcmy patient who precents with fever and signe of infection, to isolate the n Report An Error organism: ‘Marke this question & => Question Ta : 29483 Question 2 of 30 A 23-year-old lady recently retumed from a holiday in Malta was admitted with a 3 day history offever. generaized lymphadenopathy and e macular rash over trunk and legs. Which of the following is the most liltely diagnosis? a) Sarcoidosis b) Infectious Mononucleosis c) Tuberculosis d) Familial Mediterranean fever ) Actinomycosis Question Explanatioy Infectious Mononucleosis occurs most commonly in adolescents and young adults, Clinical features occur after 2 2-5 week incubation period and include fever malaise pharyngitis, and lymphadenopathy Rashes occur more cammenly in patients that have received penicilin or ampicillin Report An Error ‘Marke this question & => Question Ta : 29483 Question 2 of 30 A 23-year-old lady recently retumed from a holiday in Malta was admitted with a 3 day history offever. generalized lymphadenopathy and a macular rach over trunk and lege. Which of the fellowing is the mostliely diagnosis? a) Sarcoidosis WV © b) Infectious Mononucleosis c) Tuberculosis od) Fanilel Mediterranean fever e) Actinomycosis Question Explanatioy Infectious Mononucleosis occurs most commonly in adolescents and young adults, Clinical features occur after 2 2-5 week incubation period and include fever malaise pharyngitis, and lymphadenopathy Rashes occur more cammenly in patients that have received penicilin or ampicillin Report An Error ‘Marke this question & => Question3 of 30 A 22 year old male presents with increased shortness of breath, He has had a cough for the past week He appears dyspneic and has a temperature of 38 3 C (101.0 F). On physical examination he has bibasilar rales and generalized lymphadenopathy. Rectal examination shows multiple perianal contusions and a stnall amount of blood ooring from the anal onfice. Chest x-ray film shows bilateral patchy alveoler infilrates. What is the most appropriate next step? @) Adit him to the hospital and begin administration of pericilin and gertamnicin, intravenously ) Begin administration of erythromycin, orally, and see him again the next day ©) Adit him to the hospital and begin administration of trimethoprim-sulfamethozazole, intravenously 6) Prescribe isoniazid and rifampin, orally ©) Recommend aspirin, fuids and rest at home Question Explanation: The history and physical raises the possibility of HIV infection, There is a strong suggestion of homoseualty, given that the rectal examination demonstrates multiple perianal contusions and blood oozing from the anal orifice. Furthermore, this man has generalized igmphadenopethy; his chief complaint is of pulmonary distress with an x-ray consistent with Pneumocystis carinii pneumonia (PCP). Since the patient has become increasingly short of breath and is febrile, he should be treated with intravenous therapy. Treatment of choice for ECP is Bacirimn (1imethoprin-sulfamethoxazole). Report An Error ‘Marke this question & => Question3 of 30 A 22 year old male presents with increased shortness of breath, He has had a cough for the past week He appears dyspneic and has a temperature of 38 3 C (101.0 F). On physical examination he has bibasilar rales and generalized lymphadenopathy. Rectal examination shows multiple perianal contusions and a stnall amount of blood ooring from the anal onfice. Chest x-ray film shows bilateral patchy alveoler infilrates. What is the most appropriate next step? @) Adit him to the hospital and begin administration of pericilin and gertamnicin, intravenously ) Begin administration of erythromycin, orally, and see him again the next day Y © ©) Admit him to the hospital and begin administration of trimethoprinn-sulfamethoxazole, intravenously 6) Prescribe isoniazid and rifampin, orally ©) Recommend aspirin, fuids and rest at home Question Explanation: The history and physical raises the possibility of HIV infection, There is a strong suggestion of homoseualty, given that the rectal examination demonstrates multiple perianal contusions and blood oozing from the anal orifice. Furthermore, this man has generalized igmphadenopethy; his chief complaint is of pulmonary distress with an x-ray consistent with Pneumocystis carinii pneumonia (PCP). Since the patient has become increasingly short of breath and is febrile, he should be treated with intravenous therapy. Treatment of choice for ECP is Bacirimn (1imethoprin-sulfamethoxazole). Report An Error ‘Marke this question & => Question Td : 51790 Question 4 of 30 “Which one of the Following is associated with Kaposi's sarcoma? a) Herpes simplex virus ) Human Herpervinus 8 c) Human papilomaviras 4) Cytomegaloviras €) Coxsackievirus Answer | Bplanation Other User's Explanation Report An Error Question Explanatior Patients with HIV infection are at increased isk for a number of opporbmistc infections, including heroes simplex, human papilomaviras infection, cytemegalovirue infection, and many others. Human Herpesvirus 8 (HHV-8) infection ic of particular concem because itis associated with Kaposi's sarcoma. The method of transmission is unknown. ‘Marke this question & => Question Td : 51790 Question 4 of 30 “Which one of the following is assccisted with Kaposi's sarcoma? a) Herpes simplex virus Y © b) Human Herpesvirus 8 c) Human papilomaviris 4) Cytomegalovias €) Coxsackievirus Answer | Bplanation Other User's Explanation Report An Error Question Explanatior Patients with HIV infection are at increased isk for a number of opporbmistc infections, including heroes simplex, human papilomaviras infection, cytemegalovirue infection, and many others. Human Herpesvirus 8 (HHV-8) infection ic of particular concem because itis associated with Kaposi's sarcoma. The method of transmission is unknown. ‘Made this question & => Question Td : 55582 Question 5 of 30 4.38 year old man comes to yen with severe pain Flictiance, erythema, and tenderness localized aver the pass of the distal long finger. Whatis the most likely diagnosis? a) A felon (whitlow) b) A paronychia ©) An epenychial abscess 4) Bacterial senosynoviis ¢) Digital cellulitis Answer | Bopanation Other User's Explanation Report An Error Question Explanation: ‘A felon, also called a whitlow, is an infection of the dgital pulp of the temminal phalanx. The anatomic cheracteristics of this region compactruentalize infoction, resulting in exquisite pressure and pain in the pulp spaces. As with all abscesses, incision and drainage is the single most impostant therapy. Paronychial and eponychial infections occur in the dorsal surface of the finger about the nail Bacterial tenosynovitis is an extension of a desp space infecticn along the tendons. Digital cellulites ic « nonspecific term and does nat adequately describe the infection depicted. The presence of fuctuance with this infection suggests abscess formation rather than a simple cellulites ‘Made this question & => Question Td : 55582 Question 5 of 30 4.38 year old man comes to yen with severe pain Flictiance, erythema, and tenderness localized aver the pass of the distal long finger. Whatis the most likely diagnosis? Y © a) Afelon (whitlow) b) A paronychia ©) An epenychial abscess 4) Bacterial senosynoviis ¢) Digital cellulitis Answer | Bopanation Other User's Explanation Report An Error Question Explanation: ‘A felon, also called a whitlow, is an infection of the dgital pulp of the temminal phalanx. The anatomic cheracteristics of this region compactruentalize infoction, resulting in exquisite pressure and pain in the pulp spaces. As with all abscesses, incision and drainage is the single most impostant therapy. Paronychial and eponychial infections occur in the dorsal surface of the finger about the nail Bacterial tenosynovitis is an extension of a desp space infecticn along the tendons. Digital cellulites ic « nonspecific term and does nat adequately describe the infection depicted. The presence of fuctuance with this infection suggests abscess formation rather than a simple cellulites Mark this question & => Question Td : 74532 Question 6 of 30 Giardia lamblia is a common source of diarthea in travelers returning from endemic areas. All of the folowing are true conceming Giardia infection EXCEPT 4) Infection occurs following ingestion of environmentally herdy cysts, which excyst in the smal intestine, releasing trophozoites. ) Following small-bowel infection, hematogenous dissemination occurs ©) The trophozoie forms a morphologically distinct cyst ifit encounters unfavorable conditions 4) Infection may occur following ingestion of as few as ten cysts. ©) Water transmission is efftctive because cysis are resistant to killing by routine chlorination methods: Anower [UBRRNGWEN) otnorUeorsExplanation Report An Error Question Explanation: Giardia lamblia is a cosmopolitan protozoal parasite; Infection occurs following ingestion of environmentally hardy cysts, which excyst in the small intestine, releasing trophosoites. These tophozcites mubiply by binary fission, Giardia remains in the small bowel and does not disseminate hematogenously. Trophozoites live within the gut lumen or ettach to the tmucosal epitheliun by a ventral sucking disk. IFthe trophozoite encounters unfavorable conditions, a morphologically distinct cyst is formed. Cysts are common in the environment: Thgestion of as few as ten cysts is sufficient to cause infection in humans. Episodic infections may be caused by waterbome transmission because cysts exhibit prolonged survival in cold water and are resistant to killng by routine chlorination technique. Mark this question & => Question Td : 74532 Question 6 of 30 Giardia lamblia is a common source of diarrhea in travelers returning from endemic areas. All of the following are true concerning Giardia infection EXCEPT a) Infection occurs following ingestion of environmentally hardy cysts, which excyst in the small intestine, releasing trophozcites. Y © b) Following small-bowel infection, hematogenous dissemination occurs. c) The trophozoite forms a morphologically distinct cyst if it encounters unfavorable conditions. 4) Infection may occur following ingestion of as few as ten cysts. ¢) Water transmission is cffective because cysts are resistant to killing by routine chlorination methods. Anower [UBRRNGWEN) otnorUeorsExplanation Report An Error Question Explanation: Giardia lamblia is a cosmopolitan protozoal parasite; Infection occurs following ingestion of environmentally hardy cysts, which excyst in the small intestine, releasing trophosoites. These tophozcites mubiply by binary fission, Giardia remains in the small bowel and does not disseminate hematogenously. Trophozoites live within the gut lumen or ettach to the tmucosal epitheliun by a ventral sucking disk. IFthe trophozoite encounters unfavorable conditions, a morphologically distinct cyst is formed. Cysts are common in the environment: Thgestion of as few as ten cysts is sufficient to cause infection in humans. Episodic infections may be caused by waterbome transmission because cysts exhibit prolonged survival in cold water and are resistant to killng by routine chlorination technique. Mark this question & => Question Id : 89589 Question 7 of 30 ATO year old aursing home resident presents with fever, feeling of malaise, petechial hemorrhages in the mouth, enlarged salivary sands, and pharyngotonsilits. The causative agent of these symptoms would be which one of the following? 2) Cytomegalovirus ') Heapes zoster 6) Candida 4) Vitamin C deficiency 6) Allergic reaction to sulfonanides Answer [FESRIRRRHR) other Users Explan Question Explanation: Approximately 80% of adults 60 years of age and older have been infected with cytomegalovinises, tut symptoms and signs of infection are ucually cubcincial. Reactivation of a latent virus can cauce enlarged salivary glands, moncnucleosic He symptoms, petechial hemorrhages, and inflaranatory sore throat Herpes zoster causes dermatomal vesicular lesions which are peinfal. The salivary glands are usually not involved. Candida albicans presents as thrush in the oral cavity Itis chacacterized by soft, whte, slightly raised adherent plaques composed of hyphae, yeast, desquamated epithelial cells, and debris. Vitamin C deficiency causes scurvy in severe caces. Allergic reaction to suonamides cances Stevene Johnson syadrome and mucocutaneous lesions inthe oral cavity, but not pharyngotonsiliis or sialadenils n Report An Error Mark this question & => Question Id : 89589 Question 7 of 30 ATO year old aursing home resident presents with fever, feeling of malaise, petechial hemorrhages in the mouth, enlarged salivary sands, and pharyngotonslits. The causative agent of these symptoms would be which one of the following? WV © 2) Cytomegalovirus ') Heapes zoster 6) Candida 4) Vitamin C deficiency 6) Allergic reaction to sulfonanides Answer [FESRIRRRHR) other Users Explan Question Explanation: Approximately 80% of adults 60 years of age and older have been infected with cytomegalovinises, tut symptoms and signs of infection are ucually cubcincial. Reactivation of a latent virus can cauce enlarged salivary glands, moncnucleosic He symptoms, petechial hemorrhages, and inflaranatory sore throat Herpes zoster causes dermatomal vesicular lesions which are peinfal. The salivary glands are usually not involved. Candida albicans presents as thrush in the oral cavity Itis chacacterized by soft, whte, slightly raised adherent plaques composed of hyphae, yeast, desquamated epithelial cells, and debris. Vitamin C deficiency causes scurvy in severe caces. Allergic reaction to suonamides cances Stevene Johnson syadrome and mucocutaneous lesions inthe oral cavity, but not pharyngotonsiliis or sialadenils n Report An Error 31/2014 8: 212 PM ‘Marke this question & => Question Td : 91253 Question 8 of 30 A 63 year old man with renal call carcinoma develops a viral infection, The most likely etiologic agentin this person is a) CMV +) EBY 6) Influenza A ©) InflucrzaB oHsv Question Explanation: Herpes simplex virus and varicella zoster viruses are the most commonly infecting viruses in cancer patients Although any one of the choices listed can infect cancer paticnts, itis HSV that is most common, 31/2014 8: 212 PM ‘Marke this question & => Question Td : 91253 Question 8 of 30 A 63 year old man with renal cell carcinoma develops a viral infection. The most likely etiologic agent in this person is a) CMV b) EBV. ) Influenza A Influenza B ¥ 0 eo HSY Question Explanation: Herpes simplex virus and varicella zoster viruses are the most commonly infecting viruses in cancer patients Although any one of the choices listed can infect cancer paticnts, itis HSV that is most common, ‘Marke this question <= => Question 9 of 30 “What is the best way to prevent congerital rubella syndrome’? a) Tinmunization of pregnant women. 5) Immunization of teenagers ©) Administration of immune serum to women who have not been vaccinated. 4) Immurization of all young children €) Avoidance by pregnant women of infected individuals. Question Explanation: The best way to prevent congenital rubella syndrome is universal vaccination of all chidren, Pregnant women should not be immunized, since this may lead to rubella infection in the fetus. Iinmunization of teenagers is not wise, since a pregnant woman might bbe inadvertently immunized, Immune serum has not been demonstrated to be beneficial in protecting against rubella infection. Avoidance of infected indwiduas is 2 poor way to try to prevent rubella, since many infected persons may be asymptomatic. ‘Marke this question <= => Question 9 of 30 “What is the best way to prewent congerital rubella syndrome? a) Thormunization of pregnant women. 'b) Immurization of teenagers ©) Administration ofimmune serum to women who have not been vaccinated. SY © 4) Immurization of all young children, ©) Avoidance by pregnant women of infected individuals Question Explanation: The best way to prevent congenital rubella syndrome is universal vaccination of all chidren, Pregnant women should not be immunized, since this may lead to rubella infection in the fetus. Iinmunization of teenagers is not wise, since a pregnant woman might bbe inadvertently immunized, Immune serum has not been demonstrated to be beneficial in protecting against rubella infection. Avoidance of infected indwiduas is 2 poor way to try to prevent rubella, since many infected persons may be asymptomatic. 3/1/2014 8:13:54 PM ‘Marke this question & => Question 10 of 30 “What is the most common cause of infective endocarditis in intravenous drag abusers? a) Streptococcus pneumoniae. £) Staphylococcus aureus, ©) Escherichia coil ©) Neisseria meningitidis. ©) Flebsiella pneumonias Question Explanatio Question Td : 92953 Staphylococcus aureus is the most common cause of infective endocarditis in the TVD.A population. The other organisms are less common, 3/1/2014 8:13:54 PM ‘Marke this question & => Question 10 of 30 “What is the most common cause of infective endocarditis in intravenous drag abusers? a) Streptococcus pneumoniae. Y © b) Staphylococcus aureus. ©) Escherichia coil 6) Neisseria meningitidis. 6) Klebsiella pneumoniae Question Explanatio Question Td : 92953 Staphylococcus aureus is the most common cause of infective endocarditis in the TVD.A population. The other organisms are less common, 3/1/2014 8: 208 PH Marke this question e& => Question Td : 96195 Question 11 of 36 “What is the number one cause of urinary tract infection? a) Enterococcus faecalis b) Klebsiella pneumoniae ©) Proteus mirabilis 4) Escherichia col €) Poeudomonas aeruginosa Avower [UEQIRIBEM) otter veorsExplanation Report An Error Question Explanation: Escherichia coll is the number ons cause of urinary tract infections. The other organisms are also possible causes, but much less likely. 3/1/2014 8: 208 PH Marke this question e& => Question Td : 96195 Question 11 of 36 “What isthe number one cause of urinary tract infection? 2) Enterococcus faecalis ) Klebsiella pneumoniae ©) Proteus mirabilis Y © 4) Escherichia coli €) Poeudomonas aeruginosa Avower [UEQIRIBEM) otter veorsExplanation Report An Error Question Explanation: Escherichia coll is the number ons cause of urinary tract infections. The other organisms are also possible causes, but much less likely. 3/1/2044 8:14:24 PM Mark this question & => Question Td : 96680 Question 12 of 30 Important immanologic factors in Legionnaires disease include all of the folowing EXCEPT 2) Antbedies are impertantin host defence. 6) Tymphocytes play a major rele in the immune process, ©) Bacterial uptake and multiplication are curtailed by the action of cytokines. 4) Natural killer and lymphokine activated kcller cells lyse infected macrophages. ©) Legionella activate complement, but are resistant to lysis. Question Explanation: Legionella activate complements by the classic pathways. Even though specific antbodies are produced, the organisms remain resistant to this complement mediated lysis. Antibodies have little fanction in either host defense or smmunity. ‘T-lymphocytes play a major role in the immune process. Bacterial uptake and multiplication ate curiailed by the action of cytokines. particularly interferon, The iniraceliular cycle of infection is aborted by natural killer and lymphokine activated killer cells through lysis of infected macrophages, n Report An Error 3/1/2044 8:14:24 PM Mark this question & => Question Td : 96680 Question 12 of 30 Important immunologic factors in Legionnaires disease include all of the folowing EXCEPT Y © 2) Antibodies are important in host defense. 6) Tymphocytes play a major rele in the immune process, ©) Bacterial uptake and multiplication are curtailed by the action of cytokines. ) Natural killer anc lymphokine activated keller cells lyse infected macrophages, ©) Legionella activate complement, but are resistant to lysis. Question Explanation: Legionella activate complements by the classic pathways. Even though specific antbodies are produced, the organisms remain resistant to this complement mediated lysis. Antibodies have little fanction in either host defense or smmunity. ‘T-lymphocytes play a major role in the immune process. Bacterial uptake and multiplication ate curiailed by the action of cytokines. particularly interferon, The iniraceliular cycle of infection is aborted by natural killer and lymphokine activated killer cells through lysis of infected macrophages, n Report An Error Mark this question —& => Question Td : 141701 Question 13 of 30 A medical worker has a needle stick accident involving an ermpty syringe that had been used on a hepatitis B patient. The most probable outcome forthe medical worker is which one of the following” a) Acute hepatitis followed by recovery ) "Healthy" carsior ©) Persistent: infection followed by recovers! 4) Persistent infection progressing io chronic hepatitis ©) Subclnical disease followed by recovery Question Explanation: Hepatitis B infection can produce @ wide variety of clinical outcomes. The most commen outcome (60% to 65%), however, tums out tp be subclinical disease followed by complete recovery. The other choices listed show other possible outcomes, and ther statistical impact is considered with the discussions of individual choices. Approximately 20% 10 25% of infected persons develop acute hepatitis which is followed in 99% of these cases by recovery andiin about 1% of cases by fulminant hepatitis. Approximately 5% to 10% of cases become “healthy” carriers. Approvimately 4M of cases develop persistent infection, 67% to 90% of which then recover and 10% to 33% of which have chronic hepatitis Mark this question —& => Question Td : 141701 Question 13 of 30 A medical worker has aneedle sticle accident invoWving an empty syringe that had been used on a hepatitis B patient. The most probable outcome for the medical worker is which one of the following? 2) Acute hepatitis followed by recovery b) "Healthy" carrier ¢) Persistent: infection followed by recovers! d) Persistent infection progressing to chronic hepaiitis Y © ©) Subclinical disezse followed by recovery Question Explanation: Hepatitis B infection can produce @ wide variety of clinical outcomes. The most commen outcome (60% to 65%), however, tums out tp be subclinical disease followed by complete recovery. The other choices listed show other possible outcomes, and ther statistical impact is considered with the discussions of individual choices. Approximately 20% 10 25% of infected persons develop acute hepatitis which is followed in 99% of these cases by recovery andiin about 1% of cases by fulminant hepatitis. Approximately 5% to 10% of cases become “healthy” carriers. Approvimately 4M of cases develop persistent infection, 67% to 90% of which then recover and 10% to 33% of which have chronic hepatitis Mark this question ez ‘Question Id: 141752 Question 14 of 30 A farmer develops abdominal pain and dianthea followed several days later by fever, periorbital edema, eosinephila, and myalgia He does not remember eating anything unusual recently, but notes thet he does make pork sausage made by her wife. Technique that would be helpfil for the diagnosis of this patient is 8) Gastric biopsy ) Muscle biopsy 6) Scotch tape test 6) Stool for ova €) Stool for protozoal parasites Question Explanati ‘The patient probably has trichinosis; the diagnosis is confirmed by demonstration of cysts in a muscle biopsy. Trichinosis is uncommon i this country, becanse the organisms are usually found in pork and can be kled ty adequate cooking, Cases are usually due to tasting of raw pork sausage (for seasoning purposes) of ingestion of poorly cooked bear meat, The organism is not usually demorstrated in stool or gastric biopsies. The scotch tape testis for Enterobius (pinwerms) Mark this question ez ‘Question Id: 141752 Question 14 of 30 A farmer develops abdominal pain and dianthea followed several days later by fever, periorbital edema, eosinephila, and myalgia He does not remember eating anything unusual recently, but notes thet he does make pork sausage made by her wife. Technique that would be helpfil for the diagnosis of this patient is 8) Gastric biopsy ¥ © 8) Muscle biopsy 6) Scotch tape test 6) Stool for ova €) Stool for protozoal parasites Question Explanati ‘The patient probably has trichinosis; the diagnosis is confirmed by demonstration of cysts in a muscle biopsy. Trichinosis is uncommon i this country, becanse the organisms are usually found in pork and can be kled ty adequate cooking, Cases are usually due to tasting of raw pork sausage (for seasoning purposes) of ingestion of poorly cooked bear meat, The organism is not usually demorstrated in stool or gastric biopsies. The scotch tape testis for Enterobius (pinwerms) Mark this question & => Question Td : 145448 Question 15 of 30 A 33 year old HIV positive men develops severe, multifocal encephalitis. His history is remarkable for the outbreak of a vesicular unilateral rach one month cavlier. Therapy is insituted with acyclovir, but the man dies on the fourth day of his hospital admission. The most likely cause of his encephalitis was 2) Cytomegalovins 1b) Herpes simplex type I ©) Herpes simplex type TL 46) Herpes zoster-varicella ©) Measles viras Anowor (REIIRNRNBN) othe: User's explana Question Explanation: ‘The specific clue to the cause of the severe encephaltis in this AIDS patient is the history of shingles, due to reactivation of the herpes zoster varicella virus. In otherwise healthy adults the virus (which is usually introduced to the body as a childhood case of chickenpox) remains dormant in a dorsal soot ganglion, only to reactivate in later life, causing a painful vesicular eruption that characteristcally conforms to the distribution of a single dermatome. In ATDS patients, the vinis can cause severe, tmutifocal encephaliis that may be resistant to acyclovir therapy. Cytomegalovinis can cause disseminated disease (including brain infection) in AIDS patients, but is less likely in this patient, given the rast histery of shingles. Herpes simplex type I and herpes simplex type II can cause disseminated disease (cluding brain infection) in “AIDS patients, bat is less likely in this patent, given the past history of shingles, and the lack oF response to acyclovir. Measles virus appears to be related to subacute sclerosing pancacephaliis, but this condition is not specifically increased in AIDS patients. Report An Error Mark this question & => Question Td : 145448 Question 15 of 30 A 33 year old HIV positive men develops severe, multifocal encephalitis. His history is remarkable for the outbreak of a vesicular unilateral rach one month cavlier. Therapy is insituted with acyclovir, but the man dies on the fourth day of his hospital admission. The most likely cause of his encephalitis was 2) Cytomegalovins 1b) Herpes simplex type I ©) Herpes simplex type TL Y © 6) Herpes zoster-varicella ©) Measles viras Anowor (REIIRNRNBN) othe: User's explana Question Explanation: ‘The specific clue to the cause of the severe encephaltis in this AIDS patient is the history of shingles, due to reactivation of the herpes zoster varicella virus. In otherwise healthy adults the virus (which is usually introduced to the body as a childhood case of chickenpox) remains dormant in a dorsal soot ganglion, only to reactivate in later life, causing a painful vesicular eruption that characteristcally conforms to the distribution of a single dermatome. In ATDS patients, the vinis can cause severe, tmutifocal encephaliis that may be resistant to acyclovir therapy. Cytomegalovinis can cause disseminated disease (including brain infection) in AIDS patients, but is less likely in this patient, given the rast histery of shingles. Herpes simplex type I and herpes simplex type II can cause disseminated disease (cluding brain infection) in “AIDS patients, bat is less likely in this patent, given the past history of shingles, and the lack oF response to acyclovir. Measles virus appears to be related to subacute sclerosing pancacephaliis, but this condition is not specifically increased in AIDS patients. Report An Error Mark this question <& => Question Td : 145698 Question 16 of 30 An 18-year old student presents with a2 week history of fever, chills, and a sore throat. His temperatare is 38.30C (1010F), blood pressure is 110/70 mm Hz, pulse is 70imin, and respirations are 1B/min, Examination shows marked pharyngeal hyperemia, tonsilar exudates, cervical lymphadenopathy, and splenomegaly Ampicillin therapy is begua. Two days later, he returns because ofa maculopapular rash. The most lice test t> confirma the diegnosis is 2) Chest Xray +) ELISA ©) Heterophile antibody test 46) Lateral x-ray film of the neck. ©) Rapid strep test Answer [REIRIRIRHBR) other User's Explanation Report An Fsror Question Explanation: This patient most ely has infectious mononncleosis, which is caused by the Epstein-Barr virus. The clinical features incinde sore throat headache fever, malaise, lymphadenopathy, pharyngitis and tonsilitis hepatosplenomegaly, periorbital edema, rash, and palatal exanthem, The diagnosis s made by heterophile antibody testing EBV antibody titers, and lymphocytosis with atypical lymphocytes ‘Treatment includes rest fluids, and analgesics. Treatment with penicillin derivatives (ampicillin) leads to a maculopapular rash. Chest “Cray is usefil for a respiratory or cardiac process. Infections mononucleosis cannot be diagnosed by any radiographic study. An ELISA would be usefid screening test for a variety of infections, including human immunodeficiency virus. A leteral X-ray of the neck 4s useful in acute epiglottits. It would show epiglottic swelling ‘The symptoms of epiglotttis include extreme sore throat, drooling, and fficuty swallowing A rapid strep testis used to diganose streptococcal pheryngitis. Strentococcal pharyngitis is characterized by fever core throat tonsillar exudates, and cervical, lymphadenopathy. A scasiatiniform rash may follow the pharyngitis. The rash, a ciffise erythema that leter desquamates, is caused by an erythrotoxin Mark this question <& => Question Td : 145698 Question 16 of 30 An 18-year old student presents with a2 week history of fever, chills, and a sore throat. His temperatare is 38.30C (1010F), blood peeccure ic 110/70 mam Hg, pulce is 70lmin, and respirations are 18/min. Examination shows masked pharyngeal hyperemia, tonsilar exudates, cervical lymphadenopathy, aud splenomegaly. Ampicilin therapy is begun. Two days later, he returns because ofa ‘maculopapular rash. The most likely test to confirma the diagnosis is 2) Chect X-ray +) ELISA WY © ©) Heterophile antibody test 6) Lateral x-ray film of the neck 6) Ragid strep test Answer [REIRIRIRHBR) other User's Explanation Report An Fsror Question Explanation: This patient most ely has infectious mononncleosis, which is caused by the Epstein-Barr virus. The clinical features incinde sore throat headache fever, malaise, lymphadenopathy, pharyngitis and tonsilitis hepatosplenomegaly, periorbital edema, rash, and palatal exanthem, The diagnosis s made by heterophile antibody testing EBV antibody titers, and lymphocytosis with atypical lymphocytes ‘Treatment includes rest fluids, and analgesics. Treatment with penicillin derivatives (ampicillin) leads to a maculopapular rash. Chest “Cray is usefil for a respiratory or cardiac process. Infections mononucleosis cannot be diagnosed by any radiographic study. An ELISA would be usefid screening test for a variety of infections, including human immunodeficiency virus. A leteral X-ray of the neck 4s useful in acute epiglottits. It would show epiglottic swelling ‘The symptoms of epiglotttis include extreme sore throat, drooling, and fficuty swallowing A rapid strep testis used to diganose streptococcal pheryngitis. Strentococcal pharyngitis is characterized by fever core throat tonsillar exudates, and cervical, lymphadenopathy. A scasiatiniform rash may follow the pharyngitis. The rash, a ciffise erythema that leter desquamates, is caused by an erythrotoxin Mark this question & => ‘Question Id : 146808 Question 17 of 30 17 year old giel presents with sore throat with a rash On examination, she is febrile, has an exudative pharyngits with tonsillar hypertrophy and a strawberry tongue. Her anterior cervical nodes are cnlarged and mildly tender. She also has a diffuse erythematous rash that has the feel of sandpaper. The enhanced virulence of the infectious agent causing the illness is an example of a) Conjugation +b) Homelogous recombination c) Lysogenic conversion 4) Site-specific recombination ¢) Transformation Anower [ETIRIIENY) ote Ueors Exptana Question Explanation: ‘The question is describing a gil with streptococcal pharyngitis with a characteristic rash. This combination is scarlet fever, and is caused by S. pyogenes via its erythrogenic exotoxins SPE-A, SPE, B, and SPE-C. The production of this extra virulence is an example oflysogenic conversion. In this case, a repressed temperate phage DNA is inserted into the bacterial chromosome, and it becomes a prophage. Besides the repressor protein, the prophage DNA also directs the synthesis of another gene product that makes the bacteria more pathogenic. Conjugation is one of the three methods whereby TINA is trarsferred from one cell to ansther Tn this cese, itis done by cell-to-cell contact, Homelogous recombination is a gene exchange process that stabilizes some genes that have been introdaced by conjugation transformation, of transduction, During the process, there is an exchange of pieces of DNA between a linear piece of DNA and arregion on the stable circular bacterial chromosome. In ste specific recombination aDNA molecule is integrated ints 2 molecule of DIA with which it has no homology, except for a small site on each DNA called an insertion site, The cad result is the sum of the two original DNA molecules. Transformation is another way in which genes ars transferred, In ths case, naked DINA ss taken up by another cell. Cells that can take up naked DNA fiom the environment need to ‘become competent, and do so under certain environmental conditions. The newly introduced DNA is generally linear, homologous DNA fromthe same type of cell, but one that maybe genetically diverse, Once introduced into the new cell, some genes may become stabilixed by homologous recombination, Report An Error Mark this question & => ‘Question Id : 146808 Question 17 of 30 17 year old giel presents with sore throat with a rash On examination, she is febrile, has an exudative pharyngits with tonsillar hypertrophy and a strawberry tongue. Her anterior cervical nodes are cnlarged and mildly tender. She also has a diffuse erythematous rash that has the feel of sandpaper. The enhanced virulence of the infectious agent causing the illness is an example of a) Conjugation +b) Homelogous recombination Y © 6) Lysogenic conversion 4) Site-specific recombination ¢) Transformation Anower [ETIRIIENY) ote Ueors Exptana Question Explanation: ‘The question is describing a gil with streptococcal pharyngitis with a characteristic rash. This combination is scarlet fever, and is caused by S. pyogenes via its erythrogenic exotoxins SPE-A, SPE, B, and SPE-C. The production of this extra virulence is an example oflysogenic conversion. In this case, a repressed temperate phage DNA is inserted into the bacterial chromosome, and it becomes a prophage. Besides the repressor protein, the prophage DNA also directs the synthesis of another gene product that makes the bacteria more pathogenic. Conjugation is one of the three methods whereby TINA is trarsferred from one cell to ansther Tn this cese, itis done by cell-to-cell contact, Homelogous recombination is a gene exchange process that stabilizes some genes that have been introdaced by conjugation transformation, of transduction, During the process, there is an exchange of pieces of DNA between a linear piece of DNA and arregion on the stable circular bacterial chromosome. In ste specific recombination aDNA molecule is integrated ints 2 molecule of DIA with which it has no homology, except for a small site on each DNA called an insertion site, The cad result is the sum of the two original DNA molecules. Transformation is another way in which genes ars transferred, In ths case, naked DINA ss taken up by another cell. Cells that can take up naked DNA fiom the environment need to ‘become competent, and do so under certain environmental conditions. The newly introduced DNA is generally linear, homologous DNA fromthe same type of cell, but one that maybe genetically diverse, Once introduced into the new cell, some genes may become stabilixed by homologous recombination, Report An Error Merk this question & => Question Ti : 149560 Question 18 of 30 A.25 year old female has a 3 day history of fever, chills, chest pain, and cough productive of reddich-grown sputum, She had splenectomy 1 year ago. CXR shows consalidetion of the right lower lobe, Blood enltures are pasitive fer a-hemolytic gram-positive diplococei. Inmunity to the infectious agent is based on 2) Alternative complement pathway activation +) Antibody to ant alpha-helisal coiled fimbria ©) IgA antibodies to C carbohydrate ) IgG antibodies to C carbohydrate ©) Antbodies to a surface avidic polysaccharide Answer { Baienaton) Other User's Explanation Report An Error Question Explanation: ‘The patient in this question has pneumococcal pneumonia, which must be considered in any patient with chil fever chest pain and cough productive of purulent, rust-colored sputum. Streptococcus pneumoniae is en a-hemolytic, gram-positive coccus that grows in chains. It can be easly distinguished from other alpha hemolytic streptococci because its exquisitely censitive to bile and bile lice compounds, such as optochin, Iris the most common cause of community acquiced paeumonia and the most common cause of community acquired meningtts in adults older than 30. The only recognized virulence factor of S. pneumoniae is its carbohydrate capsule (which contains acidic polysaccherides), Antibady to a specific capsule type is necessary to overcome irfectien. More than 20 capsule types have been recognized ‘The 23 types that most commonly cause disease are contained in a vaccine that is recommended for high risk groups, including the elderly and those undergoing splenectomy. Increased susceptibility is also found in patients with Hodgkin disease, chronic lymphocytic leukemia, and myeloma, The vaccine should stil be given to patients with these conditions, butit is less successfil The alternative coraplement pathway is important in clearing Neisseria irfections. Individuals with dleficieacies in C5 through CB are at increased risk of disease from Neisseria. The fimbria of Streptococeus progenes (Group A bbeta-hemolstic streptococcus) is composed of an alpha-helically coiled M protein, Antibody against a specific M type will prevent infection, However, raising antivodies to M proteins can lead to rheumatic fever, so Strep throat infections are routinely treated with ppenicilin to prevent an antibody response. The C! carbohydrate is an antigen of [hemolytic streptocacci used to divide ther into different groups. Antibody against C cerbohydrate is net protective Merk this question & => Question Ti : 149560 Question 18 of 30 A.25 year old female has a 3 day history of fever, chills, chest pain, and cough productive of reddich-grown sputum, She had splenectomy 1 year ago. CXR shows consalidetion of the right lower lobe, Blood enltures are pasitive fer a-hemolytic gram-positive diplococei. Inmunity to the infectious agent is based on 2) Alternative complement pathway activation +) Antibody to ant alpha-helisal coiled fimbria ©) IgA antibodies to C carbohydrate ) IgG antibodies to C carbohydrate Y © 6) Antbodies to a surface atidic polysaccharide Answer { Baienaton) Other User's Explanation Report An Error Question Explanation: ‘The patient in this question has pneumococcal pneumonia, which must be considered in any patient with chil fever chest pain and cough productive of purulent, rust-colored sputum. Streptococcus pneumoniae is en a-hemolytic, gram-positive coccus that grows in chains. It can be easly distinguished from other alpha hemolytic streptococci because its exquisitely censitive to bile and bile lice compounds, such as optochin, Iris the most common cause of community acquiced paeumonia and the most common cause of community acquired meningtts in adults older than 30. The only recognized virulence factor of S. pneumoniae is its carbohydrate capsule (which contains acidic polysaccherides), Antibady to a specific capsule type is necessary to overcome irfectien. More than 20 capsule types have been recognized ‘The 23 types that most commonly cause disease are contained in a vaccine that is recommended for high risk groups, including the elderly and those undergoing splenectomy. Increased susceptibility is also found in patients with Hodgkin disease, chronic lymphocytic leukemia, and myeloma, The vaccine should stil be given to patients with these conditions, butit is less successfil The alternative coraplement pathway is important in clearing Neisseria irfections. Individuals with dleficieacies in C5 through CB are at increased risk of disease from Neisseria. The fimbria of Streptococeus progenes (Group A bbeta-hemolstic streptococcus) is composed of an alpha-helically coiled M protein, Antibody against a specific M type will prevent infection, However, raising antivodies to M proteins can lead to rheumatic fever, so Strep throat infections are routinely treated with ppenicilin to prevent an antibody response. The C! carbohydrate is an antigen of [hemolytic streptocacci used to divide ther into different groups. Antibody against C cerbohydrate is net protective Mark this question —& => Question Td : 149769 Question 19 of 30 A 62 year old aleoholic smoker develops high fever. shakes. a severe headache, and muscle pain. He has a dry insignificant cough initiclly but over the next few days he develope marked chortuece of breath requiring accisted vertilation. CXR shows homogenecus radiographic shadowing that intialy involves the lett lower lobe but continues to spread ntl both lungs are extensively involved. Culture of bronchoalveolar lavage on buffered charcoal yeast extract (BCYE) shows a coccobatillary pathogen. The most likely infections agent is a) Legionella pneumophila +) Listeria monocytagenes ) Spitilium 4) Staphylococcus aureus ) Streptococcus pneumoniae Answer | Explanation Other User's Explanation Report An Error Question Explanation: ‘The patient has a severe potentially fata, pneumonia with prominent systemic symptoms. Cultare on BCYEis the specific clue that the organism is Legionella pneumoptila, The disease is respiratory Legionellosis. also known as Legionnaire disease. because the disease was first described when + occurred in epidemic form following an American Legion convention at a Philadelphia hotel. Patients tend to be clder (40-70 years old) and may have risk factors including cigarette use, alcoholism, diabetes, chronic iiness, or immanosuppressive therapy. Listeria monocytogenes causes listeriosis andis aot a notable cause of pneumonia. Spitillum minus is a cause of rat-bite fever and is not 2 notable cause of pneumonia, Staphylococcus aurens can cause pneumonia, but is easily cultured on routine media, Streptococcus pneumoniae can cause pacumonia, butis easily cultured on routine media Mark this question —& => Question Td : 149769 Question 19 of 30 A 62 year old aleoholic smoker develops high fever. shakes. a severe headache, and muscle pain. He has a dry insignificant cough initiclly but over the next few days he develope marked chortuece of breath requiring accisted vertilation. CXR shows homogenecus radiographic shadowing that intialy involves the lett lower lobe but continues to spread ntl both lungs are extensively involved. Culture of bronchoalveolar lavage on buffered charcoal yeast extract (BCYE) shows a coccobatillary pathogen. The most likely infections agent is Y © a) Legionella pneumophila +) Listeria monocytagenes ) Spitilium 4) Staphylococcus aureus ) Streptococcus pneumoniae Answer | Explanation Other User's Explanation Report An Error Question Explanation: ‘The patient has a severe potentially fata, pneumonia with prominent systemic symptoms. Cultare on BCYEis the specific clue that the organism is Legionella pneumoptila, The disease is respiratory Legionellosis. also known as Legionnaire disease. because the disease was first described when + occurred in epidemic form following an American Legion convention at a Philadelphia hotel. Patients tend to be clder (40-70 years old) and may have risk factors including cigarette use, alcoholism, diabetes, chronic iiness, or immanosuppressive therapy. Listeria monocytogenes causes listeriosis andis aot a notable cause of pneumonia. Spitillum minus is a cause of rat-bite fever and is not 2 notable cause of pneumonia, Staphylococcus aurens can cause pneumonia, but is easily cultured on routine media, Streptococcus pneumoniae can cause pacumonia, butis easily cultured on routine media Mark this question = => Question Td : 150118 Question 20 of 30 A 25 year old female is evaluated for diffase pelvic pain. Her husband was treated 1 month ago for a sexually transmitted infection, ‘but she was not treated at that time. She is now febrile and has a tender mass around herleft adnexa, The mechanism by which the most licey infectious agent may cause fallopian tube scarring is a) Cytotoxic T lymphocyte mediated cytolysis +) Immune complex mediated cytolysis ©) Neutrophil infiltration 4) Stimulation of interleukin & ©) Type I hypersensitivity Question Explanation: This woman has developed pelvic inflammatory disease, one cf the sequelae of untreated sexually transmitted diseases Ascending infections involving cither gonccocci or Chlamydia may cater the uterine tubes producing salpingitis and eventually infecting the fimbriated ends and the ovary In the United States, Chlamycla trachomatis is the most common bacterial sexually transmitted disease Chlamycia are intracellular pathogens, and thus elicit a TH] lymphocyte response and stimulation of CDE+ lymphocyte effector cells ‘The damage and scarring of the fallopian tubes, therefore, is at least in part due to the development of a type IV hypersensitivity reaction, Trane complex-mediated cytolysis is involved in the production of rashes in type TIT hypersensitivties. Since Chlamydia are intracellular pathogens, however, antibody formation does not play a major role in pathogenesis. Neutrophilic infiltration is a common mecharism of abscess and pus formation in Tcases in which extracellular pathogens invade the fissaes. Since Chlamydia are intracellular organisms, they would not stimulate the influx of neutrophils. Stimulation of interleukin-8 production occurs in areas, where extracellular pathogens invade the tissues. Interleukin-8 is a product of tissue mactophages that is strongly chemotactic for neutrophils. Type I hypersensitivity involves autoantibodies binding to self cell or tissue antigens, and causes damage via complement activation or phagocytosis. Tt is a mechanism of damage to the heart in rheumatic fever, but would not he a common mechanism of Report An Error pathogenesis in an intracelldar pathogen such as Chlamydia. Mark this question = => Question Td : 150118 Question 20 of 30 A 25 year old female is evaluated for diffase pelvic pain. Her husband was treated 1 month ago for a sexually transmitted infection, ‘but she was not treated at that time. She is now febrile and has a tender mass around herleft adnexa, The mechanism by which the most licey infectious agent may cause fallopian tube scarring is Y¥ © a) Cytotoxic T lymphocyte mediated cytolysis +) Immune complex mediated cytolysis ©) Neutrophil infiltration 4) Stimulation of interleukin & ©) Type I hypersensitivity Question Explanation: This woman has developed pelvic inflammatory disease, one cf the sequelae of untreated sexually transmitted diseases Ascending infections involving cither gonccocci or Chlamydia may cater the uterine tubes producing salpingitis and eventually infecting the fimbriated ends and the ovary In the United States, Chlamycla trachomatis is the most common bacterial sexually transmitted disease Chlamycia are intracellular pathogens, and thus elicit a TH] lymphocyte response and stimulation of CDE+ lymphocyte effector cells ‘The damage and scarring of the fallopian tubes, therefore, is at least in part due to the development of a type IV hypersensitivity reaction, Trane complex-mediated cytolysis is involved in the production of rashes in type TIT hypersensitivties. Since Chlamydia are intracellular pathogens, however, antibody formation does not play a major role in pathogenesis. Neutrophilic infiltration is a common mecharism of abscess and pus formation in Tcases in which extracellular pathogens invade the fissaes. Since Chlamydia are intracellular organisms, they would not stimulate the influx of neutrophils. Stimulation of interleukin-8 production occurs in areas, where extracellular pathogens invade the tissues. Interleukin-8 is a product of tissue mactophages that is strongly chemotactic for neutrophils. Type I hypersensitivity involves autoantibodies binding to self cell or tissue antigens, and causes damage via complement activation or phagocytosis. Tt is a mechanism of damage to the heart in rheumatic fever, but would not he a common mechanism of Report An Error pathogenesis in an intracelldar pathogen such as Chlamydia. 3/1/2014 8:16:53 PM ‘Marke this question & => Question Ta : 150859 Question 21 of 30 An 18 year old student presents with tender lymph nodes in her necke and the left side. Her past medical history is not significant. Two weeks ago, she updated her vaccines in preparation for college. Lymph node biopsy shows benign paracortical expansion and scattered multinucleated giant cells with eosinophilic cytoplasmic and nuclear inclusion bodies. The vaccine that is most likely responsible for this woman's lymphadenitis is a) Hepatitis B ) Measles ) Rubella 4) Steallpox «) Tetanus Question Explanation: ‘The Warthin-Finkeldey (WF) giant cell described here is pathognomonic for measles o: the lve aitemuated measles vaccine. Most gant cells are composed of histiocytes, but the WE giant cellis created by fusion of limphiocytes. Although postwaccinal igmphadentis may be seen with different vaccines, the usual reaction is amimunoblastic proliferation with in the paracortical regions of a hyperplastic lymph node. Hepatits B, rubella, and tetanus are rarely associated with postvaccinial iymphaderitis. Smallpox is classically followed by tender regional adenopalty, one to several weeks following immunization, There are no associated giant cells, 3/1/2014 8:16:53 PM ‘Marke this question & => Question Ta : 150859 Question 21 of 30 An 18 year old student presente with tender Ipmph nodes ia her neck and the left side, Her past medical history ie not significant. Two ‘weeks ago, she updated her vaccines in preparation for college. Lymph node biopsy shows benign paracortical expansion and scattered rmultinucleated gient cells with eosinophilic cytoplasmic and nuclear inclusion bodies. The vaccine thatis most likely responsible for this woman's lymphadenitis ie 2) Hepatitis B WV © b) Measles ) Rubella 4) Smallpox «) Tetanus Question Explanation: ‘The Warthin-Finkeldey (WF) giant cell described here is pathognomonic for measles o: the lve aitemuated measles vaccine. Most gant cells are composed of histiocytes, but the WE giant cellis created by fusion of limphiocytes. Although postwaccinal igmphadentis may be seen with different vaccines, the usual reaction is amimunoblastic proliferation with in the paracortical regions of a hyperplastic lymph node. Hepatits B, rubella, and tetanus are rarely associated with postvaccinial iymphaderitis. Smallpox is classically followed by tender regional adenopalty, one to several weeks following immunization, There are no associated giant cells, ‘Mark this question & => (Question Td : 201525 Question 22 of 30 Gut of the following, which one is a sign ofimmunedeficiency in the mouth? 2) Gingivitis b) Herpes labialis ©) Leucoplakia 6) Oral hairy leucoplakia 6) Oral wart Anewor [RERIRIBHERY) Other Users Explanation Report An Ever Question Explanation: ral hairy leucoplakia is a sign of immunodefiency. Ttis due to reactivation of Epstein-Barr virus infection. Gingivitis is not a sign of immminodefiency, Leuroplakia is not a sian of immnodefiency butit is a precancerous lesion, Henpes labialis is due to herpes simplex infection, which causes cold sores in immunocompetent patents and chronic herpes labialis in immunocompromised patierts. Oral warts can occur in healthy people. Ibis due to HPV infection usually due to benign types, 6 and 11 ‘Mark this question & => (Question Td : 201525 Question 22 of 30 Gut of the following, which one is a sign ofimmunedeficiency in the mouth? @) Gingivitis ) Hespes labialis ©) Leucoplakia © 4) Oral hairy leucoplakia 6) Oral wart Answer | Bsplanation Other User's Explanation Report An Error Question Explanation: Oral hairy leucoplakiais a sign of imnminodefioncy. Ibis dus to reactivation of Epstein-Barr virus infection. Gingivitis is not a sign of jmmunodefiency. Leuroplakia is not a sian of immnodefiency butit is a precancerous lesion, Henpes labialis is due to herpes simplex infection, which causes cold sores in immunocompetent patents and chronic herpes labialis in immunocompromised patierts. Oral warts can occur in healthy people. Ibis due to HPV infection usually due to benign types, 6 and 11 Mark this question e& => Question Td : 203524 Question 23 of 30 A 42-year-old man presented with tiredness, breathlessness, and nose bleeds for three weeks. On examination there were several ‘bruises on his arms and legs, 2 cm splenomegaly and funclal haemomrhages. Investigations revealed: [Hacmoglobin [7.2 gall (13.0-18.0) [White cell count |13.8 x109/L (4-11 2109) Platelet count [24 2109/1. (150-400 x 109) [Blood fia white cells predominantly mayeloblacts and promyelocytes Promyelocytes ‘Which of the following investigations would be most prognostic value? a) cerebrospinal uid examination ) cytochemistry 6) cytogenic karyotype 4) Tinmunophenotyping €) Bone marrow trephine biopsy Anower [UBIIRIENY) ter cor Explanation Report An Error Question Explanation: Cytogenetic evahtation of malignant haematological cells may have important implications for the prognosis and treatment options in, For example t(8:21) confers a good prognozis in adult AML, and about 70% of patients inthis low-risic group can be cured with intensive chemotherapy alone, radiotherapy being reserved for patients who relapse. Mark this question e& => Question Td : 203524 Question 23 of 30 A 42-year-old man presented with tiredness, breathlessness, and nose bleeds for three weeks. On examination there were several ‘bruises on his arms and legs, 2 cm splenomegaly and funclal haemomrhages. Investigations revealed: [Hacmoglobin [7.2 gall (13.0-18.0) [White cell count |13.8 x109/L (4-11 2109) Platelet count [24 2109/1. (150-400 x 109) [Blood fia white cells predominantly mayeloblacts and promyelocytes Promyelocytes “Which of the following investigations would be most prognostic value? a) cerebrospinal uid examination ) cytochemistry oY © 0) cytogenic karyotype 4) Tinmunophenotyping €) Bone marrow trephine biopsy Anower [UBIIRIENY) ter cor Explanation Report An Error Question Explanation: Cytogenetic evahtation of malignant haematological cells may have important implications for the prognosis and treatment options in, For example t(8:21) confers a good prognozis in adult AML, and about 70% of patients inthis low-risic group can be cured with intensive chemotherapy alone, radiotherapy being reserved for patients who relapse. Merk this question & => Question Ta : 206304 Question 24 of 30 A.15 year old girl is a close friend of student whe has developed meningtis. The last contact she had with her friend was two days ago when her ftiend developed headache. She has not received any previous vaccination for meningtis. Tae most appropriate action for this gil is which one of the following? 2) No treatments sequited and the grl can be reassured +) Treat with rifampicin only ©) She should receive the meningococcal A and C vaccination only 4) She should receive meningococcal iemuneglobilin only 6) She should receive the meningococcal A and C vaccinetion plus rifampicin Question Explanation: This gil runs a reasonably high risk of developing meningitis and should receive Meningitis vaccination together with rifampicin Antibiotics used for chemoprophylaxis are rifampicin, mnocycline, spiramycia, ciprofloxacin and ceftriaxone. Merk this question & => Question Ta : 206304 Question 24 of 30 A.15 year old girl is a close friend of student whe has developed meningtis. The last contact she had with her friend was two days ago when her ftiend developed headache. She has not received any previous vaccination for meningtis. Tae most appropriate action for this gil is which one of the following? 2) No treatments sequited and the grl can be reassured +) Treat with rifampicin only ©) She should receive the meningococcal A and C vaccination only 4) She should receive meningococcal iemuneglobilin only V © 6) She should receive the meningococcal A and C vaccination plus rifampicin Question Explanation: This gil runs a reasonably high risk of developing meningitis and should receive Meningitis vaccination together with rifampicin Antibiotics used for chemoprophylaxis are rifampicin, mnocycline, spiramycia, ciprofloxacin and ceftriaxone. 3/1/2014 8:17:49 PM Mark this question —& => Question Td : 206419 Question 25 of 30 ‘Thue statement regarding toxoplasmosis is which one of the following? @) Can present with fis in patients with AIDS 'b) Infection in the frst trimester of pregnancy is seldom harmfil to fetus 6) Infection usually by respiration 4) Prophylactic immunoglobulin should be given to pregnant women if their TgM anti-toxoplasma antibodies detected, 6) Raw eggs are an important source of infection Answer | Explanation Other User's Explanation Report An Error Question Explanatio ‘Transmission of Toxoplasma gondiiis after ingestion of cysts from contact with cat faeces or raw / undercooked meat. The definitive host is the cat. Oocysts excreted with ct faeces can semain in soil for morthe. The risk of fetopathy is seduced by mate than 50% if spiramycin, which can prevent maternal fetal transtaission, is given to mothers. 3/1/2014 8:17:49 PM Mark this question —& => Question Td : 206419 Question 25 of 30 ‘Thue statement regarding toxoplasmosis is which one of the following? ¥ © 2 Can present with fis in patients with AIDS +) Infection in the frst trimester of pregnancy is seldom harmful to fetus ©) Infection usualy by respiration 4) Prophylactic immunoglobulin chould be given to pregnant women f their IgM anti-tozoplasma antibodise detected 6) Raw eggs are an important source offinfection Answer | Explanation Other User's Explanation Report An Error Question Explanatio ‘Transmission of Toxoplasma gondiiis after ingestion of cysts from contact with cat faeces or raw / undercooked meat. The definitive host is the cat. Oocysts excreted with ct faeces can semain in soil for morthe. The risk of fetopathy is seduced by mate than 50% if spiramycin, which can prevent maternal fetal transtaission, is given to mothers. ‘Marke this question & => Question Td : 206604 Question 26 of 30 Most correct statement regarding the antibiotic combination quimupristin and dalfopristin is which one of the following? a) Adainistered oraly. +) Effective against muiti-resictant Staph, aureus ) Eifective against resistant mycobacterium TB. 6) Indicated in subjects with chronic renal inp airment ©) Particulanly effective in the treatment ofpseudomonas infection in cystic fibrosis Question Explanation: Quinupristin and dalfopristin are a synergistic combination of a streptogramin A and B respectively. They are effective against grem postive serobes and are particularly usefil against resistart Strep pneumoniae and Staph aureus They can be administered orly via a central lice. Report An Error ‘Marke this question & => Question Td : 206604 Question 26 of 30 Most correct statement regarding the antibiotic combination quimupristin and dalfopristin is which one of the following? a) Adainistered oraly. J © b) Effective against multi-resistant Staph. aureus ) Eifective against resistant mycobacterium TB. 6) Indicated in subjects with chronic renal inp airment ©) Particulanly effective in the treatment ofpseudomonas infection in cystic fibrosis Question Explanation: Quinupristin and dalfopristin are a synergistic combination of a streptogramin A and B respectively. They are effective against grem postive serobes and are particularly usefil against resistart Strep pneumoniae and Staph aureus They can be administered orly via a central lice. Report An Error ‘Merle this question <_c=> Question 27 of 30 Question Td : 206645 A.20 year old student presents with fever, malaise and sore throat. Exam reveals a temperature of 38.3°C with cervical lymphadenopathy. Her Hb is 12.9 g/dL, WEC count is 9.8 x10°/L with neutrophils 3 x10%L (1 5-7 x10?) and lymphocytes 4.5 x10°%L (1.5-4 210%), Blood film reveals atypical monoruclear cells. The most likely diagnosis is which one of the following? a) Acute lymphoblastic leukaemia t) Brucellosis c) Epstein-Barr viral (EBV) infection 6) Hodgkin's disease 6) Sarcoidosis Question Explanation: The diagnosis ic EBV infection, infectious mononucleosis, which may be confirmed by the presence of immineglobulin (Iz) Mto EBY. ‘Merle this question <_c=> Question 27 of 30 Question Td : 206645 A.20 year old student presents with fever, malaise and sore throat. Exam reveals a temperature of 38.3°C with cervical lymphadenopathy. Her Hb is 12.9 g/dL, WEC count is 9.8 x10°/L with neutrophils 3 x10%L (1 5-7 x10?) and lymphocytes 4.5 x10°%L (1.5-4 210%), Blood film reveals atypical monoruclear cells. The most likely diagnosis is which one of the following? a) Acute lymphoblastic leukaemia t) Brucellosis ¥ © ©) Epstein-Barr viral (EBV) infection 6) Hodgkin's disease 6) Sarcoidosis Question Explanation: The diagnosis ic EBV infection, infectious mononucleosis, which may be confirmed by the presence of immineglobulin (Iz) Mto EBY. Mark this question e& => Question Td : 206695 Question 28 of 30 A. 16 year old boy presents with fever and headache. He has received oral amoxicilin for three days. ‘Which one of the following cerebrospinal Aud (CSF) fading: would excinde partially treated meningitis? a) CSF ghicose of 45% of blood ghicose ) A negative CSF culture ©) A negative gram stain 4) A negative Ieernig’s cign ©) A white cell count of 50 Question Explanation: ‘The assessinent of children with suspected bactesial meningtis who have already received antibiotic therapy is a diagnostic corundrum, This applies to about 25-50% of children, s¢ it is an important problem, Panial treatment mnay reduce the incidence of postive CSE gram stain to than 60%, and it also reduces the ability to grow the bacteria, particularly meningococcus. CSF glucose, proteia, neutrophils and bacterial antigen testing of polymnerase chain reaction (PCR) should be completely unalfected, Report An Error Mark this question e& => Question Td : 206695 Question 28 of 30 A. 16 year old boy presents with fever and headache. He has received oral amoxicilin for three days. ‘Which one of the following cerebrospinal Aud (CSF) fading: would excinde partially treated meningitis? Y © a) CSF gincose of 45% of blood ghucose ) A negative CSF culture ©) A negative gram stain 4) A negative Ieernig’s cign ©) A white cell count of 50 Question Explanation: ‘The assessinent of children with suspected bactesial meningtis who have already received antibiotic therapy is a diagnostic corundrum, This applies to about 25-50% of children, s¢ it is an important problem, Panial treatment mnay reduce the incidence of postive CSE gram stain to than 60%, and it also reduces the ability to grow the bacteria, particularly meningococcus. CSF glucose, proteia, neutrophils and bacterial antigen testing of polymnerase chain reaction (PCR) should be completely unalfected, Report An Error 3/1/2014 8:18:43 PM [fea emed.cmran- Yahoo Wi *) 98 vww.nterface.edu.pkin x \ [STs]: € > C BG www.interface edu nk /medical-exams/test-analysis pho 2utd=14759 * 8 Ht apps El cocgle A seungs Cisonn 8... wibhe tule © go uediliane Gla ye Yoioil JS [) Freerotmal — » C3 other bookmar ‘Mark this question <-> Question Td : 216966] Total Questions . i. ose Question 29 of 30 oe |ASt-year-old wemnan after cctuning fiom India has scleral icterus and a palpable, milly tendor liver that extends 2em below the 2x costal margin, Relevant labs are 4. % [Other laboratories are shown: 5 x [WBC: 12.600/mm3 S. Bs hteutroptils 50% z, Oa IE ocinophals: la%4 2. BS [Monocytes Be e [Lrmphocyies fare cm ie i oe last (SGOT) soo ea [ALT SGPT) 224 UL i ix [Atkatine phosphatase a5 UL a ox [Biirabin 5-7 masa. em [Hepatitis panel 16 Xx [Anti HAV 1g6 [ifegasive aw x [Anti-HAY TENE Positive 1B Xx [Hepatiis B surface antigen |[Negarive 2 Xx [Hepatitis B surface antibody [Positive 200% [AnthepatitisE core antibody| Negative 3 e [Hepatiis C antibody screen [Negative a % Stool toxin for C. Difcile [Pending me ‘Wat isthe most appropriate managemens? = & a) Evahtation of close cortacts for hepatitis A immunoglobulin and vaccination ee ) Star the patiert cn Metronidazole to treat the Pseuslomembranous colt. o ) Hepatitis vaccination with combined A and B vaccine and immunoglobuln me ) No treatment or soreening, schedule a follow-up vist with liver function tests — = 2) Evahiate clove contacts after the patient acute infection has subsided a ee Question Explanatioy This patient's behavior places her at risk for a variety of infections inchiding hepatitis A, which she has, and hepatitis F, both of which are likely to lurch in the Ganges River and among street children. A sell list ofthe infections she placed herseif at risl for include typhoid fever, E. col, shigellosis, amebic dysentery, cholera, malaria, brucellosis (rom "raw" milk), tuberculosis, bubonic plague, and leptospirosis (also likely in the Ganges River, as the disease is carried in water contaminated with anircal urine). Only a small percentage of patients with Hepatitis A progresses to fulminant hepatitis. Close contacts, particularly family members, should be evaluated for treatment. Vaccination and hepatitis A inmuneglobulix given promptly can provent, or ifthe disease is already brewing, modulate, the severity of tae disease There is no need to start the patient on metronidazole to treat the Pseudomembrencus clits. This patient most probably does not hhave Pseudomembranous colitis, even though ker stool antigen rep ott is still pending. This patient clearly has hepatitis A, and treatment should not be instituted without a stool antigen report. Furthermore, doxycycline is not aszociated with Pseudomembrancus colitis. Drugs that can be associated are: clindamycin, cephalosporins and recently ciprofloxacin ‘As this patient already has immunity to hepatitis B and now has full- blown hepatitis A, litle beneSt will be gained fiom vaccine and imminoglobulin. Vaccination before travel would likely have prevented this disease “Abhough this patient will lkcely recover without treatment, screening is appropsiats, Humans are the orly ecervoirs of hepaitis A, end any chance to prevent spread of the disease can theoretically protect an entire community ‘Waiting until the patients acute infection has subsided is wrong, because close contacts should be evaluated and given immunoglobulin [esas ects aah Sacre ke 3/1/2014 8:18:43 PM [fea emed.cmran- Yahoo Wi *) 98 vww.nterface.edu.pkin x \ [STs]: € > C BG www.interface edu nk /medical-exams/test-analysis pho 2utd=14759 * 8 Ht apps El cocgle A seungs Cisonn 8... wibhe tule © go uediliane Gla ye Yoioil JS [) Freerotmal — » C3 other bookmar ‘Mark this question <-> Question Td : 216966] Total Questions | 1 x Question 29 of 30 = A S-year-old woman after returning fiom Tadia has scleral icterus and a palpable, milly tender liver that extends 2cm below the 30 Xx costal margin, Relevant labs are 4° % [Other laboratories are shown: 5 x [WBC: 12.600/mm3 S. Bs hteutroptils 50% z, Oa IE ocinophals: la%4 2. BS bMfonocytes B% e [Lymphocytes 9% cm ie i oe [ast SGOT) laso0 TZ. 2 x [ALT (SGPT) 42240 Bx [Alkaline phosphatase bas UL 4 x [Biirabin 6.7 mesa, ‘ie ae [Hepatitis panel 16 x [Anti HAY IgG btegative 2! ox [Anti-HAY Teh Positive ig x [Hepatiis B surface antigen |[Negarive 2 Xx [Hepatitis B surface antibody [Positive 200% [AnthepatitisE core antibody| Negative 3 e [Hepatiis C antibody screen [Negative = & [Stool toxin for C. Difficile: [Pending a Xx ‘What is the most appropriate management? a V © a) Evaluation of close contacts for hepatitis A immunoglobulin and vaccination ie ae +b) Start the patient on Metronidazole to treat the Pseudomembranous colitis ‘(x ©) Hepatitis vaccination with combined A and B vaccine and immunoglobulin se 4) No treatment or screening, schedule a follow-up visit with liver finction tests oe Se ¢) Evaluate close contacts after the patient acute infection has subsided. a oe Question Explanatioy This patient's behavior places her at risk for a variety of infections inchiding hepatitis A, which she has, and hepatitis F, both of which are likely to lurch in the Ganges River and among street children. A sell list ofthe infections she placed herseif at risl for include typhoid fever, E. col, shigellosis, amebic dysentery, cholera, malaria, brucellosis (rom "raw" milk), tuberculosis, bubonic plague, and leptospirosis (also likely in the Ganges River, as the disease is carried in water contaminated with anircal urine). Only a small percentage of patients with Hepatitis A progresses to fulminant hepatitis. Close contacts, particularly family members, should be evaluated for treatment. Vaccination and hepatitis A inmuneglobulix given promptly can provent, or ifthe disease is already brewing, modulate, the severity of tae disease There is no need to start the patient on metronidazole to treat the Pseudomembrencus clits. This patient most probably does not hhave Pseudomembranous colitis, even though ker stool antigen rep ott is still pending. This patient clearly has hepatitis A, and treatment should not be instituted without a stool antigen report. Furthermore, doxycycline is not aszociated with Pseudomembrancus colitis. Drugs that can be associated are: clindamycin, cephalosporins and recently ciprofloxacin ‘As this patient already has immunity to hepatitis B and now has full- blown hepatitis A, litle beneSt will be gained fiom vaccine and imminoglobulin. Vaccination before travel would likely have prevented this disease “Abhough this patient will lkcely recover without treatment, screening is appropsiats, Humans are the orly ecervoirs of hepaitis A, end any chance to prevent spread of the disease can theoretically protect an entire community ‘Waiting until the patients acute infection has subsided is wrong, because close contacts should be evaluated and given immunoglobulin [esas ects aah Sacre ke ‘Merk this question €& Question Td : 217130 Question 30 of 30 A 26-year-old medical student presents to the emergency department 30 minutes after 2 needle-stick injury. He was stuck with a nvedle from a patient with advanced hepatitis C and HIV. Waat is the MOST appropriate intervention at this ime? 2) HIV testing and prophylaxis three antiretroviral medications 6) Observation ©) Obtain blood test for HIV ) Start prophylaxis with AZT ©) Start HIV prophylaxis if HIV testis positive Question Explanation: Postexposure prophylaxis is effective in reducing the risk of HIV transtrission, Blood should be taken and three antireroviral drugs should be given immediately for ETV prophylaxis fora total of 4 weeks. A repeat HIV test should be dene 6 weeks, 12 weeks, and 6 months after exposure. Ths first response to a percutaneous cxposure should be to wash the arca thoroughly with soap and water of with an alcohol-based solution. For a significant exposure to a patient with advanced HIV the risk of seroconversion is about©.3%. Th such cases, observation would not be appropriate A blood test for HIV should be done but t should be followed by a prophylactic regimen. A single-agent prophylaxis with PIT is no longer recommended, IF the HIV test is positive, HIV treatment (not prophylaxis) should be started n Report An Error ‘Merk this question €& Question Td : 217130 Question 30 of 30 A 26-year-old medical student presents to the emergency department 30 minutes after a needle-stick injury, He wes stuck with 2 needle from a patient with advanced hepatitis C and HIV. What is the MOST appropricte intervention at this time? Y © a) HIV testing and prophylaxis three antiretroviral medications b) Observation ©) Obtain blood test for HIV ) Start prophylasis with AZT ¢) Start HIV prophylaxis if HIV testis positive Question Explanation: Postexposure prophylaxis is effective in reducing the risk of HIV transtrission, Blood should be taken and three antireroviral drugs should be given immediately for ETV prophylaxis fora total of 4 weeks. A repeat HIV test should be dene 6 weeks, 12 weeks, and 6 months after exposure. Ths first response to a percutaneous cxposure should be to wash the arca thoroughly with soap and water of with an alcohol-based solution. For a significant exposure to a patient with advanced HIV the risk of seroconversion is about©.3%. Th such cases, observation would not be appropriate A blood test for HIV should be done but t should be followed by a prophylactic regimen. A single-agent prophylaxis with PIT is no longer recommended, IF the HIV test is positive, HIV treatment (not prophylaxis) should be started n Report An Error Mark this question => Question 1 of 30 Question Td : 29865 A 16 year old boy presents to you with a strongly positive Mamour test. The correct starement regarding this immune reaction is? a) The response is mediated by B lymphocytes 8) The area of induration will be less than 10 mun in diamerer ©) Ikis a cell mediated immune response 4) The reaction typically develops within 24 hours 6) IFa skin biopsy were taken, immmohistachemistry would show immane complex deposition Question Explanatio ‘The tuberculin skin test (TST) is an intradesmnal tes: for cell-mediated hypersensitivity to tuberculprotein. A stronaly positive Mantowr is an area of induration of > 15mm (10 taberculin units). The test is negative in those who have never been exposed to infection with TB. Ibusually becomes positive 3-5 weeks alter infection. False-negative TST are seen in debilitated and immune-deficieat patients. Mark this question => Question 1 of 30 Question Td : 29865 A 16 year old boy presents to you with a strongly positive Mamour test. The correct starement regarding this immune reaction is? a) The response is mediated by B lymphocytes 8) The area of induration will be less than 10 mun in diamerer Y © 0) tis acell mediated immune response 4) The reaction typically develops within 24 hours 6) Fa skin biopsy were taken, immunohistochemistry would show immune complex depesition Question Explanatio ‘The tuberculin skin test (TST) is an intradesmnal tes: for cell-mediated hypersensitivity to tuberculprotein. A stronaly positive Mantowr is an area of induration of > 15mm (10 taberculin units). The test is negative in those who have never been exposed to infection with TB. Ibusually becomes positive 3-5 weeks alter infection. False-negative TST are seen in debilitated and immune-deficieat patients. ‘Marke this question e& => Question Td : 48671 Question 2 of 30 A.4S year old man presents with a history of pain and swelling in the right testicle for 2 days. He has no dysuria or urinary frequency, and denies any se:mal coxtact except with his wife. On examination you find tendemess in the right posterior aspect of the sight testicle, along with some swelling and erythema of the overlying scrotal skin, The true statement regarding this situation is 2) The most likely etiologic agentis Chlamydia trachornatis, +b) Immediate surgical referral is indicated ) Anaerobic bacteria are the infecting agents in most caces 4) The patient should be treated symptomaticaly untl results from a urethral culture are available 6) Antibiotic therapy chould include coverage for coliform bacteria Anewor [UEXVISNEAR) ter Usors Explanation Report An Error Question Explanation: Epididymiti is an inflammation of the epididymis due to various infectious agents or te local trauma Pain may develop over a day or ‘vo, of even more gradually. In men under the age of 35 who are sexually active, the most common etiologic organisms are Chlamydia trachomatis and Neisseia gonorrhoeae. In men over 35, infectious epididymitis is usually nonspecific and is caused by coliform bacteria or Pseudomonas species. The preferred treatment is ofloxacin, 400 mg orally twice a day for I days. Altemative therapy is a single dose of ceftriaxone, 250 mg, plus doxycycline, 100 mg twice a day for 1D days. ‘Marke this question e& => Question Td : 48671 Question 2 of 30 A.4S year old man presents with a history of pain and swelling in the right testicle for 2 days. He has no dysuria or urinary frequency, and denies any se:mal coxtact except with his wife. On examination you find tendemess in the right posterior aspect of the sight testicle, along with some swelling and erythema of the overlying scrotal skin, The true statement regarding this situation is 2) The most likely etiologic agentis Chlamydia trachornatis, +b) Immediate surgical referral is indicated ) Anaerobic bacteria are the infecting agents in most caces 4) The patient should be treated symptomaticaly untl results from a urethral culture are available YM © ©) Antibiotic therapy should include coverage for coliform bacteria Anewor [UEXVISNEAR) ter Usors Explanation Report An Error Question Explanation: Epididymiti is an inflammation of the epididymis due to various infectious agents or te local trauma Pain may develop over a day or ‘vo, of even more gradually. In men under the age of 35 who are sexually active, the most common etiologic organisms are Chlamydia trachomatis and Neisseia gonorrhoeae. In men over 35, infectious epididymitis is usually nonspecific and is caused by coliform bacteria or Pseudomonas species. The preferred treatment is ofloxacin, 400 mg orally twice a day for I days. Altemative therapy is a single dose of ceftriaxone, 250 mg, plus doxycycline, 100 mg twice a day for 1D days. ‘Mark this question & => Question Ta : 50760 Question 3 of 30 A.27 year old HIV postive fernale comes to the emergency department because of increasing headaches, right-sided weakness and disorientation for the pact week. A major motor (grand mal) seine occurs shortly after admission. On physical examination following the seizwe, vital sigs are normal. No muchal sigidity is present. Funduscopic examination shows papiledema There is also right hemiparesis and aphasia The most likely dhamnosis is 2) Meniagioma ) meningococcal meningitis o) Meurosyphilis © Toxoplasmosis, 2) Tuberculous meningitis Question Explanation: ‘The patient hes CN'S symptoms, and is EIV-positive. Some 15% of such petients have toxoplasmosis, making this the most likely diagnosis. The neurological symptoms of CNS toxoplasmosis will vary depending upon the ste and severity of the infection, A glicblastoma, neurosyphilis, meringococcal, or tibercular meningitis could cause some of all ofthe symptoms, but given the patient's history, are much less likely than toxoplasmosis, Obviously the neat step in management would be to confirm the diagnosis. ‘Mark this question & => Question Ta : 50760 Question 3 of 30 A.27 year old HIV postive fernale comes to the emergency department because of increasing headaches, right-sided weakness and disorientation for the pact week. A major motor (grand mal) seine occurs shortly after admission. On physical examination following the seizwe, vital sigs are normal. No muchal sigidity is present. Funduscopic examination shows papiledema There is also right hemiparesis and aphasia The most likely dhamnosis is 2) Meniagioma ) meningococcal meningitis o) Meurosyphilis Y © A Toxoplasmosis 2) Tuberculous meningitis Question Explanation: ‘The patient hes CN'S symptoms, and is EIV-positive. Some 15% of such petients have toxoplasmosis, making this the most likely diagnosis. The neurological symptoms of CNS toxoplasmosis will vary depending upon the ste and severity of the infection, A glicblastoma, neurosyphilis, meringococcal, or tibercular meningitis could cause some of all ofthe symptoms, but given the patient's history, are much less likely than toxoplasmosis, Obviously the neat step in management would be to confirm the diagnosis. Mark this question & => Question Td : 54509 Question 4 of 30 A.66 year old woman presents with a band like buming pain in the right upper quadrant extending from the epigastrium around to the tidiine of the back. No abdominal tendemess is present Findings cn ultrasonography of the gallbladder are normal, serum amylase concentration is normal. Whatis the most likely diagnosis? 2) Acalculous cholecystitis b) Chronic relapsing pancreatitis ©) Diverticulitis of the cecum 4) Penetrating duodenal ulcer ) Hespes zoster Question Explanation: Herpes zoster (Shingles) is characterized by a vesicular rash that is preceded by the bandlike burning pain described here. It usually involves only one or two dermatomes. Itis always a reactivationflare-up of a previous Varicella infection. In this patieat, there is no abdominal tendemess, which decreases the likelihood of an intra-abdominal organ being involved, Acalculous cholecystitis is a diagnosis made more commonly in the ICU. The patient is usually post-op, in shock, or oritcally il. Chronic relapsing pancreatitis is characterized by a boring or cramaping epigastric pain radiating to the back. Iris more commonly seen in alcoholics, who will have some abdominal tendemess, Amylase and lpase levels may support the diagnosis ifthey come back high. Report An Error Band like pain is very characteristic for herpes zoster. And the investigations here rule out any intra abdominal pathclogy. Yes wesicles and rash should appear but net always simultaneously with the pain. Mark this question & => Question Td : 54509 Question 4 of 30 A.66 year old woman presents with a band like buming pain in the right upper quadrant extending from the epigastrium around to the tidiine of the back. No abdominal tendemess is present Findings cn ultrasonography of the gallbladder are normal, serum amylase concentration is normal. Whatis the most likely diagnosis? 2) Acalculous cholecystitis b) Chronic relapsing pancreatitis ©) Diverticulitis of the cecum 4) Penetrating duodenal ulcer Y © ¢) Herpes zoster Question Explanation: Herpes zoster (Shingles) is characterized by a vesicular rash that is preceded by the bandlike burning pain described here. It usually involves only one or two dermatomes. Itis always a reactivationflare-up of a previous Varicella infection. In this patieat, there is no abdominal tendemess, which decreases the likelihood of an intra-abdominal organ being involved, Acalculous cholecystitis is a diagnosis made more commonly in the ICU. The patient is usually post-op, in shock, or oritcally il. Chronic relapsing pancreatitis is characterized by a boring or cramaping epigastric pain radiating to the back. Iris more commonly seen in alcoholics, who will have some abdominal tendemess, Amylase and lpase levels may support the diagnosis ifthey come back high. Report An Error Band like pain is very characteristic for herpes zoster. And the investigations here rule out any intra abdominal pathclogy. Yes wesicles and rash should appear but net always simultaneously with the pain. 3/1/2014 Mark this question & => Question 5 of 30 7 PM ‘The most common mode of spread of HIV among heterosemials in developing counties is 2) Prostitution ) Shared needles, ©) Reuse of needles in hospitals & Blood transfusions, ©) Needle sticks Question Explanation: Prostitution is responsible for the rapid spread of HIV among heteroseauals in developing nations. Question Td : 62253 3/1/2014 Mark this question & => Question 5 of 30 7 PM ‘The most common mode of spread of HIV among heterosemials in developing counties is Y © 2) Prostitution, ) Shared needles, ©) Reuse ofneedtes in hospitals & Blood transfusions, «) Needle sticks. Question Explanation: Prostitution is responsible for the rapid spread of HIV among heteroseauals in developing nations. Question Td : 62253 “Mark this question & => Question Ta : 65870 Question 6 of 30 A 19-year-old female presents with a white "cottage-cheese" type of discharge from the vagina This discharge is odorless and is associated with vaginal and vulvar pruritis. The physician saw spores and hyphee on the KOH wet preparation All of the following are common causes of this disorder EXCEPT a) Diabetes ») HIV infection ©) Hepatitis A infection @ Pregnancy 2) Amtibiotics Question Explanation: Hepatitis A infection is transmitted by fecal-oral route and it does not feciltate anghow the development of Candidiasis. Diabetes inelltus is a major risk factor for candidiasis. Cancida and other orgenisms may be found in small numbers in the normal vagina Overgrowti of candida with a "cheesy discherge" can occur in immunocompromised states such as HIV infection Pregnancy can predispose a woman to yeastinfection, Antibiotics can compromise immune function, thereby increasing the risk for infection, “Mark this question & => Question Ta : 65870 Question 6 of 30 A. 19-year-old fernale presents with a white "cottage-cheese" type of discharge from the vagina Thi discharge is odorless andis associated with vaginal and vulvar pruritis. The physician saw spores and hyphee on the KOH wet preparation All of the Following, ate common causes of this disorder EXCEPT 2) Diabetes ») HIV infection. Y © ©) Hepatitis A infection @ Pregnancy 2) Antibiotics Question Explanation: Hepatitis A infection is transmitted by fecal-oral route and it does not feciltate anghow the development of Candidiasis. Diabetes inelltus is a major risk factor for candidiasis. Cancida and other orgenisms may be found in small numbers in the normal vagina Overgrowti of candida with a "cheesy discherge" can occur in immunocompromised states such as HIV infection Pregnancy can predispose a woman to yeastinfection, Antibiotics can compromise immune function, thereby increasing the risk for infection, Mark this question eo Question 7 of 30 ‘A patient develops diarrhea and muscle pain after eating a lot of meat at a barbeque. His symptoms have been present for two weeks now. His intial complete blood count shows eosinophilia. The best investigation to reach the diagnosis is 2) Stool culture b) Muscle biopsy 6) Stool gram stain 4) Stool ova end parasites Question Explanation: Trichinosis is infection with Trishinella spiral is or related Trichinella species. Symptoms include iritial GT invitation followed by periorbital edema, muscle pain, fever, and eosinophilia Humans become infected by eating raw, undercooked, or processed meat from infected animals, most commonly pigs, wild boar, or bear. Diagnosis is clnical and with serologic tests. Muscle biopsy may be diagnostic but is seldom necessary. Treatment is with mebendazole or albendazole phus prechisone symptoms are severe. No sperific tests to chagnose the intestinal stage are avaiable “After the 2nd week of infection, a muscle biopsy may disclose larvae and cysts but is seldom necessary. Diffie inflammation in muscle tissue indicates recent infection Mark this question eo Question 7 of 30 ‘A patient develops diarrhea and muscle pain after eating a lot of meat at a barbeque. His symptoms have been present for two weeks now. His intial complete blood count shows eosinophilia. The best investigation to reach the diagnosis is 2) Stool culture Y © ») Muscle biopsy 6) Stool gram stain 4) Stool ova end parasites Question Explanation: Trichinosis is infection with Trishinella spiral is or related Trichinella species. Symptoms include iritial GT invitation followed by periorbital edema, muscle pain, fever, and eosinophilia Humans become infected by eating raw, undercooked, or processed meat from infected animals, most commonly pigs, wild boar, or bear. Diagnosis is clnical and with serologic tests. Muscle biopsy may be diagnostic but is seldom necessary. Treatment is with mebendazole or albendazole phus prechisone symptoms are severe. No sperific tests to chagnose the intestinal stage are avaiable “After the 2nd week of infection, a muscle biopsy may disclose larvae and cysts but is seldom necessary. Diffie inflammation in muscle tissue indicates recent infection Mark this question => Question Id : 74542 Question 8 of 30 Of the following choices, the most common means of transmission of hepatitis A among humans is 2) Blood transfusions +) Factor VII adkninistration ) Nasopharyngeal droplets 4) Pesi-natal «) Shared needles Question Explanation: Hepatitis A is spread via upper respiratory secretions, Ibalso is spread via contact with siocl. None of the other activites are respousible for the spread of hepatitis A as much as. n Report An Error Mark this question => Question Id : 74542 Question 8 of 30 Of the following choices, the most common means of transmission of hepatitis A among humans is 2) Blood transfusions +) Factor VII adkninistration Y © © Nasopharyngeal droplets. 4) Pesi-natal «) Shared needles Question Explanation: Hepatitis A is spread via upper respiratory secretions, Ibalso is spread via contact with siocl. None of the other activites are respousible for the spread of hepatitis A as much as. n Report An Error Mark this question & => Question Td : 81086 Question 9 of 30 A.25 year cld female with a history of domestic abuse presents with a characteristic "slapped face" appearance. The most appropriate diagnosis is 2) Measles +) Erythema infectiosum ©) Rocky Mountain spotted fever 4) Kawesaki’s disease €) Menirgocoscemia Question Explanation: Patients with erythema infectiosum extibit an appearance of a "slapped face." Report An Error Mark this question & => Question Td : 81086 Question 9 of 30 A.25 year cld female with a history of domestic abuse presents with a characteristic "slapped face" appearance. The most appropriate diagnosis is 2) Measles Y © b) Erythema infectiosum ©) Rocky Mountain spotted fever 4) Kawesaki’s disease €) Menirgocoscemia Question Explanation: Patients with erythema infectiosum extibit an appearance of a "slapped face." Report An Error ‘Mark this question —& => Question Td : 81371 Question 10 of 30 A 25 year old man presents with a purulent urethral discharge. An infectious etiology is suspected. The antibiotic treatment of choice is 2) Penicillin ) Ceftriaxone ©) Doxyeycline ) Vaacomyen 6) Ceftriaxone and doxycycline Question Explanatio ‘The most common etiology of urethritis in males is Neisseria gonottheae. The treatment of choice is a single intramuscular shot of ceftriaxone, Because of co-infection of Chlamydia trachomats (rom 25-40%), doxycyline is also indicated, Penicilln (A) is not used because of resistance via plasmid-mediated penicilinase Neisseria gonorrheae (PENG). Vancomycin (D) is not effective. ‘Mark this question —& => Question Td : 81371 Question 10 of 30 A.26 year old man presents with a purulent urethral discharge. .An infectious etiology is suspected, The antibiotic treatment of choice is 2) Pexicilin ) Ceftriaxone 6) Doxycycline 6) Vancomyen SV © ©) Ceftriaxone and dexycyoline Question Explanation: ‘The most common etiology of urethritis in males is Neisseria gonottheae. The treatment of choice is a single intramuscular shot of ceftriaxone, Because of co-infection of Chlamydia trachomats (rom 25-40%), doxycyline is also indicated, Penicilln (A) is not used because of resistance via plasmid-mediated penicilinase Neisseria gonorrheae (PENG). Vancomycin (D) is not effective. “Mark this question e& => Question Td : 93139 Question LI of 30 41 year old man received one week of amoxicilin and two weeks of clindamycin prior to oral surgery. On the 2nd postoperative day he presents with severe abdominal pain and copious diarrhea. What should be the antimicrobial of choice? a) Oral metronidazole ) Oral penicillin ©) Oral clindamycin. 4) IV metronidazole ) IV vancomycin. Answer | Boplanation Other User's Explanation Report An Error Question Explanation: ‘The treaiment of C. diicle is oral metroridazole or oral vancomycin, IV antibiotics [(D) and (E)] are not useful, since they are not absorbed by the gut. Clindamycin (C) causes C. difcile, but is not usefilin treatment, Oral penicillin is necessary to prevent dehydration. “Mark this question e& => Question Td : 93139 Question LI of 30 A.41 year oldman received one week of amoxicillin and two weeks of clindamycin prior to oral surgery. On the 2nd postoperative day he presents with severe abdominal pain and copious diarrhea. What should be the antimicrobial of choice? Y © a) Oral metronidazole. b) Oral penicillin, ¢) Oral clindamycin, d) IV metronidazole, ¢) IV vancomycin. Answer | Boplanation Other User's Explanation Report An Error Question Explanation: ‘The treaiment of C. diicle is oral metroridazole or oral vancomycin, IV antibiotics [(D) and (E)] are not useful, since they are not absorbed by the gut. Clindamycin (C) causes C. difcile, but is not usefilin treatment, Oral penicillin is necessary to prevent dehydration. Mark this question & => Question Td : 94312 Question 12 of 30 One of the following associated with streptococcal pharynaitis is a) Cervical iyisphadenopathy 6) Cough ©) Afebrile patient &) Group B strep. Question Explanatios Streptococci are gram-positive aerobic organisms that cause many disorders, including pharyngitis. pneumonia, wound and skin irfections, sepsis, and endocarditis. Symptoms vary with the organ infected. Sequelae include rheumatic fever and glomerulonephntis Clinical diagnoses are confirmed by Gram stain and culture. Strep throat (Streptococcal pharyngitis) is a form of Group A streptococcal infection that affects the pharye. Symptoms include: severe sore throat, yellow and white patches in the throat, dficulty swallowing, tender cervical lymphadenopathy, red and enlarged tonsils, halitosis, fever of 38°C: (101F) or greater, rash and absence of cough Mark this question & => Question Td : 94312 Question 12 of 30 One of the following associated with streptococcal pharyngitis is Y © a) Cervical lymphadenopathy 6) Cough ©) Afebrile patient &) Group B strep. Question Explanatios Streptococci are gram-positive aerobic organisms that cause many disorders, including pharyngitis. pneumonia, wound and skin irfections, sepsis, and endocarditis. Symptoms vary with the organ infected. Sequelae include rheumatic fever and glomerulonephntis Clinical diagnoses are confirmed by Gram stain and culture. Strep throat (Streptococcal pharyngitis) is a form of Group A streptococcal infection that affects the pharye. Symptoms include: severe sore throat, yellow and white patches in the throat, dficulty swallowing, tender cervical lymphadenopathy, red and enlarged tonsils, halitosis, fever of 38°C: (101F) or greater, rash and absence of cough Marke this question & => Question Td: Question 13 of 30 A. patient is given an injection af ceftriaxone far a diagnosis of gonorrhea, Addltional medication that the patient should receive is a) Tetracycline, because gonorrhea requires two-antibiotic coverage ') Doxycycline to trect chlamydia infection ¢) Ibuprofen to reduce inflammation 4) Erythromycin, because ceftriaxone does not offer broad enough coverage «) 1,000,000 unts of Vitamin C because it works synergistically with cefiriaxone Question Explanation: ‘While tetracycline, doxycyline, and erythromycin are all possibly correct antibiotics, the reason way additonal antibiotic coverage is given is because of the high association of chlamydia infection with gonorrhea. Ceftriaxone has broad spectrum coverage. and gonorthea does not require two antbiotics. Ibaprofen does not play a role in the treatment of this infection and despite numerous tales of the power of high dose vitamin C, it has never been shown lo work syaergistically with any antibiotics Marke this question & => Question Td: Question 13 of 30 A. patient is given an injection af ceftriaxone far a diagnosis of gonorrhea, Addltional medication that the patient should receive is a) Tetracycline, because gonorrhea requires two-antibiotic coverage J © ») Doxyeycline to treat chlamydia infection ¢) Ibuprofen to reduce inflammation 4) Erythromycin, because ceftriaxone does not offer broad enough coverage «) 1,000,000 unts of Vitamin C because it works synergistically with cefiriaxone Question Explanation: ‘While tetracycline, doxycyline, and erythromycin are all possibly correct antibiotics, the reason way additonal antibiotic coverage is given is because of the high association of chlamydia infection with gonorrhea. Ceftriaxone has broad spectrum coverage. and gonorthea does not require two antbiotics. Ibaprofen does not play a role in the treatment of this infection and despite numerous tales of the power of high dose vitamin C, it has never been shown lo work syaergistically with any antibiotics “Mark this question & => Question Id : 97178 Question 14 of 30 ‘The infection that was once considered neatly eradicated in a developed country is now recurring at an alarmingly increasing rave is which one of the following? 2) Meningococcal meringitis b) Pneumococcal pneumonia ©) Syphilis 46) Tuberculosis 8) Varicella Question Explanation: Public health measures and new antibiotics had nearly eliminated tuberculosis in the late 1970s, but a combination of failure to Glagnose subsequent cases. inadequate treatment, and the development of resistant strains of the bacteria have caused a rapid. increase in is occurrence A, B, and C'never came close to eradication. E can be prevented significantly by immunization of all school age children. “Mark this question & => Question Id : 97178 Question 14 of 30 ‘The infection that was once considered nearly eradicated in a developed country is now recurring at an clarmingly increasing rate is which one of the following? 2) Meningococcal meringitis +b) Pneumococcal pneumonia 6) Syphilis Y © 4) Tuberculosis 8) Varicella Question Explanation: Public health measures and new antibiotics had nearly eliminated tuberculosis in the late 1970s, but a combination of failure to Glagnose subsequent cases. inadequate treatment, and the development of resistant strains of the bacteria have caused a rapid. increase in is occurrence A, B, and C'never came close to eradication. E can be prevented significantly by immunization of all school age children. Mark this question ¢<=> Question 15 of 30 All ofthe following are manifestations of papillomavirus infection, EXCEPT 2) Warts ) Cervical cancer ©) Vulvar lesions 6) Antibody formation ©) Chancroid Answer [FEIRIRIRHBN) Other Users Explanation Repost An Esor Question Explanation: ‘Warts, cervical cancer, vulvar lesions, and antibody formation to the infecting virus are all manifestations of papillomavims infection. Chancroid is caused by Haemophilus ducreyi Mark this question ¢<=> Question 15 of 30 All ofthe following are manifestations of papillomavirus infection, EXCEPT 6) Warts ) Cervical cancer 6) Vulvar lesions 6) Antibody formation YM © &) Chancroid Answer (REIRIRIRHBN) Other Users Explanation Repost An Esor Question Explanation: ‘Warts, cervical cancer, vulvar lesions, and antibody formation to the infecting virus are all manifestations of papillomavims infection. Chancroid is caused by Haemophilus ducreyi ‘Marke this question ¢=> Question 16 of 30 A 20 year old boy presents with productive sputum, fever, and chest pain, He has been ill all his life with frequent bouts of bronchitis and pneumonia. Symptoms of malabsorption and occasional hemoptysis are also present. Exemination showed clubbing and ronchi at the hing bases. The organism responsible for his pulmonary symptoms is a) Klebsiella ) Haemophilus influenzae o TWAR 6) Pseadotnonas aeruginosa ©) Pneumocystis carinii Question Explanation: This patient has cystic flbrosis, which is an autosomal recessive disorder and can be fatal. The sweat test makes the diagnosis in which 100 mg of sweat is collected from the patient and ifthe sodinm content is greater than 70 meqj/ in adults, cystic fibrosis is Report An Error likely. Pancreatic exocrine dysfimction causes malabsorption and subsequent steatorrhea and pulmonary symptoms are common. Pseudomonas aeruginosa is one of the most common organisms colonizng the respiretory tree in these individuals. Klebsiella is a Gram-negative organism which is common in alcoholics. Haemophilus influenzae is a common cause of pneumonia in smokers and asthmatics. TWAR is a chlamydial organism that can cause pneumonia, Pneumocystis carinii is a protozca that causes bilateral interstitial infltrates in immunocompromised patients ‘Marke this question ¢=> Question 16 of 30 A 20 year old boy presents with productive sputum, fever, and chest pain, He has been ill all his life with frequent bouts of bronchitis aad pneumonia, Symptorns of malabsomption and occasional hemaptysie ate alsa present. Examination chowed clubbing and ronchi at the ing bases. The organism responsible for is pulmonary symptoms is a) Klebsiella ) Haemophilus influenzae o TWAR o © @ Pseudomonas aeruginosa e) Pheumocystis carinii Question Explanation: This patient has cystic flbrosis, which is an autosomal recessive disorder and can be fatal. The sweat test makes the diagnosis in which 100 mg of sweat is collected from the patient and ifthe sodinm content is greater than 70 meqj/ in adults, cystic fibrosis is Report An Error likely. Pancreatic exocrine dysfimction causes malabsorption and subsequent steatorrhea and pulmonary symptoms are common. Pseudomonas aeruginosa is one of the most common organisms colonizng the respiretory tree in these individuals. Klebsiella is a Gram-negative organism which is common in alcoholics. Haemophilus influenzae is a common cause of pneumonia in smokers and asthmatics. TWAR is a chlamydial organism that can cause pneumonia, Pneumocystis carinii is a protozca that causes bilateral interstitial infltrates in immunocompromised patients Marl this question = => Question Td : 107535 | Question 17 of 30 A.34 year old HIIV positive woman presents with low grade fever. severe perianal nain, and previous history of herpetic vulvovaginal lecicns Which of the followingis NOT aliely cause of anal lesionc in AIDS? a) Herpes simplex virus +) Cytomegalovirus ©) Anorectal squamous carcinoma and lymphoma &) Scabies. ©) Intractable anogenital warts secondary to human papillomavirus (Question Explanation: Sarcoptes scabiei is a mite which causes scabies and presents as pruritis, commonly worse at night. Burrows are present typically in the web spaces of the fingers. It would be rare for an HIV individual to have scabies in the perirectal area. Anal herpetic ulcers aifect both women and men and are associated with anal receptive intercourse. Cytomegalovirus is found in human cervical secretions, semen, saliva, feces, and blood for months after exposure. It can be transmitted semsally; consequently, anal intercourse can result in perirectal CMV lesions. Anorectal squamous carcinoma and lymphoma are increased in HIV individuals. They should produce induration and distortion. Human papillomavirus causes anogenital warts and tends to be more exophytic and confluent. Idicpathic ulceration of the anus has also been reported in HIV infected individuals. n Report An Error Merle this question Question Td : 10° ¢<-=> Question 17 of 30 A 34 year old HIV positive wornan presents with low grade fever. severe perianal pain, and previous history of herpetic mulvovaginal lecicns Which of the followingis NOT aliely cause of anal lesionc in AIDS? a) Herpes simplex virus +) Cytomegalovirus ©) Anorectal squamous carcinoma and lymphoma YM © d Scebies. 8) Intractable anogenital warts secondary to human papillomarirus, Answer | Bolanation Other User's Explanation Report An Error (Question Explanation: Sarcoptes scabiei is a mite which causes scabies and presents as pruritis, commonly worse at night. Burrows are present typically in the web spaces of the fingers. It would be rare for an HIV individual to have scabies in the perirectal area. Anal herpetic ulcers aifect both women and men and are associated with anal receptive intercourse. Cytomegalovirus is found in human cervical secretions, semen, saliva, feces, and blood for months after exposure. It can be transmitted semsally; consequently, anal intercourse can result in perirectal CMV lesions. Anorectal squamous carcinoma and lymphoma are increased in HIV individuals. They should produce induration and distortion. Human papillomavirus causes anogenital warts and tends to be more exophytic and confluent. Idicpathic ulceration of the anus has also been reported in HIV infected individuals. Marke this question & => Question Td : 141564 Question 18 of 30, A 36 year old newly married male presents with rmaltiple blister lke lesions on the glans penis appearing over the past years Examination shows tender, 3 to 4 mm vesicular lesions on the shaft of his penis with no apparent crusting, drainage, or bleeding, Slight bilateral inguinal adenopathy is also present. The most ley infectious agent is best described by which one of the following? a) The agent becomes latent in trigeminal garglia b) Diagnosis could be confitmed by a Tzanck smear ©) The infection is cured with acyclovir ) Dis a common cause of encephaltis ¢) vesicular lesions are caused by exfoliatine Question Explanation: This is a classic example of an infection with herpes simplex vinas (probably type 2). This agent causes Iytic infections in mucoepithelal cells. It then moves by retrograde transport into neurons of the sacral ganglia, where it ies dormant curing the asyinptomatic phase of the disease. The Tzanck smear is a diagnostic test involving the touching of microscope slide to a vesicular lesion, Maltinucleated giant cells will be lifted off onto the slide and can be stained and identified as having a herpesvirus origin. Since the present lesions are on the genitalia, the vims would move into latency in the sacral ganglia, not the trigeminal ganglia. HSV introduced into the oral mucosa becomes latent in the trigeminal gangia. Acyclovir is not a cure for herpes viruses. Once the viral DNA has entered the nucleus of a cell there is no way to remove it, so the infection is lifelong. Herpes simplex 2 is rot a common cause of encephalitis, When ceatral nervous system infections are caused by this agent the result is usvally low level meningitis rather than encephalitis. HSV-1 is the cause of dangerous encephaltis. The lesions of bullous impetigo and scalded skin syndrome associated with Staphylococcus aureus are vesicular lesions caused by exfblatins Marke this question & => Question Td : 141564 Question 18 of 30, A 36 year old newly married male presents with rmaltiple blister lke lesions on the glans penis appearing over the past years Examination shows tender, 3 to 4 mam vesicular lesions on the shaft of his penis with no epparent crusting, ccainage, or bleeding. Slight bilateral inguinal adenopathy is also present. The most likely infectious agent is best described by which one of the following? a) The agent becomes latent in trigeminal ganglia ¥ © b) Diagnosis could be confirmed by a Tzanck smear ©) The infection is cured with acyclovir 4) This a common cause of encephaltis ¢) vesicular lesions are canced by exfaliatine Question Explanation: This is a classic example of an infection with herpes simplex vinas (probably type 2). This agent causes Iytic infections in mucoepithelal cells. It then moves by retrograde transport into neurons of the sacral ganglia, where it ies dormant curing the asyinptomatic phase of the disease. The Tzanck smear is a diagnostic test involving the touching of microscope slide to a vesicular lesion, Maltinucleated giant cells will be lifted off onto the slide and can be stained and identified as having a herpesvirus origin. Since the present lesions are on the genitalia, the vims would move into latency in the sacral ganglia, not the trigeminal ganglia. HSV introduced into the oral mucosa becomes latent in the trigeminal gangia. Acyclovir is not a cure for herpes viruses. Once the viral DNA has entered the nucleus of a cell there is no way to remove it, so the infection is lifelong. Herpes simplex 2 is rot a common cause of encephalitis, When ceatral nervous system infections are caused by this agent the result is usvally low level meningitis rather than encephalitis. HSV-1 is the cause of dangerous encephaltis. The lesions of bullous impetigo and scalded skin syndrome associated with Staphylococcus aureus are vesicular lesions caused by exfblatins Bemadlamvan- Yahoo M x) 9% www.nterfaceedu.pkyi x | € Di www. interface. edu pk /medicel-exarns/test-analysis pho ®ubd=14759 apps J coagle A seungs Pisgnin 2. wbbe dul © yo ueulléaye Ol ..tio ye gosoil JS [)) Free Hotmall ‘Marte this question = => Question Td : 141722 Question 19 of 30 A47 year old waiiress presents with maleise, loss of appetite, nausea, moderate fever, and jaundice, Serum ALT is 3200 U/L. Serology for hepatitis viruses indicates positive results for the presence of HBsAg, HBc IgM antibody and HCV antibody. Antibody tests for HBsAb and HAV are negative. Which one ofthe followingiis indicated by the resuits? a) Acute HBV infection ) Chronic hepatitis B infection ©) A duallinfection of EBV and HAV ) Hepatitis C infection ©) Past hepatitis A infection Avowor [ERT omer usors Expt Question Explanation: The presence of hepatitis B surface antigen HBsAg) alorg with hepatitis B core [gM antbedy (HBc IgM Ab), and the absence of hepatitis B surface antibody (HBSAb) incicate the early stages of an acute infection with hepatitis B (HBV). The presence of antibody to hepattis C (HCV) only indicetes exposure, but not a specific tine of exposure; however, 85% of patients who are infected with HCV develop chronic Infections. indicating that this patient has an 85% chence of having a dual infection with HEW and HCV. The acate or chronic HCV infection can be confirmed by POP. Serology of acute hepaitis can be shown graphically as follows jon Report An Error jandce Sympiams oy 3 3 3 224 Months after exposure owe rece Wes Chronic HBV infection is unlely because the patient has HBc IeMAb which is characterise ofan acute infection eather than a chronic infection. A dual infection of HBV and HAV is not plausible since the IgM ani HAV serology is negative. Hepatitis C infection is not confirmed by these data because the presence of HCV Ab indicates only exposure to the virus and act the state of infection. This could be caused by exposure at some earlier time (the elevated serum transaminases might be due to HBV infection. ‘An active or chronic HCV infection can be confirmed only by PCR. Pasthepatits A infection would be manifested by the presence ofloC anthodies againet TAY I le fo bes be k BRBERERBEBEBREE G other bookmar Total Questions xxxXxXXKXKXKXKXKXKXKKXKXKKKKKKKKKKKKKKKK Bemadlamvan- Yahoo M x) 9% www.nterfaceedu.pkyi x | € Di www. interface. edu pk /medicel-exarns/test-analysis pho ®ubd=14759 apps J coagle A seungs Pisgnin 2. wbbe dul © yo ueulléaye Ol ..tio ye gosoil JS [)) Free Hotmall ‘Marte this question = => Question Td : 141722 Question 19 of 30 A.47 year old waitress presents with malaise, loss of appetite, nausea, moderate fever, and jaundice, Serum ALT is 3200 U/L. Serology for hepatitis viruses indicates positive resulls for the presence of HBsAg, HBc IgM antibody and HCV antibody. Antibody tests for HBsAb and HAV are negative. Which one ofthe followingis indicated by the resuits? Y © a) Acute HEV infection ) Chronic hepatitis B infection ©) A dual infection of EBV and HAW ) Hepatitis C infection c) Past hepatitis A infection Avowor [ERT omer usors Expt Question Explanation: The presence of hepatitis B surface antigen HBsAg) alorg with hepatitis B core [gM antbedy (HBc IgM Ab), and the absence of hepatitis B surface antibody (HBSAb) incicate the early stages of an acute infection with hepatitis B (HBV). The presence of antibody to hepattis C (HCV) only indicetes exposure, but not a specific tine of exposure; however, 85% of patients who are infected with HCV develop chronic Infections. indicating that this patient has an 85% chence of having a dual infection with HEW and HCV. The acate or chronic HCV infection can be confirmed by POP. Serology of acute hepaitis can be shown graphically as follows jon Report An Error jandce Sympiams oy 3 3 3 224 Months after exposure owe rece Wes Chronic HBV infection is unlely because the patient has HBc IeMAb which is characterise ofan acute infection eather than a chronic infection. A dual infection of HBV and HAV is not plausible since the IgM ani HAV serology is negative. Hepatitis C infection is not confirmed by these data because the presence of HCV Ab indicates only exposure to the virus and act the state of infection. This could be caused by exposure at some earlier time (the elevated serum transaminases might be due to HBV infection. ‘An active or chronic HCV infection can be confirmed only by PCR. Pasthepatits A infection would be manifested by the presence ofloC anthodies againet TAY I le fo bes be k BRBERERBEBEBREE G other bookmar Total Questions xxxXxXXKXKXKXKXKXKXKKXKXKKKKKKKKKKKKKKKK Mark this question —& => Question Td : 141732 Question 20 of 30 An Asian immigrant develops malaise, fever, and rigors, followed by right upper quadrant abdominal pain, vorriting, jaundice, and itching His urine is dark and bis feces are pale. The parasite infection is strongly suggested by this patient's condition is, a) Clonorchis sinensis ) Enterobins vermiculais ©) Plasmodiurn 4) Taenia soliam ¢) Trypanosoma cruzi Question Explanation: ‘The patients suffering from suppurative (bacterial) cholangitis, which can occur as a complication of infection by the roundwormn, “Ascaris Inmbricoides and by the liver Bukes, Clonorchis sinensis and Fasciola hepatica. Biliary tract obstruction produces jaundice and extreme itching, with dark urine and pale feces. Therapy typically includes emergency endoscopic sphincterectomy to improve tiliery drainage, antibiotics, and antheiminthic agents. Enterobius vermicularis causes pinworm infections. Plasmodium ovale causes malaria, Taenia solium is the pork tapeworm. Adu tapeworms cause taeriasis, while the larvae are responsible for cysticercosis. ‘Trypanosoma cruzi causes Chagas disease. Mark this question —& => Question Td : 141732 Question 20 of 30 An Asian immigrant develops malaise, fever. and rigors. followed by right upper quadrent abdominal pain, vomiting, jaundice, and itching His urine is dark and his feces are pale. The parasite infection is strongly suggested by this patient's condition is Y © 2) Clonorchis sinensis +b) Enterobius vermicularis ©) Plasmodiura d) Tacnia solium ¢) Trypanosoma cruzi Question Explanation: ‘The patients suffering from suppurative (bacterial) cholangitis, which can occur as a complication of infection by the roundwormn, “Ascaris Inmbricoides and by the liver Bukes, Clonorchis sinensis and Fasciola hepatica. Biliary tract obstruction produces jaundice and extreme itching, with dark urine and pale feces. Therapy typically includes emergency endoscopic sphincterectomy to improve tiliery drainage, antibiotics, and antheiminthic agents. Enterobius vermicularis causes pinworm infections. Plasmodium ovale causes malaria, Taenia solium is the pork tapeworm. Adu tapeworms cause taeriasis, while the larvae are responsible for cysticercosis. ‘Trypanosoma cruzi causes Chagas disease. EZ € 2014 J % wow interface.ecu pki CL www. interface.edu.pk/medi Putid=14759 apps Eljcoogle Asetings [)Sgnin Zou wos dul xams/t oo wsdl dine Ofte ge sgsoll JS) Free Hatmall * 8 Gi other bookmar ‘Matic this question = => Question Id : 141742 Question 21 of 30 A composite wins is created in a laboratory. It contains the genome oftmolluscum contagosum virus and the capsid of adenovirus The additional reagent to be provided so that viral progeny would be created in appropriate host cells is a) DNA dependent DIVA polymerase b) DIA dependent RITA polymerase ©) BNA dependent DNA polymerase 4) RNA dependent RNA polymerase €) No additional reagent is required Anewor [RERRNGIN othr usors Explanation Report An Error Question Explanation: Composite viruses are viruses that are created in laboratories by artificially combining components of different parental viruses. Th this case, the genetic material is DIVA, because molluscum contagiosum vinus is a double siranded DNA virus in the poxvirus fal. The capsid of this composite virus is from adenovirus, so the tropism of the composite virus would be for adenovinus target cells Poxwinases are nique among the DIA viruses in that they replicate their DNVA in the cytoplasm, Since there are no cellular enzymes for DNVA metabolism in the cytoplasm, the virus has to enter the cell caring several enzymes. Of the erzymes on the lst, the poxviruses have to have a DNA dependent RIVA polymerase inside each capsid so that transcription of messenger RIVA from the genomic DNA can proceed in the cytoplasm (a locetion where these enzymes are not available in the cell). Other things thet are carried in the capsid inchade a transcription activator, a multi subunit RNA polymerase, and enrymes for polyadenylate addition and capping of mRNA, The carly proteins produced by the virus include an uacoatase, which then exposes the DIVA for replication in intracytoplasmic factories, called Guamnieti bocies. A DNA dependent DNA polymerase is not carried in the capsid of any virus of medical importance. DNA viruses simply travel to the mucleus, where they use cellular enzymes to replicate their genomes. There are only two DNA vimses that use their ow virion encoded enzymes: poxwises, which need to create DNA in the cytoplasm of the cell, and hepadnaviruses, which replicate through an RNA intermediate and thus sequire an enzyme with reverse trenscriptese activity. An RIVA dependent DNA polymerase, or reverse transcriptase, is carried by retroviruses inside their capsids. The hepadnavinuses syathesize their own reverse transcriptese erzyme inside the cell (tis called DNA polymerase because itis used to replicate their DIA) but do not carry it with them in their capsids. An RNA dependent RMA polymerase is required by all the negative sense RNA viruses (they are not infsclious without it). Other RNA viruses will synthesize their own RMA dependent RMVA polymerase once they are in the cell, but do not require é for stasting the infection, None of the DNA viruses carry such a polymerase with them into the cell, No additional reagent is required would be true to start an infection with the majority of the DIVA viruses (exchiding poxviruses) and the positive sense RNA vimises (excluding the retroviruses). These viruses are directly infectious, i the naked genomic nucleic acidis directly injected into a cell Total Questions BEBE EK ERB BEGY eRe eRe He SRBBARBEBS XxXxXXKXKXKXKXKXKXKXKXKKXKKXKKKKKKKKKKKKKK EZ € 2014 J % wow interface.ecu pki CL www. interface.edu.pk/medi Putid=14759 apps Eljcoogle Asetings [)Sgnin Zou wos dul xams/t oo wsdl dine Ofte ge sgsoll JS) Free Hatmall * 8 Gi other bookmar ‘Matic this question = => Question Id : 141742 Question 21 of 30 A. composite vins is created in a laboratory. It contains the genome of molluscum contagiosum virus and the capsid of adenovirus, The additional reagent to be provided so that viral progeny would be created in appropriate host cells is a) DNA dependent DIVA polymerase ¥ © b)DNA dependent RIVA polymerase c) RNA dependent DNA polymerase 4) RNA dependent RNA polymerase ) No additional reagent is required Anewor [RERRNGIN othr usors Explanation Report An Error Question Explanation: Composite viruses are viruses that are created in laboratories by artificially combining components of different parental viruses. Th this case, the genetic material is DIVA, because molluscum contagiosum vinus is a double siranded DNA virus in the poxvirus fal. The capsid of this composite virus is from adenovirus, so the tropism of the composite virus would be for adenovinus target cells Poxwinases are nique among the DIA viruses in that they replicate their DNVA in the cytoplasm, Since there are no cellular enzymes for DNVA metabolism in the cytoplasm, the virus has to enter the cell caring several enzymes. Of the erzymes on the lst, the poxviruses have to have a DNA dependent RIVA polymerase inside each capsid so that transcription of messenger RIVA from the genomic DNA can proceed in the cytoplasm (a locetion where these enzymes are not available in the cell). Other things thet are carried in the capsid inchade a transcription activator, a multi subunit RNA polymerase, and enrymes for polyadenylate addition and capping of mRNA, The carly proteins produced by the virus include an uacoatase, which then exposes the DIVA for replication in intracytoplasmic factories, called Guamnieti bocies. A DNA dependent DNA polymerase is not carried in the capsid of any virus of medical importance. DNA viruses simply travel to the mucleus, where they use cellular enzymes to replicate their genomes. There are only two DNA vimses that use their ow virion encoded enzymes: poxwises, which need to create DNA in the cytoplasm of the cell, and hepadnaviruses, which replicate through an RNA intermediate and thus sequire an enzyme with reverse trenscriptese activity. An RIVA dependent DNA polymerase, or reverse transcriptase, is carried by retroviruses inside their capsids. The hepadnavinuses syathesize their own reverse transcriptese erzyme inside the cell (tis called DNA polymerase because itis used to replicate their DIA) but do not carry it with them in their capsids. An RNA dependent RMA polymerase is required by all the negative sense RNA viruses (they are not infsclious without it). Other RNA viruses will synthesize their own RMA dependent RMVA polymerase once they are in the cell, but do not require é for stasting the infection, None of the DNA viruses carry such a polymerase with them into the cell, No additional reagent is required would be true to start an infection with the majority of the DIVA viruses (exchiding poxviruses) and the positive sense RNA vimises (excluding the retroviruses). These viruses are directly infectious, i the naked genomic nucleic acidis directly injected into a cell Total Questions BEBE EK ERB BEGY eRe eRe He SRBBARBEBS XxXxXXKXKXKXKXKXKXKXKXKKXKKXKKKKKKKKKKKKKK 3/1/2014 8:24:47 PM Mark this question & => Question Td : 141762 Question 22 of 30 A patient presents to the emergency department with chest pain end cough occasiorally productive of bloody sputum, CXR reveals a cavitary lesion in the lung. Auramine rhodanine stain of the sputum is postive. The characteristic of the likely infectious agent that ives it resistance to Gram stain reagents is 2) Mycolic acid C +) Polymer of N acetyl gucosamine and N acetyl muramic acid 6) Sulfatide 6) Tuberculin ©) 2, 3- Ketodzoxvoctonate Anower [JEQIRNSVER) otner User=txplanation Report An Error Question Explanation: This patient has tberculosis, as revealed by the Auramine rhodamine stain of the sputum, Tuberculosis is caused by Mycobacterium tuberculosis, which is a non Gram staining bacilus. Itis the high lipid content of the cellular envelope that makes the orgarisma refractory to the agents of the Gram stain and very resistant to changes in temperature and pH. These long chain faits/acids found in the envelope are called mycolic acids. A polymer of N-acetyl gucosamine and N-acetyl muramic acid describes the peptidegtycan cell wall, This is the layer that gives cels their Gram staining characteristics, but because itis hidden inside the maycolic acids of the envelope of Mycobacterium . the organism does not stain with the Gram stain, Sulfatides are sulfolinids that are hydrolyzed to forrn sulfuric acid. These are the molecules that inhibit the fusion of the phagosome and the lysosome aad allow M. tuberculosis te survive inside macrophages. Tuberculin is the surface protein of the organism, which is used to elicit the delayed-type hypersensitivity skin test (Mantom test), used to diagnose exposure to the disease. 2, 3 Ketodeoxyoctonate is the molecule thatis missing from the fipopolysaccharide fram the genus Bacteroides. Absence of this component makes Racteraides endotoxin less toxic than that af most gram negative bacill, 3/1/2014 8:24:47 PM Mark this question & => Question Td : 141762 Question 22 of 30 A patient presents to the emergency department with chest pain and cough occasionally productive of bloody spuinam, CXR reveals a cavitary lesion in the lung. Auramine rhodanine stain of the sputum is postive. The characteristic of the likely infectious agent that ives it resistance to Gram stain reagents is Y © 2 Mycolic acid C +) Polymer of N acetyl gucosamine and N acetyl muramic acid 6) Sulfatide 6) Tuberculin ©) 2, 3- Ketodzoxvoctonate Anower [JEQIRNSVER) otner User=txplanation Report An Error Question Explanation: This patient has tberculosis, as revealed by the Auramine rhodamine stain of the sputum, Tuberculosis is caused by Mycobacterium tuberculosis, which is a non Gram staining bacilus. Itis the high lipid content of the cellular envelope that makes the orgarisma refractory to the agents of the Gram stain and very resistant to changes in temperature and pH. These long chain faits/acids found in the envelope are called mycolic acids. A polymer of N-acetyl gucosamine and N-acetyl muramic acid describes the peptidegtycan cell wall, This is the layer that gives cels their Gram staining characteristics, but because itis hidden inside the maycolic acids of the envelope of Mycobacterium . the organism does not stain with the Gram stain, Sulfatides are sulfolinids that are hydrolyzed to forrn sulfuric acid. These are the molecules that inhibit the fusion of the phagosome and the lysosome aad allow M. tuberculosis te survive inside macrophages. Tuberculin is the surface protein of the organism, which is used to elicit the delayed-type hypersensitivity skin test (Mantom test), used to diagnose exposure to the disease. 2, 3 Ketodeoxyoctonate is the molecule thatis missing from the fipopolysaccharide fram the genus Bacteroides. Absence of this component makes Racteraides endotoxin less toxic than that af most gram negative bacill, ‘Marke this question <>=> Question 23 of 30 A 56 year old mate renamns from a vacation at the beach, On vacation he and his family consumed several meals consisting primarily of seafood, including lobster and raw oysters. Although the other members of his family cid not get sick, he developed hypotension, confusion, and! a bullous rash over his lower extremities. The past medical history of this patient would be significant for 2) cate pharyngitis ) Alcoholism and liver disease ¢) Imaraunocompromice 4) Intravenous deug abuse ¢) Renal transplent Anower (UBQURISEM) otter vsors Explanation Report An Error Question Explanation: Vibrio vulnificus septicemia is contracted by consuming raw oysters, Paticnis with chronic renal insufficiency, liver disease hematopoietic disorders, and a past history of alcoholism are particularly at risk. Septicemia cause by V. vulnificus generally begins with chills fever, and hypotension, and skin lesions tend to occur24to48 hours after the onset of the infection. Acute pharmaitis is a precursor condition to the development of the poststreptococcal immunologic sequelae: sheumatic fever and acute glomerulonephnitis. Rheumatic fever is a type I hypersensitivity disease, and poststreptococcal glomerdonephntis is a type TI hypersensitivity disease. Tmmunocompromise predisposes patients to a wide variety ofiinfectious diseases such as cytomegalovirus, Peumocystis, Listeria, and Mycobacterium as well as many cthers Intravenous dmg abuse commonly introchices normal flora organisms into the bloodstream. These individuals have problems with endocarditic and septicemia; however, since Vibrio enters the body through the digestive tract, itis not more common folowing intravenous injection. Renal transplant patients are immunocompromised to prevent rejection of the transplanted organ and are therefore susceptible to a broad range of infectious diseases, but are not an especially "at-risk" group for Vibrio infectons. ‘Marke this question <>=> Question 23 of 30 A 56 year old mate renamns from a vacation at the beach, On vacation he and his family consumed several meals consisting primarily of seafood, including lobster and raw oysters. Although the other members of his family cid not get sick, he developed hypotension, confusion, and! a bullous rash over his lower extremities. The past medical history of this patient would be significant for 2) cate pharyngitis Y © b) Alcoholism and liver disease ¢) Imaraunocompromice 4) Intravenous deug abuse ¢) Renal transplent Anower (UBQURISEM) otter vsors Explanation Report An Error Question Explanation: Vibrio vulnificus septicemia is contracted by consuming raw oysters, Paticnis with chronic renal insufficiency, liver disease hematopoietic disorders, and a past history of alcoholism are particularly at risk. Septicemia cause by V. vulnificus generally begins with chills fever, and hypotension, and skin lesions tend to occur24to48 hours after the onset of the infection. Acute pharmaitis is a precursor condition to the development of the poststreptococcal immunologic sequelae: sheumatic fever and acute glomerulonephnitis. Rheumatic fever is a type I hypersensitivity disease, and poststreptococcal glomerdonephntis is a type TI hypersensitivity disease. Tmmunocompromise predisposes patients to a wide variety ofiinfectious diseases such as cytomegalovirus, Peumocystis, Listeria, and Mycobacterium as well as many cthers Intravenous dmg abuse commonly introchices normal flora organisms into the bloodstream. These individuals have problems with endocarditic and septicemia; however, since Vibrio enters the body through the digestive tract, itis not more common folowing intravenous injection. Renal transplant patients are immunocompromised to prevent rejection of the transplanted organ and are therefore susceptible to a broad range of infectious diseases, but are not an especially "at-risk" group for Vibrio infectons. Mark this question <& => Question Td : 145343 Question 24 of 30 A 28 year old Mexican American female receives an intradermal tuberculin injection and later develops an indurated, erythematous papale 12 mmin diameter. The molecule most important in simalating this response is found in which one of the following location? a) Cell wall ) Cytoplasmic membrane ©) Flagella 4) Outer curface protein ©) Fenplasmic space Question Explanation: “This woman has developed a posttive tuberculin skin test, indicating exposure to Mycobacterum tuberculosis, The molecule injected into the skin of the forearm to elicit this response is a purified protein derivative (PPD), which is composed of tuberculin (an outer surface protein) and mycolic acids of the organism. The cell wall of mycobacteria cortains a thinner than normal layer of peptidoglycan, but this material does not contribute to the formation of the delayed-type hypersensitivity response. The cytoplasmic membrane of mycobacteria is a phospholipid bilayer that is similar to the inner membranes of all other prokaryotes and does not contribute to the specific skin test for tuberculosis. Flagrlla are not present on Mycobacterum tuberculosis, because the organism is not motile. A periplasmic space is not possessed by Mycobacterium tuberculosis because + is an organism that is taxonomically related to the gram-positives. Only gram negative bacteria possess a periplasmic space Mark this question <& => Question Td : 145343 Question 24 of 30 A 28 year old Mexican American female receives an intradermal tuberculin injection and later develops an indurated, erythematous papule 12 mumin diemeter. The molecule most important in stimulating thie responce is found ia which one of the following location? 2) Cell wall ) Cytoplasmic membrane ) Flagella YM © 4) Outer surface protein ©) Ferplasmic space Question Explanation: “This woman has developed a posttive tuberculin skin test, indicating exposure to Mycobacterum tuberculosis, The molecule injected into the skin of the forearm to elicit this response is a purified protein derivative (PPD), which is composed of tuberculin (an outer surface protein) and mycolic acids of the organism. The cell wall of mycobacteria cortains a thinner than normal layer of peptidoglycan, but this material does not contribute to the formation of the delayed-type hypersensitivity response. The cytoplasmic membrane of mycobacteria is a phospholipid bilayer that is similar to the inner membranes of all other prokaryotes and does not contribute to the specific skin test for tuberculosis. Flagrlla are not present on Mycobacterum tuberculosis, because the organism is not motile. A periplasmic space is not possessed by Mycobacterium tuberculosis because + is an organism that is taxonomically related to the gram-positives. Only gram negative bacteria possess a periplasmic space Mark this question = => Question Td : 145571 Question 25 of 30 Urine grams stain of a 64 year old black male with a history of fatigue, tachycardia, tachypuea, and anemia who presents with fever, inritative voiding symptoms, and perineal pain. is positive for gram negative rods. Which one of the following, when given in high dose for 21 days, is associated with the highest risk for development of hemolytic anciaia? a) Ampicillin ) Cefaclor ©) Ciprofloxacin 4) Sulfamethoxazole/timethoprim ) Tetracycline Question Explanation: ‘The patient has acute prostatitis, which is characterized by fever, chills and dysuria, with a swollen, extremely tender prostate on rectal exam The urine Gram’s stain and culture will generally he positive. The treatment regimen for this bacterial infection is typically 2.21 day course of ampicillin, a fuoroquinolone, or sulfamethoxazole/trimethoprim(SM3CTMP). However, this patient has symptoms and signs of G6ED deficiency, which is an X-lnked recessive disorder affecting 10% to 15% of African-American males. The medications most commonly associated with the induction of hemolytic anemia in deficient patients are sulfonamides, ntrofurantoin, dapsone, primacuine, and quinine. The sulfamethoxazole in the SMM{-TMP combination is a sulfonemie, and can produce hemolytic anemia in patients with G6ED deficiency. Ampicilinis a broad spectrum penicillin antibiotic commonly used in the treaiment of infections in the genitourinary respiratory, or Gl tracts, as well as in the skin and soft tissues. Cefaclor is a second generation, cephalosporin indicated for a vatiety of bacterial infections, including those of the respiratory and Gi tracts. Ciprofloxacin is a fucroquinclone commonly used in the treatment of serious infections caused by gram negative organisms. Tetracycline is most commonly used in the treatment of acne vulgatis and gonococcal infections. Report An Error Mark this question = => Question Td : 145571 Question 25 of 30 Urine grams stain of a 64 year old black male with a history of fatigue, tachycardia, tachypuea, and anemia who presents with fever, inritative voiding symptoms, and perineal pain. is positive for gram negative rods. Which one of the following, when given in high dose for 21 days, is associated with the highest risk for development of hemolytic anciaia? a) Ampicillin ) Cefaclor ©) Ciprofloxacin Y © 4) Sulfamethoxazole/timethoprim: ) Tetracycline Question Explanation: ‘The patient has acute prostatitis, which is characterized by fever, chills and dysuria, with a swollen, extremely tender prostate on rectal exam The urine Gram’s stain and culture will generally he positive. The treatment regimen for this bacterial infection is typically 2.21 day course of ampicillin, a fuoroquinolone, or sulfamethoxazole/trimethoprim(SM3CTMP). However, this patient has symptoms and signs of G6ED deficiency, which is an X-lnked recessive disorder affecting 10% to 15% of African-American males. The medications most commonly associated with the induction of hemolytic anemia in deficient patients are sulfonamides, ntrofurantoin, dapsone, primacuine, and quinine. The sulfamethoxazole in the SMM{-TMP combination is a sulfonemie, and can produce hemolytic anemia in patients with G6ED deficiency. Ampicilinis a broad spectrum penicillin antibiotic commonly used in the treaiment of infections in the genitourinary respiratory, or Gl tracts, as well as in the skin and soft tissues. Cefaclor is a second generation, cephalosporin indicated for a vatiety of bacterial infections, including those of the respiratory and Gi tracts. Ciprofloxacin is a fucroquinclone commonly used in the treatment of serious infections caused by gram negative organisms. Tetracycline is most commonly used in the treatment of acne vulgatis and gonococcal infections. Report An Error [Wa emac.omran- Yahoo Wi x ) 8 www.interface.edu.pk/tr x \\_— € > C BG www.interface edu nk /medical-exams/test-analysis pho 2utd=14759 * 8 Apps EJ coogle A settings [)Sgnin Bi isle dul En yo uedlldaye GP..aio yt Soioll JS |) Free Hotmall >» (©) Other bookmar ‘Matte this question <= => Question Td : 146758 | Total Questions i. ose Question 26 af 30 a ‘A person whe is not known to be infected with the kuman immunodeficiency virus @IIV) may become symptomatic afer avaiable = 3X number of years of disease, IEno treatment has been given and the patient has not developed an AIDS defining ilness, the pattem that 4X ‘would most icely to be found in this patieat is 5 x a) & x [Virus, p24, RNA[CD4+ _[Anti-p24 [Anti-env 7 x fincreasing [Decreasing [Decreasing] |ncreasing slighty a x by 2 x [Virus, p24, RNA[CD4+ — [Anti-p24 |[Ant-env lg x ffncreasing [Decreasing |Increasing| [Decreasing slightly] dd, ee ) 1 Xx 2 oe (Virus. p24, RNACD4+ _ [Aati-p24 [[Anti-eav ee {increasing ho changelfincreasing|Decreasing slighty] is ie 7] 16 Xx [Wirus. p24, RNA[CD4+ [Anti-n24 |[Ant-eav a x [Decreasing _[[NosmallVo change|Vo change! fe We °) 2. [Wirus. p24, RNA[CD4+ [Acti-n24 |[Anti-env 2 Xx fTndetectable _|f'Tormall[Decreasinglincreasing| Ql, oes 22 Answer (Bannon) Other User's Explanation Report An Error ae Se Question Explanation: “a x The p24 protein is a capsid protein produced by the gag gene. After acute infection the HIV virns, p24, and HIV ENA can be a ihe detected; however within months these drop to undetectable levels coincident with antibody production to p24. During this acute ee phase, the CD4 count begins to fill, Antibody to the env antigen also becomes detectable af the same time as the p24 antibody but its rise is much slower and continues at an aknost constary level for many years. After he intial rapid rise in p24 anibody the level «22. remains constant for a variable number of years. When it starts to fall the patieat becomes symptomatic. Atthis tme, the CD4 count 28 X drops cramatically. During the syraptomatic phase as the p24 antbody and CD4 counts are dropping the vis p24 antigen and viral 29 RNA again become detectable Without treatmort viral replication cortinses in the lymphatics. Therefore ifa patort whe hasbeen 3. infected is now symptomatic the virus p24 antigen and viral RIVA are increasing the CD4 count is decreasing, the antibody to p24 is decreasing and there is a slight increase in the antibody to the env antigen. Therefore the only correct combination is answer A. None eBay ape ne ce [Wa emac.omran- Yahoo Wi x ) 8 www.interface.edu.pk/tr x \\_— € > C BG www.interface edu nk /medical-exams/test-analysis pho 2utd=14759 * 8 Apps EJ coogle A settings [)Sgnin Bi isle dul En yo uedlldaye GP..aio yt Soioll JS |) Free Hotmall >» (©) Other bookmar ‘Matte this question <= => Question Td : 146758 | Total Questions i. ose Question 26 af 30 a ‘A person whe is not known to be infected with the kuman immunodeficiency virus @IIV) may become symptomatic afer avaiable = 3X number of years of disease, IEno treatment has been given and the patient has not developed an AIDS defining ilness, the pattem that 4X ‘would most icely to be found in this patieat s 5 x YOoa 6 x [Virus, p24, RNA[CD4+ _[Anti-p24 [Anti-env 7 x fincreasing [Decreasing [Decreasing] |ncreasing slighty a x ») 2 Xx (Virus, p24, RNACD4+ __[Anti-p24 |[Ant-env lo Xx ffncreasing [Decreasing |Increasing| [Decreasing slightly] dd, ee ) 1 Xx 2 oe (Virus. p24, RNACD4+ _ [Aati-p24 [[Anti-eav ee {increasing ho changelfincreasing|Decreasing slighty] is ie 7] 16 Xx [Wirus. p24, RNA[CD4+ [Anti-n24 |[Ant-eav a x [Decreasing _[[NosmallVo change|Vo change! fe We °) 2. [Wirus. p24, RNA[CD4+ [Acti-n24 |[Anti-env 2 Xx fTndetectable _|f'Tormall[Decreasinglincreasing| Ql, oes 22 Answer (Bannon) Other User's Explanation Report An Error ae Se Question Explanation: “a x The p24 protein is a capsid protein produced by the gag gene. After acute infection the HIV virns, p24, and HIV ENA can be a ihe detected; however within months these drop to undetectable levels coincident with antibody production to p24. During this acute ee phase, the CD4 count begins to fill, Antibody to the env antigen also becomes detectable af the same time as the p24 antibody but its rise is much slower and continues at an aknost constary level for many years. After he intial rapid rise in p24 anibody the level «22. remains constant for a variable number of years. When it starts to fall the patieat becomes symptomatic. Atthis tme, the CD4 count 28 X drops cramatically. During the syraptomatic phase as the p24 antbody and CD4 counts are dropping the vis p24 antigen and viral 29 RNA again become detectable Without treatmort viral replication cortinses in the lymphatics. Therefore ifa patort whe hasbeen 3. infected is now symptomatic the virus p24 antigen and viral RIVA are increasing the CD4 count is decreasing, the antibody to p24 is decreasing and there is a slight increase in the antibody to the env antigen. Therefore the only correct combination is answer A. None eBay ape ne ce 3/1/2044 8:26:00 PM ‘Mark this question & => Question Id : 162365, Question 27 of 30 ‘An alcoholic male aged 57 years is hospitalized a road trafic accident. Twenty hows after the accident he develops fever and a cough productive of purulent sputum IFCXR shows lobar consolidation and the sputum reveals the presence of gram-negative encapsulated rods, what would be the appropriate intial therapy? a) Cefotasime (CV) b) Erythromycin c) Gertamicin (IV) +vancomycin (IV) &) Ticarcillin-clavalanic acid (IV) +vancomycin, e) Trimethoprim-sulfamethozazole (IV) Answer (Brisnaton) Other User's Explanation Report An Error Question Explanation: ‘The patient is presenting with signs and symptoms of pneumonia, Since the patient's signs and symptoms are appearing 20 hours after admission, bis pneumonia is most likely a 'commnnity-acqnired” preumonia; ane of the criteria for diagnosing a nosocomial infection is that the infection must not occur before 48 hours after admission The appearance of plump gram negetive encapsulated rods in an alcoholic is highly suggestive of Klebsiella pneumonia The most appropriate treatment of Klebsiella pneumonia in this patient is the administration of a third-generation cephalosporin, such as cefotaxime, ceftriaxone, ot ceftazidime, Erythromycin is a macrolide antibiotic used in the treatment of a variety of gram-negative and gram-positive infections. Although itis not the treatment of choice in this patient, itis the treatment of choice of Legionella pneumenia, Gentamicin is an aminoglycoside antbiotic that is generally added to improve the efficacy of broad spectrum antibiotics, such as penicilin and cephalosponns, in the treatment of Pseudomonas and Enterococcus infections. Vancomycin is an antibiotic used in the treatment of life-threatening infections caused by gram-positive infections, the use of this agent in a gram-negative infection is inappropriate, Because of the severe nature of this infection, the use of oral agent such as trimethoprim sulfamethoxazole, would be inappropriate. 3/1/2044 8:26:00 PM ‘Mark this question & => Question Id : 162365, Question 27 of 30 ‘An alcoholic male aged 57 years is hospitalized a road traffic accident. Twenty hours after the accidert he develops fever and a cough productive of punutent spuittm TFCXR shows lobar consolidaton ard the sputim reveals the presence of gram-negative eacapauleted rods, what would be the appropriate intial therapy? Y © a) Cefotaxime IV) b) Exythromycin ¢) Gentamicin (V) +vancomycin IV) ) Ticarcillin-clavulanic acid (TV) Hvancomycin, e) Trimethoprim-sulfamethozazole (IV) Answer (Brisnaton) Other User's Explanation Report An Error Question Explanation: ‘The patient is presenting with signs and symptoms of pneumonia, Since the patient's signs and symptoms are appearing 20 hours after admission, bis pneumonia is most likely a 'commnnity-acqnired” preumonia; ane of the criteria for diagnosing a nosocomial infection is that the infection must not occur before 48 hours after admission The appearance of plump gram negetive encapsulated rods in an alcoholic is highly suggestive of Klebsiella pneumonia The most appropriate treatment of Klebsiella pneumonia in this patient is the administration of a third-generation cephalosporin, such as cefotaxime, ceftriaxone, ot ceftazidime, Erythromycin is a macrolide antibiotic used in the treatment of a variety of gram-negative and gram-positive infections. Although itis not the treatment of choice in this patient, itis the treatment of choice of Legionella pneumenia, Gentamicin is an aminoglycoside antbiotic that is generally added to improve the efficacy of broad spectrum antibiotics, such as penicilin and cephalosponns, in the treatment of Pseudomonas and Enterococcus infections. Vancomycin is an antibiotic used in the treatment of life-threatening infections caused by gram-positive infections, the use of this agent in a gram-negative infection is inappropriate, Because of the severe nature of this infection, the use of oral agent such as trimethoprim sulfamethoxazole, would be inappropriate. mec.cmran- Yahoo Wi x) 8 www. interface.edu.pkii € php dutid=14759 oo wsdl dine Ofte ge sgsoll JS) Free Hatmall CL www. interface.edu.pk/med apps Eljcoogle Asetings [)Sgnin Zou wos dul xarns ftest-analy' * 8 Gi other bookmar ‘Marte this question <= => (Question Td : 201568 Question 28 of 30 (Out of the following, which one is characteristic of Barter’s syndrome? a) Diatthora ) Hyperkalaemia ©) Metabolic acidosis 4d) Secondary hyperaldosteronism ¢) Reduced renal concentrating ability Question Explanation: Bartter's syndrome is a rare form of renal potassium wasting characterised by hypokalaemia, notmotension, and elevated renin and aldosterone levels, Itis occasionally autosomal recessive. There is hyperplasia ofthe justaglomeruler apparatus in most cases. Itis postulated that the primary defects in chloride reabsorption in the escending limb, resulting in sodium chloride excessively presented to the distal tubule, with sodium reabsorption in exchange for potassium, resulting in urinary sodium wasting There is secondary stimulation of prostaglandin synthesis, which activates the renin angiotensin aldosterone system which exacerbates the renal potassium, wasting - Grown failure -muscle weakness = constipation -polyuria and -dehydration are typical in younger children; with muscle weakness ~ cramps or -carpopedal spasms, presentin older children. The potassium is <2.SmmolL, there is metabolic alkalosis, and hyperammonaemia with hyperaldosteronisrn, There are high levels of urnery potassium and chloride. The high urinary chloride level is helpful in distinguishing i: fom similar presentations which have low urinary chlotide levels, such as -lquotice, laxative, or diuretic use ~ persistent vomiting or diarrhoea -pyelonephritic, or -diabetes insipidus Oral potassium and indomethacin may be used. Total Questions SBRARBEBBEBBRHR EK EE BESSY eR eee eee XxXxXXKXKXKXKXKXKXKXKXKKXKKXKKKKKKKKKKKKKK mec.cmran- Yahoo Wi x) 8 www. interface.edu.pkii € php dutid=14759 oo wsdl dine Ofte ge sgsoll JS) Free Hatmall CL www. interface.edu.pk/med apps Eljcoogle Asetings [)Sgnin Zou wos dul xarns ftest-analy' * 8 Gi other bookmar ‘Marte this question <= => (Question Td : 201568 Question 28 of 30 ‘Out of the folowing, which one is characteristic of Barter’s syndrome? a) Diatthora b) Hyperkalaemia ©) Metabolic acidosis Y © 4) Secondary hyperaldosteronism €) Reduced renal concentrating abilisy Question Explanation: Bartter's syndrome is a rare form of renal potassium wasting characterised by hypokalaemia, notmotension, and elevated renin and aldosterone levels, Itis occasionally autosomal recessive. There is hyperplasia ofthe justaglomeruler apparatus in most cases. Itis postulated that the primary defects in chloride reabsorption in the escending limb, resulting in sodium chloride excessively presented to the distal tubule, with sodium reabsorption in exchange for potassium, resulting in urinary sodium wasting There is secondary stimulation of prostaglandin synthesis, which activates the renin angiotensin aldosterone system which exacerbates the renal potassium, wasting - Grown failure -muscle weakness = constipation -polyuria and -dehydration are typical in younger children; with muscle weakness ~ cramps or -carpopedal spasms, presentin older children. The potassium is <2.SmmolL, there is metabolic alkalosis, and hyperammonaemia with hyperaldosteronisrn, There are high levels of urnery potassium and chloride. The high urinary chloride level is helpful in distinguishing i: fom similar presentations which have low urinary chlotide levels, such as -lquotice, laxative, or diuretic use ~ persistent vomiting or diarrhoea -pyelonephritic, or -diabetes insipidus Oral potassium and indomethacin may be used. Total Questions SBRARBEBBEBBRHR EK EE BESSY eR eee eee XxXxXXKXKXKXKXKXKXKXKXKKXKKXKKKKKKKKKKKKKK Mark ths question <= => Question 1d: 203402 Question 29 of 30 A contraindication to immunization is which one of the following? 8) A contraindication to immunization is which one of the following? ) A child with congenital adrenal typerplasia on oral cortisone ©) A history of prolonged jaundice 6) Infantile eczema requiring topical stervids. 8) Oral potiomyelitis vaccine to a child on oral steroids Question Explanation Common misconceptions regarding immunizations include + A family history of adverse reaction, or a previous history pertussis, measles, mbella or rmamps infection + Promaturity or low birth weight + Stable neurological conditions such as cerebral palsy or Down syndrome + Asthma, eczema, hagfever or sauffles * Contact with an infectious disease, or treatment with antibiotics or topical steroids + Pregnant mother or 2 mother who is breast feeding + Prolonged jaundice + Patients on replacement corticosteroids ral polio vaccine should not be given to immunosuppressed children, their siblings or household contacts Tn children with HIV, there is litle evidence that they themselves will have problems, but excretion may be prolonged, and this may give rise to an increased risk of infection of HIV positive household contacts. Mark ths question <= => Question 1d: 203402 Question 29 of 30 A contraindication to imraunization is which one of the following? a) A contraindication to immunization is which one of the following? b) A child with congenital adrenal hyperplasia on oral cortisone ©) A history of prolonged jaundice ) Infantile eczema requiring topical steroids. ¥ © ®) Oral poliomyelitis vaccine to a child on oral steroids Question Explanation Common misconceptions regarding immunizations include + A family history of adverse reaction, or a previous history pertussis, measles, mbella or rmamps infection + Promaturity or low birth weight + Stable neurological conditions such as cerebral palsy or Down syndrome + Asthma, eczema, hagfever or sauffles * Contact with an infectious disease, or treatment with antibiotics or topical steroids + Pregnant mother or 2 mother who is breast feeding + Prolonged jaundice + Patients on replacement corticosteroids ral polio vaccine should not be given to immunosuppressed children, their siblings or household contacts Tn children with HIV, there is litle evidence that they themselves will have problems, but excretion may be prolonged, and this may give rise to an increased risk of infection of HIV positive household contacts. ‘Marke this question <= Question 30 of 30 Question Td = 206 A 62 year old man is admitted with community acquired pneumonia and deteriorated over the next few hours. Poor prognosis is indicated by which one of the following? a) A total white cell count of 17 x10 4-11) b) Blood pressure of 110/70 mmHg c) Respiratory rate of 35 breaths min ) Rigors @) Temperature of 39°C: Question Explanatiot The presence of -raised wea (*7mM) - hypotension (diastolic EP equal or <60mmEz) and - respiratory rate equal or > 30/min is associated with significantly increased risk of death. (Other fess important features of severe pneumonia include - older age (>60) - comorbidiy - confiision - cyanosis - white blood cells (WBC) <4000 er > 20000 = hypoxia and ~ chest X-ray (CCR) with multi-lobe invelvement, Report An Error ‘Marke this question <= Question 30 of 30 Question Td = 206 A 62 year old man is admitted with community acquired pneumonia and deteriorated over the next few hours. Poor prognosis is indicated by which one of the following? a) A total white cell count of 17 x10 4-11) b) Blood pressure of 110/70 mmHg VY © 6) Respiratory rate of 35 breaths /min ) Rigors @) Temperature of 39°C: Question Explanatiot The presence of -raised wea (*7mM) - hypotension (diastolic EP equal or <60mmEz) and - respiratory rate equal or > 30/min is associated with significantly increased risk of death. (Other fess important features of severe pneumonia include - older age (>60) - comorbidiy - confiision - cyanosis - white blood cells (WBC) <4000 er > 20000 = hypoxia and ~ chest X-ray (CCR) with multi-lobe invelvement, Report An Error Merle this question => Question Td : 31047 Question 1 of 30 An 80 year old lady is reviewed aiter the staff of the aursing home in which she resides expressed concem regarding a vaginal discharge. She has been in the aursing home for the last one year with a profound Alcheimer‘s dementia, Neisseria gonorshoea® is seen on culture of discharge. The most appropriate strategy in this patient is a) Contact the pelice ) Contact tracing of sexual partners ) Informal enquiry to the nursing home &) Treat the patient and discharge back to the rursing hore ©) Seek advice from the MDU Question Explanation: These ethical questions can be quite tough to answer with accuracy. ‘This question specifically relates to elder abuse in this case potential elder sexual abuse. The scenano is one thats often played out m the press, in which a care worker sexually abuses elderly patients in his or her care. However, you are given very litle information here and what you would do is undoubte dly treat the patients and establish how she contracted gonorchoea The question states that she has a profound dementia, suggesting that abuse has occurred rather than consensual sex. Nonetheless, you nced to establish the fact Youhave a personal duty of care to the patients and next of kin to do this before contacting the police. Itis likely that the police vwillneed to be called but first it would be worth telking things through and obtaining advice from the MDU. The advice may entail investigating the set-up at the nursing home, talking with the next of kin, (and social services if social worker has been invelved) and finely detailing any injuries that may be present on examination, Only then should the decision io contact the police be made Merle this question => Question Td : 31047 Question 1 of 30 An 80 year old lady is reviewed aiter the staff of the aursing home in which she resides expressed concem regarding a vaginal discharge. She has been in the aursing home for the last one year with a profound Alcheimer‘s dementia, Neisseria gonorshoea® is seen on culture of discharge. The most appropriate strategy in this patient is a) Contact the pelice ) Contact tracing of sexual partners ) Informal enquiry to the nursing home &) Treat the patient and discharge back to the rursing hore WV © 2) Seek advice from the MDU Question Explanation: These ethical questions can be quite tough to answer with accuracy. ‘This question specifically relates to elder abuse in this case potential elder sexual abuse. The scenano is one thats often played out m the press, in which a care worker sexually abuses elderly patients in his or her care. However, you are given very litle information here and what you would do is undoubte dly treat the patients and establish how she contracted gonorchoea The question states that she has a profound dementia, suggesting that abuse has occurred rather than consensual sex. Nonetheless, you nced to establish the fact Youhave a personal duty of care to the patients and next of kin to do this before contacting the police. Itis likely that the police vwillneed to be called but first it would be worth telking things through and obtaining advice from the MDU. The advice may entail investigating the set-up at the nursing home, talking with the next of kin, (and social services if social worker has been invelved) and finely detailing any injuries that may be present on examination, Only then should the decision io contact the police be made € > C BG www.interface.edu.nk/medical-exams/test-analysis pho 2utd=14759 #8 HE apps Jcoogle A settings (Cisgnin Zo. wlohe dul Eon go usilldye Gla ye Sosoll JS [| Free totmal — » (3 other bookmar © ‘Mark this question @qc> Question Id :59380_] Total Questions Question 2 of 30 A 32 year old woman comes in with a soft smooth erythematous nodule on her lower ip. See picture: She states that a few weeks ago she had some chapped lips with occasional bleeding. Now, the lips have healed but this lesion arose suddenly in its place. Itis occasionally tender on pressure What is the most likely diagnosis? a) HSV1 'b) Pyogeric gramloma c) Cherry hemangioma d) Dermal news Answer Other User's Explanation Report An Error Question Explanatios Pyogenic granioma can occur at any mucosal location of arute or chronic trauma, or infection. Th the mouth the vast majority of these very commen lesions occur on the gingiva, where they may develop as damb bell shaped masses on the facial and lingual susfaces of the dental arch, connected by a thin isthmus between adjacent teeth, Other sites of common involvemert include the tongue, the lip mucosa and vermilion, and the buccal mucosa. Allages and both genders are susceptible to this exuberant inflammatory response. The lesion is usually a pedunculated, bright red mass with or without white areas of sueface ulceration; seme lesions have a normal coloration, Rarely does pyogenic granuloma excsed 2.5 cm. in size andiit usually reaches iis full size within weeks or months, remeining indefinitely thereafter XxXxXXKXKXKXKXKXKXKXKXKKXKKXKKKKKKKKKKKKKK SBBARBEBBEBBERHREKEEBREBY eRe eee ee € > C BG www.interface.edu.nk/medical-exams/test-analysis pho 2utd=14759 #8 HE apps Jcoogle A settings (Cisgnin Zo. wlohe dul Eon go usilldye Gla ye Sosoll JS [| Free totmal — » (3 other bookmar © ‘Mark this question @qc> Question Id :59380_] Total Questions Question 2 of 30 A 32 year old woman comes in with a soft smooth erythematous nodule on her lower ip. See picture: She states that a few weeks ago she had some chapped lips with occasional bleeding. Now, the lips have healed but this lesion arose cucdenly in ite place. Ikic occasionally tender on preseure. What is the most likely diagnosis? a) BSV1 JY © b)Pyogenic granuloma c) Chesry hemangioma 4) Dermal news Answer Other User's Explanation Report An Error Question Explanatios Pyogenic granioma can occur at any mucosal location of arute or chronic trauma, or infection. Th the mouth the vast majority of these very commen lesions occur on the gingiva, where they may develop as damb bell shaped masses on the facial and lingual susfaces of the dental arch, connected by a thin isthmus between adjacent teeth, Other sites of common involvemert include the tongue, the lip mucosa and vermilion, and the buccal mucosa. Allages and both genders are susceptible to this exuberant inflammatory response. The lesion is usually a pedunculated, bright red mass with or without white areas of sueface ulceration; seme lesions have a normal coloration, Rarely does pyogenic granuloma excsed 2.5 cm. in size andiit usually reaches iis full size within weeks or months, remeining indefinitely thereafter XxXxXXKXKXKXKXKXKXKXKXKKXKKXKKKKKKKKKKKKKK SBBARBEBBEBBERHREKEEBREBY eRe eee ee ‘Marke this question = Question Ta : 59400 Question 3 of 30 Tnfections agert more often associated with hospital acquired pneumonia than community acquired pneumonia is a) Streptococcus pneumoniae 6) Hemophilus influenza ©) Chlamydia pneumoniae 6) Legionella €) Mycoplasma pneumoniae Question Explanation: Legionella pneumophila most often causes pneumonia, with extrapulmonary features. Diagnosis requires specilic growth media, serologic testing, ot PCR analysis. Treatment is with doxycycline, macrolides, or fluoroquinolones. The Ist appearance of this organism was in 1976 at a conventica of the American Legion, thus the name Legionnaires disesse. Nonpnoumonic Infection is called Pontiac fever. The organisms can be found in soil and freshwater. Manufactured water storage containers, inciuding water-cocled air conditioning urits, enhance its growth. Spread is most likely by aerosols of potable water. Legionnaires’ disease is a flu-like syndeeme with acute fever, chills, malaise, myalgics, headache, or confusion. Frequertly nausea, loose stoolshvatery diarchea, abdominal pain, cough, and arthralgias also occur. Pucumonic manifestations may include dyspnea, pleustic pain, and hemoptysis. n Report An Error ‘Marke this question = Question Ta : 59400 Question 3 of 30 Tnfections agent mare aften associated with hospital acquired preumonia than commnnity accnired pneumonia is a) Streptococcus pneumoniae 'b) Hemophitue inuenza ©) Chlamydia pneumoniae SY @ A Legionella ©) Mycoplasma pneumoniae Question Explanation: Legionella pneumophila most often causes pneumonia, with extrapulmonary features. Diagnosis requires specilic growth media, serologic testing, ot PCR analysis. Treatment is with doxycycline, macrolides, or fluoroquinolones. The Ist appearance of this organism was in 1976 at a conventica of the American Legion, thus the name Legionnaires disesse. Nonpnoumonic Infection is called Pontiac fever. The organisms can be found in soil and freshwater. Manufactured water storage containers, inciuding water-cocled air conditioning urits, enhance its growth. Spread is most likely by aerosols of potable water. Legionnaires’ disease is a flu-like syndeeme with acute fever, chills, malaise, myalgics, headache, or confusion. Frequertly nausea, loose stoolshvatery diarchea, abdominal pain, cough, and arthralgias also occur. Pucumonic manifestations may include dyspnea, pleustic pain, and hemoptysis. n Report An Error Mark this question & => Question Ti : 66845 Question 4 of 30 A.red, painfil rash associated with swelling of the face or extremities and characterized by a sharp margin is associcted with infection of which of the following organism? a) Candida albicans +) Staphylococcus aureus ©) Herpes simplex & Streptococcus pyogenes ©) Paillomaviruses Anower [UBRPINSHER) otter Users Explanation Report An Error Question Explanation: The skin condition described is erysipelas, which is caused by S. pyogenes and responds well to penicillin Mark this question & => Question Ti : 66845 Question 4 of 30 ‘A.red, painfil rash associated with sweling of the face or extremities and chacacterized by a sharp margin is associated with infection of which of the following organism? a) Candida albicans 6) Staphylococcus aureus ©) Herpes simplex Y © @) Streptococcus pyogenes ©) Paillomaviruses Anower [UBRPINSHER) otter Users Explanation Report An Error Question Explanation: The skin condition described is erysipelas, which is caused by S. pyogenes and responds well to penicillin Mark this question = => Question Td : 70285 Question 5 of 30 Tncteased risk for the transmission of HIV/ AIDS to women has been linked to all ofthe following types of contact EXCEPT 2) Unprotested intercourse with maultiole partaers +) Unprotected intercourse with one partner. «) Blood transfusion, 4) Lesbian contacts ¢) Intravenoue drug ure. Answer [NEIGIRERHNY) Other Users Explanation Report An Exot Question Explanation: Lesbian practices have not been shown to be associated with an increased risk of the transmission of HIV/AIDS, while all ofthe other activities have been associated with its spread. Mark this question = => Question Td : 70285 Question 5 of 30 Tncteased risk for the transmission of HIV/ AIDS to women has been linked to all ofthe following types of contact EXCEPT 2) Unprotested intercourse with maultiole partaers +) Unprotected intercourse with one partner. «) Blood transfusion, Y © @) Lesbian contacts ¢) Intravenoue drug ure. Answer [NEIGIRERHNY) Other Users Explanation Report An Exot Question Explanation: Lesbian practices have not been shown to be associated with an increased risk of the transmission of HIV/AIDS, while all ofthe other activities have been associated with its spread. Marke this question << => Question Td : 75853 Question 6 of 30 A 33 year-old woman develops acute diarrhea after a week's conrse of broad-spectrum antibiotics A stain for fecal leukocytes is positive, Whatis the most licely organism responsible? a) Salmonella typhi ) Shigella dysentery ¢) Campylobacter Jejuni 4) Clostridiam dificile #) Mycobacterium Tuberculosis Question Explanation: Diacthea after broad-spectrum antibiotics is associated with pseudomembranous colitis, which occurs due to the oversrowth of Clostridium difficile, The other organisms are not as commonly associcted with post-antibiotic diarrhea. Marke this question << => Question Td : 75853 Question 6 of 30 A733 yeer-old worman develops acute diarrhea after a week's conrse of broad-spectrum artibiatics A stain fer fecal leukocytes is positive. Whatis the most licely organism responsible? 2) Salmonella typhi ) Shigella dysentery ¢) Campylobacter Jejuni Y © 4) Clostridium difficile #) Mycobacterium Tuberculosis Question Explanation: Diacthea after broad-spectrum antibiotics is associated with pseudomembranous colitis, which occurs due to the oversrowth of Clostridium difficile, The other organisms are not as commonly associcted with post-antibiotic diarrhea. ‘Marke this question ¢-=> Question 7 of 30 ‘Which one of the following disease is NOT caused by a spirochetal organism? a) Lyme disease ) Yaws ) Cat-soratch dsease &) Weil's disease ¢) Relapsing fever Answer (Explanation | Other User's Explanation Report An Error Question Explanation: Cat-scratch disease is due to a coccobarillary agent. Lyne disease (A) is caused by B. burgdorferi, yaws (B) is a forma of syphilis (T. pallidum); end Weil's disease (D) is a severe form of leptospirosis. Relapsing fevers (E) are due to infection with various Borrelia spirochetes ‘Marke this question ¢-=> Question 7 of 30 ‘Which one of the following disease is NOT caused by a spirochetal organism? a) Lyme disease ) Yaws Y © ¢) Cat-scratch disease 4) Weil's diseace «) Relapsing fever Answer (Explanation | Other User's Explanation Report An Error Question Explanation: Cat-scratch disease is due to a coccobarillary agent. Lyne disease (A) is caused by B. burgdorferi, yaws (B) is a forma of syphilis (T. pallidum); end Weil's disease (D) is a severe form of leptospirosis. Relapsing fevers (E) are due to infection with various Borrelia spirochetes Marke this question & => Question Td : 81351 Question 8 of 30 Infection with Haeomphilus ducreyi causes which one of the following? 2) lyinphogranaloma venereum, ) Chaneroid ©) Syphilis ©) Bacterial vaginosis, ©) Granuioma inguinale. Question Explanation: Tnfection with Calymmatobecteriuin granulomatis causes painful beefy-red granulomarous lesions (E). Ibis a sexually transtnitted disease. H. ducreyi causes chancroid (B), which presents with a painful ulcer and secondary inguinal adenopathy. Treponema ralldum causes syphilis (C). Gardenerella spp. is a factor in bacterial vaginosis (D). Chlamydia trachomnatis subtypes L1-L3 may cause lymphogranuloma venereum (A), which may produce a vesicular lesion followed by inguinal iymphadentis and, eventually, ‘bubo inguinal n Error Report, Marke this question & => Question Td : 81351 Question 8 of 30 Infection with Haeomphilus ducreyi causes which one of the following? 2) lyinphogranaloma venereum, Y © 6) Chancroid ©) Syphilis ©) Bacterial vaginosis, ©) Granuioma inguinale. Question Explanation: Tnfection with Calymmatobecteriuin granulomatis causes painful beefy-red granulomarous lesions (E). Ibis a sexually transtnitted disease. H. ducreyi causes chancroid (B), which presents with a painful ulcer and secondary inguinal adenopathy. Treponema ralldum causes syphilis (C). Gardenerella spp. is a factor in bacterial vaginosis (D). Chlamydia trachomnatis subtypes L1-L3 may cause lymphogranuloma venereum (A), which may produce a vesicular lesion followed by inguinal iymphadentis and, eventually, Report An Error ‘bubo inguinal ‘Mark this question —& => Question Td | 86313 Question 9 of 30 A 35 year old man is brought to the emergency detartment with fever, systemic hypotension, tachycardia, and depressed consciousness, Initial examination demonstrates muitiple white plaques in hic oral cavity and on his tongue. His blood cultures are positive for aspergillus, The most important factor responsible in the pathogenesis of this patient's acute presentation is a) C3a and Ca >) Histamine ©) Interferon-y ) Prostaglandin E, 2) Tumer necrosis Factor Question Explanation The form of shock presenting in this patient who likely has ATDS is septic shock, which has a high mortality rate (approximately 50%), Sepiic shock is usually duc to sepsis from endotoxin producing, gram-negative bacteria, but mey also result from gram positive bacteria and fingl. Endotozins produced by gram-negative bacteria are lpopclysaccharides (LPS) that may reproduce all ofthe hemodynamic effects of septic shock when injected into experimental animals. Macrophages respond to LPS by producing tumor necrosis factor (TNF), which, in tum, triggers a cytokine cascade (IL-1, IL-6, IL-8, NO, and PAF) leading to anumber of potentially deleterious systemic responses. There is generalized vasodilatation resulting in acute hypotension (shock), which leads to decreased inctropism there is widespread activation of the coagulation cascade anc may cause disseminated intravascular coagulation (DIC) C3a and C5a are anaphslatoxins released in response to activation of the complement cascade. Histamine is released by mest cells in response to a vaticty of stimuli All three are released in massive amounts in anaphylactic shock triggered by an IgE-mediated type Thypersensitivty response, Histamine and enaphylacoxins cause vasodilatation and increased vascular permeability. Interferon y plays a crucial role in T lymphocyte-meciated activation of macrophages, leading to formation of epithelioid cells and multinucleated giant cells in granulomas. Prostaglandin E is an important mediator of temperature controling mechenisms in the bran increased PGE in the brain occurs as part of the acute phase responses of inflammation and results in fever: ‘Mark this question —& => Question Td | 86313 Question 9 of 30 A 35 year old man is brought to the emergency detartment with fever, systemic hypotension, tachycardia, and depressed consciousness, Initial examination demonstrates muitiple white plaques in hic oral cavity and on his tongue. His blood cultures are positive for aspergillus, The most important factor responsible in the pathogenesis of this patient's acute presentation is a) Ca and C5a ) Histamine o) Interferon-y 4) Prostaglandin E JS © c) Tumor necrosis Factor Answer | Baolanation Other User's Explanation Report An Error Question Explanation The form of shock presenting in this patient who likely has ATDS is septic shock, which has a high mortality rate (approximately 50%), Sepiic shock is usually duc to sepsis from endotoxin producing, gram-negative bacteria, but mey also result from gram positive bacteria and fingl. Endotozins produced by gram-negative bacteria are lpopclysaccharides (LPS) that may reproduce all ofthe hemodynamic effects of septic shock when injected into experimental animals. Macrophages respond to LPS by producing tumor necrosis factor (TNF), which, in tum, triggers a cytokine cascade (IL-1, IL-6, IL-8, NO, and PAF) leading to anumber of potentially deleterious systemic responses. There is generalized vasodilatation resulting in acute hypotension (shock), which leads to decreased inctropism there is widespread activation of the coagulation cascade anc may cause disseminated intravascular coagulation (DIC) C3a and C5a are anaphslatoxins released in response to activation of the complement cascade. Histamine is released by mest cells in response to a vaticty of stimuli All three are released in massive amounts in anaphylactic shock triggered by an IgE-mediated type Thypersensitivty response, Histamine and enaphylacoxins cause vasodilatation and increased vascular permeability. Interferon y plays a crucial role in T lymphocyte-meciated activation of macrophages, leading to formation of epithelioid cells and multinucleated giant cells in granulomas. Prostaglandin E is an important mediator of temperature controling mechenisms in the bran increased PGE in the brain occurs as part of the acute phase responses of inflammation and results in fever: Mark this question = => Question Td : 95392 Question 10 of 30 A 36 year old man presents to an emergency department with an exquisitely tender prostate aland. An acute bacterial infection is suspected. The most common pathogen in this setting is which of the following? a) Enterobacter ) Escherichia ©) Rlebsiella 6) Proteus ©) Peendomonas Answer [FEIRIRIRTAN) Other Users Explanation Report An Esor Question Explanation: The diseace is acute bacterial prostatic; the urual route of infection is by direct extension from a bladder or urethral cource. Less commonly, hematogenous or Iymmphatic spread from a distant site may occur. The most commonly isolated organism is Escherichia coli, other common pathogens include Klebsiella, Proteus, Pseudomonas, Enterobacter Serrata, Enterococcus, and Staphylococcus aureus, Enterobacter, Klebsiella, Proteus, and Pseudomonas are all grare-negative rode that can cause acute bacterial prostatitis, but they are not as common as E, col. Mark this question = => Question Td : 95392 Question 10 of 30 A 36 year old man presents to an emergency department with an excusitely tender prostate gland. An acute bacterial infection is suspected. The most common pathogen in this seting is which of the following? a) Enterobacter v © b) Escherichia ©) Rlebsiella 6) Proteus ©) Peendomonas Answer [FEIRIRIRTAN) Other Users Explanation Report An Esor Question Explanation: The diseace is acute bacterial prostatic; the urual route of infection is by direct extension from a bladder or urethral cource. Less commonly, hematogenous or Iymmphatic spread from a distant site may occur. The most commonly isolated organism is Escherichia coli, other common pathogens include Klebsiella, Proteus, Pseudomonas, Enterobacter Serrata, Enterococcus, and Staphylococcus aureus, Enterobacter, Klebsiella, Proteus, and Pseudomonas are all grare-negative rode that can cause acute bacterial prostatitis, but they are not as common as E, col. Mark this question & => Question 11 of 30 ‘The best method for the prevention of sesmally transmitted diseases is 2) Prophylactic antibiotics ') Spermicidal jelly uD 4) Use ofa condom ©) Antibiotics after intercourse newer [UEIRERIBNY Other Users Explanation Report An Exes Question Explanation: Question Td : 96752 Tee of condoms, a form of "safe sex," is the best choice among these live choices. The only better means of preventing STDs is abstinence, Mark this question & => Question 11 of 30 ‘The best method for the prevention of sexually transmitted diseases is a) Prophylactic antibiotics +) Spermicidal jelly 2D Y Od Use ofa condom 2) Antibiotics after intercourse newer [UEIRERIBNY Other Users Explanation Report An Exes Question Explanation: Question Td : 96752 Tee of condoms, a form of "safe sex," is the best choice among these live choices. The only better means of preventing STDs is abstinence, ‘Mark this question << => Question Td : 119189 Question 12 of 30 ATT year old mursing home resident has severe pain in the left thorax radicting around the back. Examination revealed an enaption of grouped vesicles on an erythematous base limited to a single dermatome on the left side ofthe body. A Tzanck preparation would, show which one of the following? a) Lymphocytes b) Mutinucleated giant cells ©) Hyphae 4) Eosnophis @) Dysplastic cytoplasmic cells Avewor [UEIPISNEHER) otnerUsors Explanation Report An Error Question Explanation: This patient hac herpes zoster, or shingles, which cccurt in the elderly and in immunocompromiced individuals. Itie very painful and resolves on ts own. On a special smear-Tzanck smear-taken from around the edge of the vesicle, one can see the multinucleated giant cells which are diagnostic for herpes. Lymphocytes are nonspecific and can be seen in almost any viral or inflammatory process. Hyphae are seen in fangal infection, which this, patient does not have, Bosinaphils are vieuclized with drug allergies. Dysplastic cstoplesmic cells are seen in malienaacy. ‘Mark this question << => Question Td : 119189 Question 12 of 30 ATT year old mursing home resident has severe pain in the left thorax radiating around the back. Examination revealed an eruption of grouped vesicles on an erythematous base limited to a single dermatome on the left side of the body, A Tanck preparation would show which one of the following? a) Lymphocytes Y © b) Mubinucleated giant cells ) Hyphae 4) Eosmophis ¢) Dysplastie cytoplasmic cells Avewor [UEIPISNEHER) otnerUsors Explanation Report An Error Question Explanation: This patient hac herpes zoster, or shingles, which cccurt in the elderly and in immunocompromiced individuals. Itie very painful and resolves on ts own. On a special smear-Tzanck smear-taken from around the edge of the vesicle, one can see the multinucleated giant cells which are diagnostic for herpes. Lymphocytes are nonspecific and can be seen in almost any viral or inflammatory process. Hyphae are seen in fangal infection, which this, patient does not have, Bosinaphils are vieuclized with drug allergies. Dysplastic cstoplesmic cells are seen in malienaacy. Marke this question = => Question Td : 138694 Question 13 of 30 A divorced woman aged 25 years develops acutely swollen joint and note afew non-tender papules on her arms. True statement regarding her is which one of the following? a) She recently retumed from a camping tip in Wisconsin 'b) Her boyffiend should have a wethral culture performed. c) She hes recently had a dental abscess. 4) She has recently had a flu-like illness ¢) Her PPD is likely to be postive Question Explanation: ‘The patient's symptoms are most consistent with disseminated gonocoscal infection, a sexually transmitted disease. While her boyfiend may be asymptomatic, his urethral culture is likely to be postive. A recent camping trip to Wisconsin might be associated with Lyme disease, but that would not produce a syndrome like this A dental abscess, a flu-like illness, and tuberculosis also are unlikely to produce such a pictare Marke this question = => Question Td : 138694 Question 13 of 30 A divorced woman aged 25 years develops acutely swollen joint and note afew non-tender papules on her arms. True statement regarding her is which one of the following? a) She recently retumed from a camping tip in Wisconsin Y © b) Her boyfiiend should have a urethral culture performed, c) She hes recently had a dental abscess. 4) She has recently had a flu-like illness ¢) Her PPD is likely to be postive Question Explanation: ‘The patient's symptoms are most consistent with disseminated gonocoscal infection, a sexually transmitted disease. While her boyfiend may be asymptomatic, his urethral culture is likely to be postive. A recent camping trip to Wisconsin might be associated with Lyme disease, but that would not produce a syndrome like this A dental abscess, a flu-like illness, and tuberculosis also are unlikely to produce such a pictare ‘Marke this question e& => Question Td: 141711 Question 14 of 30 A pregnant female aged 28 years develops a fluke illness with fever, headache, myalata, and back pain. She has a spontaneous abortion as a complication of illness. Examination of the abortus demonstrates severe amnionitis. The most licely mean of acquiring this infection was a) Eating soft cheese ) Handling cats ©) Rat bite 6) Sexual transmission 8) Tick bite Answer | Explanation Other User's Explanation Report An Error Question Explanation: Spontaneous abortion is a problem associated with listeriosis, caused! by Listeria monocytogenes. The pattern of abortions was first recognized in herd animals, notably sheep and cattle, and then listeriosis was later implicated as a cause of spontaneous abertion in pregnant women, Listeriosis can occur in cither epidemic (food borne or hospital acqnired) forms or may be sporadic (noticed in animal or atimal product handlers). Soft cheeses such as Brie are a particularly common source offood bome listeriosis, The disease ‘may range in severity from asymptomatic carrier cases, to fie illness, to spontanzous abortion or neoratal death, to fatal ilness in children or adults secondary to septicemia or meningsencephalits. Other localized infections can also occur primarily in the immunosuppressed, The treatment of choice is intravenous administration of ampicillin or penicillin often in combination with an aminoglycoside, Trimethoprim suamethoxazole has been used successfully in patients with penicillin allergy. Handling cats is a source of infection with Toxoplasma gondii. T. goncii may be a cause of fetal death but matemel symotoms and sions are usually very mildly symptomatic, with low lever and generalized lymphadenopathy. Infections from rat bite may be caused by Spiillum minus or Streptobacilus monliformis. These agents would not be commen causes of fetal death, Sexual transmission is the means of acquistion of cytomegalovirus, which may cfoss the placenta and cause fetal death ‘The infection in the mother usuelly presents as a mononucleosis type syndrome with low level fever and lymphadenopathy. Tick bite is the means of transmission of Lyme disease, tularemia, Rocky Mountain spotted fever, Colorado tick fever ehrlichiosis, and babesiosis. None of these agents would be expected to cause spontaneous abortion. ‘Marke this question e& => Question Td: 141711 Question 14 of 30 ‘A pregnant female aged 28 years develops a flue illness with fever, headache, myalgia, and back pain, Ske has a spontaneous abortion as a complication of illness. Examination of the abortus demonstrates severe amnionitis. The most likely mean of acquiring this infeckon was ¥ © a) Eating soft cheese b) Handling cats ©) Rat bite Sexual transmission 6) Tick bite Answer | Explanation Other User's Explanation Report An Error Question Explanation: Spontaneous abortion is a problem associated with listeriosis, caused! by Listeria monocytogenes. The pattern of abortions was first recognized in herd animals, notably sheep and cattle, and then listeriosis was later implicated as a cause of spontaneous abertion in pregnant women, Listeriosis can occur in cither epidemic (food borne or hospital acqnired) forms or may be sporadic (noticed in animal or atimal product handlers). Soft cheeses such as Brie are a particularly common source offood bome listeriosis, The disease ‘may range in severity from asymptomatic carrier cases, to fie illness, to spontanzous abortion or neoratal death, to fatal ilness in children or adults secondary to septicemia or meningsencephalits. Other localized infections can also occur primarily in the immunosuppressed, The treatment of choice is intravenous administration of ampicillin or penicillin often in combination with an aminoglycoside, Trimethoprim suamethoxazole has been used successfully in patients with penicillin allergy. Handling cats is a source of infection with Toxoplasma gondii. T. goncii may be a cause of fetal death but matemel symotoms and sions are usually very mildly symptomatic, with low lever and generalized lymphadenopathy. Infections from rat bite may be caused by Spiillum minus or Streptobacilus monliformis. These agents would not be commen causes of fetal death, Sexual transmission is the means of acquistion of cytomegalovirus, which may cfoss the placenta and cause fetal death ‘The infection in the mother usuelly presents as a mononucleosis type syndrome with low level fever and lymphadenopathy. Tick bite is the means of transmission of Lyme disease, tularemia, Rocky Mountain spotted fever, Colorado tick fever ehrlichiosis, and babesiosis. None of these agents would be expected to cause spontaneous abortion. 312 € 2014 Di www. interface. edu.pk /medical-exarns/test-analysis.pho®ubd=14759 apps EJ cocgle \ seungs Pisonin 2. ube dul © yo umilléaye Olutio ue gosoil JS [) Free Hotmall new interface.edupk/h x \ ele] * 8 Gi other bookmar ‘Matte this question e => (Question Td : 144486 Question 15 of 30 A34 year old teacher with CML receives an allogeneit bone marrow transplant ftom an HLA matched donor. He is placed on long term cyclosporine and prednisone therapy to reduce graft versus host disease. 2 months after transplantation, he develops fever, nausea, diartea, and epigastric pain Sloughed macosal cells in the urinary seciment are found (Photomicrograph below). The infectious agent is best desorbed by ee gee ae Sn a) Donible stranded DTA, enveloped, icosahedral virus b) Double stranded DIVA, naked, icosahedral virus ©) Single stranded DNA, naked, icosahedral virus 4) Double stranded RIVA, naked, icosahedral virus ¢) Single stranded negative sense RNA, enveloped, helical virus Question Explanation: This is a case of cytomegalovirus infection, which is a common problem among immunosuppressed patients, such as the one presented here. The owl's eye inclusion is a basophilic intranuclear inchision body that can be found in infected cells through cut the body and in sloughed cells in the urinary seciment CIMIV belongs to the herpes virus family and es such, is a double stranded DNA, vvimas that takes its envelope from the nuclear membrane. All DNA viruses are icosahedral Double stranded DNA naked, icosahedral virus describes papovaviruses and adenoviruses, neither of which produce the incfnsion bodies that are shown hers. Single stranded DNA naked, icosahedral virus describes parvovirus, the agent of erythema infectiosum or slapped cheek fever, a mild febrile ilness of children Double stranded RNA, naked, icosahedral virus describes the reovirus family, the major member of which is rotavirus, is a diarveal agent of infants. Single stranded negative sense RNA, enveloped, helical viruses include paramyxovirus, shabdovirus, flovirus, onthemyzovins, bunyavirus, and arenavirus; none of which create intranuclear inclusions or a symptom course like the one described here. Total Questions ts SBBARBEBBEBEE & XxXxXXXXKXXKXXKKXKXXKKXKRKKKXKKKKKKKKKK 312 € 2014 Di www. interface. edu.pk /medical-exarns/test-analysis.pho®ubd=14759 apps EJ cocgle \ seungs Pisonin 2. ube dul © yo umilléaye Olutio ue gosoil JS [) Free Hotmall new interface.edupk/h x \ ele] * 8 Gi other bookmar ‘Matte this question e => (Question Td : 144486 Question 15 of 30 A34 year old teacher with CML receives an allogeneit bone marrow transplant ftom an HLA matched donor. He is placed on long term cyclosporine and prednisone therapy to reduce graft versus host disease. 2 months after transplantation, he develops fever, nausea, diartea, and epigastric pain Sloughed macosal cells in the urinary seciment are found (Photomicrograph below). The infectious agent is best desorbed by ee gee ae Sn Y © a)Double stranded DNA, enveloped, icosahedral virus b) Double stranded DIVA, naked, icosahedral virus ©) Single stranded DNA, naked, icosahedral virus 4) Double stranded RIVA, naked, icosahedral virus ¢) Single stranded negative sense RNA, enveloped, helical virus Question Explanation: This is a case of cytomegalovirus infection, which is a common problem among immunosuppressed patients, such as the one presented here. The owl's eye inclusion is a basophilic intranuclear inchision body that can be found in infected cells through cut the body and in sloughed cells in the urinary seciment CIMIV belongs to the herpes virus family and es such, is a double stranded DNA, vvimas that takes its envelope from the nuclear membrane. All DNA viruses are icosahedral Double stranded DNA naked, icosahedral virus describes papovaviruses and adenoviruses, neither of which produce the incfnsion bodies that are shown hers. Single stranded DNA naked, icosahedral virus describes parvovirus, the agent of erythema infectiosum or slapped cheek fever, a mild febrile ilness of children Double stranded RNA, naked, icosahedral virus describes the reovirus family, the major member of which is rotavirus, is a diarveal agent of infants. Single stranded negative sense RNA, enveloped, helical viruses include paramyxovirus, shabdovirus, flovirus, onthemyzovins, bunyavirus, and arenavirus; none of which create intranuclear inclusions or a symptom course like the one described here. Total Questions ts SBBARBEBBEBEE & XxXxXXXXKXXKXXKKXKXXKKXKRKKKXKKKKKKKKKK ‘Mark thie question & => Question Td : 146727 Question 16 of 30 A 28 year old woman presents to the FE. complaining of 10-12 episodes of non-bloody diarrhea per day for the past 2 days, along, with severe abdominal cramps, nausea, vomiting, and a low-graile fever. She just retuned from a vacation to Mexico. While in ‘Mexico, she dd not drink any of the local water and ate only cooked foods and a few fresh salads. The patient should most likely be empirically treated with which one of the following if fecal leucocytes are present? @) Acyclovir ) Ciprofloxacin ©) Mebendazole 4) Quinine 6) Tetracycline Question Explanation: “When individuals travel from one country to another with marlced differences in climete and sanitation standards the risk for development of traveler's ciarthea is dramatically incteased. This type of diarrhea is most likely to develop within, 2-10 days after ingesting local water andlor eating fresh Sruts and vegetables (such as afresh salad) that could have been washed off with the local water, Traveler’s darthea is often accompanied by the appearance of approximately 10 or more episodes of diamthea per day, as wellas severe abdominal cramps, nausea, yoming, and a mild lever. Depending on the causative bacteria, blood and/or fecal leukocytes may be present Most cases of traveler's diarrhea are caused by enterotoxigeric Escherichia coli, Shigella species, and Campylobacter jejuni. The most commonly used agents to treat traveler's diarrhea are the fluoroquinolones, such as ciproflexxacin, ofloxacin, and norfloxacin, Trimethoprim/sulfamethoxazole is most commonly used in treatment of children. None of the other agents ace indicated for the treatment of traveler's diarrhea. Acyclovir is an antiviral agent indicated for the treatment of berpes virus, infections, Mebendazole is a broad spectrum anthelminthic indicated for the treatment of "worm" infections such as Enterobins vermicularis, Trichuris trichiura and Ascaris lunbricoides. Quinine is an antimalarial agent indicated for the treatment of malavia and severe leg cramps. Tetracycine is an antibacterial agent with a limited antibacterial spectrum: i is pritianly used in the treatment of acne vulgaris and gonorthea infections ‘Mark thie question & => Question Td : 146727 Question 16 of 30 A 28 year old woman presents to the FE. complaining of 10-12 episodes of non-bloody diarrhea per day for the past 2 days, along, with severe abdominal cramps, nausea, vomiting, and a low-graile fever. She just retuned from a vacation to Mexico. While in ‘Mexico, she dd not drink any of the local water and ate only cooked foods and a few fresh salads. The patient should most likely be empirically treated with which one of the following if fecal leucocytes are present? @) Acyclovir Y © +) Ciprofloxacin ©) Mebendazole 4) Quinine 6) Tetracycline Question Explanation: “When individuals travel from one country to another with marlced differences in climete and sanitation standards the risk for development of traveler's ciarthea is dramatically incteased. This type of diarrhea is most likely to develop within, 2-10 days after ingesting local water andlor eating fresh Sruts and vegetables (such as afresh salad) that could have been washed off with the local water, Traveler’s darthea is often accompanied by the appearance of approximately 10 or more episodes of diamthea per day, as wellas severe abdominal cramps, nausea, yoming, and a mild lever. Depending on the causative bacteria, blood and/or fecal leukocytes may be present Most cases of traveler's diarrhea are caused by enterotoxigeric Escherichia coli, Shigella species, and Campylobacter jejuni. The most commonly used agents to treat traveler's diarrhea are the fluoroquinolones, such as ciproflexxacin, ofloxacin, and norfloxacin, Trimethoprim/sulfamethoxazole is most commonly used in treatment of children. None of the other agents ace indicated for the treatment of traveler's diarrhea. Acyclovir is an antiviral agent indicated for the treatment of berpes virus, infections, Mebendazole is a broad spectrum anthelminthic indicated for the treatment of "worm" infections such as Enterobins vermicularis, Trichuris trichiura and Ascaris lunbricoides. Quinine is an antimalarial agent indicated for the treatment of malavia and severe leg cramps. Tetracycine is an antibacterial agent with a limited antibacterial spectrum: i is pritianly used in the treatment of acne vulgaris and gonorthea infections ‘Mark this question e& => Question Td : 146915 Question 17 of 30 4.25 year old sexually active male without HIV tisk factors presents with fever, rash, a mild headache, and a sore throat. His temperatare is 38. 20C (100 BoF); and his pulse is 9O/tnim. There is a diffuse erythematous, maculopapular rash over most of his body. Generalized adenopethy is noted. Photephobia is noted when finduscopic examination is cttemped. rhe is not treated what change in his serologic states will most likely occur? a) The FTA-ABS titer would fall ) The FTA-ABS titer would rise ©) The VDRI. titer would fall The VDRL titer would rise ¢) Both FTA-ABS and VDRL would fall Question Explanation: ‘This is a case of syphilis, which is amosed serclogically, using either treponemel or nontreoonemal tests, The FTA-ABS (@uorescent treponemal anthody-absorbed test) is the mast widely used of the specific tests and depends on fucrescent labeling of the organisms with anti-treponemal antibody. The specific tests tend te rise cauly in titer and stay (elevated throughout the lifetime of the host, or at least until well ater crug therapy has been completed. The VDEL (Venereal Disease Research Laboratory) is a nontreponemal test that detects antibodies that cross-react with mammalian cardiolini called reaginic antibodies. This test will hecome positive after the specific test, and its titer will fall late in infection, with or without drug therapy. The FTA-ABS titer would fallis incorrect because this titer will remain high throughout the lie of the host faling off only very slowly alter drug cure. Ths FTA ABS titer ould rise is incorrect because the specific antbedy test wil reach 2 high positive titer and remain at that level in untreated late stage syphilis. The VDRL titer would rise is incorrect because reaginic antody levels will fallin late stage syphilis, with or without drug treatment. Both FTA-ARS and VDRL would fal is incorrect hecause speciic antibady levels ureuld remain high, while seaginic antibodies would fall ‘Mark this question e& => Question Td : 146915 Question 17 of 30 4.25 year old sexually active male without HIV tisk factors presents with fever, rash, a mild headache, and a sore throat. His temperatare is 38. 20C (100 BoF); and his pulse is 9O/tnim. There is a diffuse erythematous, maculopapular rash over most of his body. Generalized adenopethy is noted. Photephobia is noted when finduscopic examination is cttemped. rhe is not treated what change in his serologic states will most likely occur? a) The FTA-ABS titer would fall ) The FTA-ABS titer would rise Y © 6) The VDRI titer would fall The VDRL titer would rise ¢) Both FTA-ABS and VDRL would fall Question Explanation: ‘This is a case of syphilis, which is amosed serclogically, using either treponemel or nontreoonemal tests, The FTA-ABS (@uorescent treponemal anthody-absorbed test) is the mast widely used of the specific tests and depends on fucrescent labeling of the organisms with anti-treponemal antibody. The specific tests tend te rise cauly in titer and stay (elevated throughout the lifetime of the host, or at least until well ater crug therapy has been completed. The VDEL (Venereal Disease Research Laboratory) is a nontreponemal test that detects antibodies that cross-react with mammalian cardiolini called reaginic antibodies. This test will hecome positive after the specific test, and its titer will fall late in infection, with or without drug therapy. The FTA-ABS titer would fallis incorrect because this titer will remain high throughout the lie of the host faling off only very slowly alter drug cure. Ths FTA ABS titer ould rise is incorrect because the specific antbedy test wil reach 2 high positive titer and remain at that level in untreated late stage syphilis. The VDRL titer would rise is incorrect because reaginic antody levels will fallin late stage syphilis, with or without drug treatment. Both FTA-ARS and VDRL would fal is incorrect hecause speciic antibady levels ureuld remain high, while seaginic antibodies would fall Mark this question &= => Question Td : 146935 Question 18 of 30 A 30 year old reporter returns from covering in earthquake andits aftermath in a third world country. Hie is febrile, exhausted and suffering from diminished mental acuity, so he consults a physician, who examines him, but decides not to admit him to the hospital He develops severe hemolytic anemia, puknonary edema, renal failure, and lapses into a come, The most likely cause is a) Babesia microti ) Flasmodiatn faiciparunn ©) Plasmnodintn malariae 4) Plasmodinm ovale e) Plasmodium vivax Question Explanatior Most forms of malatia cause chronic disease with significant morbidity and mortality. The most serious is malaria caused by Plasmodium falciparum (cometimes still called malignant tertian malaria), which has a propencity for being severe and having life threatening completions. P. falciparum malaria may have very severe anemia with hemoglobin less than 5 g/dL and more than half of the erythrocytes bearing parasites. Complications of this form of malaria include pulmonary edema, renal fadure with or without hhemogiobimuria, shock hyposlycemia, and cerebral malaria. Episodes occur with an imegular periodicity with fever spikes at 48-hour intervals during symptomatic perieds. Babesia microti is an intrasrythrocytio parasite that causes relatively mild, solf- limited disease called babesiosis. Ir would not be acquired outside the United States. Plasmodium malatiae causes relatively mild malaria characterized by fever spikes, with a72 hour periodicity. Plasmodum ovale and Plasmodium vivax cause malaria characterized by lever spikes with 42-hour pericdlcity. P. vives is the most common cauce of malaria accounting for 85% of all cases, but the most likely complication of infection is splenic ruptare Mark this question &= => Question Td : 146935 Question 18 of 30 A 30 year old reporter returns from covering in earthquake andits aftermath in a third world country. Hie is febrile, exhausted and ceuffering from diinished mental acuity, so he consults a physician, who examines him, but decides aot to admit him to the hospital He develops severe hemolytic anemia, pulmonary edema, renal failure, end lapses into a coms. The most likely cause is a) Babesia microti Y © ») Plasmodium falciparum ©) Plasmnodintn malariae 4) Plasmodium ovale e) Plasmodium vivax Question Explanatior Most forms of malatia cause chronic disease with significant morbidity and mortality. The most serious is malaria caused by Plasmodium falciparum (cometimes still called malignant tertian malaria), which has a propencity for being severe and having life threatening completions. P. falciparum malaria may have very severe anemia with hemoglobin less than 5 g/dL and more than half of the erythrocytes bearing parasites. Complications of this form of malaria include pulmonary edema, renal fadure with or without hhemogiobimuria, shock hyposlycemia, and cerebral malaria. Episodes occur with an imegular periodicity with fever spikes at 48-hour intervals during symptomatic perieds. Babesia microti is an intrasrythrocytio parasite that causes relatively mild, solf- limited disease called babesiosis. Ir would not be acquired outside the United States. Plasmodium malatiae causes relatively mild malaria characterized by fever spikes, with a72 hour periodicity. Plasmodum ovale and Plasmodium vivax cause malaria characterized by lever spikes with 42-hour pericdlcity. P. vives is the most common cauce of malaria accounting for 85% of all cases, but the most likely complication of infection is splenic ruptare ‘Mark this question —& => Question 19 of 30 A.66 year old man with moderate renal dysfinction presents with influenza like symptoms with ared macular rash that first appeared on the andes then spread centrally. He has nausea, vomiting, and profound restlessness. According to him he wes hiking in the mountaine, cbout a week ago. Medication that would be moct appropriate to treat this patient's infection is a) Demeclorycline +) Doxyeyoline ©) Methacyeline 9) Oxytetracyoline 6) Tetracycline Question Explanation: ‘This patient has Rocky Mountain spotted fever. This diagnosis can be confirmed with the immunohistologic demonstration of B. rickettsiae in the skin biopsy. The tetracyclines are a class of antibiotics commonly used in the treatment of rickettsial infections, acne, and various sexually transmitted diseases, is well is in the treatment of infections caused by susceptible organisms in penicillinllergic pationts, The key to this question is to knew which tetracycline antibiotic is safest t> administer to pationt with renal dysfinction Doxycyciine is secreted in an inactive form into the intestinal iamen and eliminated in the feces, therefore, its half lif is largely independent of renal or hepatic fnction. The other tetracyclines listed are concentrated by the liver in the bile and excretediin the urine and feces unchanged. Therefore, dosage adjustments need to be made in patients with renal impairment because of accumnlatioa of the drug in the body. In other words demeclocycline, methecyciine, oxytetvacycline and tetracycline will accumulate in patients with renal imp irment. ‘Mark this question —& => Question 19 of 30 A.66 year old man with moderate renal dysfinction presents with influenza like symptoms with ared macular rash that first appeared on the anikles then spread centrally. He has nausea, vontiting. and profound restlessness. According to hirn he wes hilting in the mountaine, cbout a week ago. Medication that would be moct appropriate to treat this patient's infection is a) Demeclorycline Y © &) Doxyeyeline ©) Methacyeline 9) Oxytetracyoline 6) Tetracycline Question Explanation: ‘This patient has Rocky Mountain spotted fever. This diagnosis can be confirmed with the immunohistologic demonstration of B. rickettsiae in the skin biopsy. The tetracyclines are a class of antibiotics commonly used in the treatment of rickettsial infections, acne, and various sexually transmitted diseases, is well is in the treatment of infections caused by susceptible organisms in penicillinllergic pationts, The key to this question is to knew which tetracycline antibiotic is safest t> administer to pationt with renal dysfinction Doxycyciine is secreted in an inactive form into the intestinal iamen and eliminated in the feces, therefore, its half lif is largely independent of renal or hepatic fnction. The other tetracyclines listed are concentrated by the liver in the bile and excretediin the urine and feces unchanged. Therefore, dosage adjustments need to be made in patients with renal impairment because of accumnlatioa of the drug in the body. In other words demeclocycline, methecyciine, oxytetvacycline and tetracycline will accumulate in patients with renal imp irment. Mark this question = => Question Id : 196670 Question 20 of 30 A 19 year old girl is concemed after exposure to meningococcal meningitis. Her latmate concentrated meningococcal meningitis and she now wants preventive treatment She is healthy without any past medical history. She takes Logynon as a contraceptive agent and ues a salbutamol inhaler infrequently. What would he the dmg of choice for her prophylaxis? a) Augmentin £) Ciproflozacin ©) Clasithromyein © Doxyeycline 6) Rifempicin newer [FEIRIRIRNN) O00: Users expan Question Explanation: Rifampicin is a reasonable choice as prophylaxis against meningococcal infection but in this 19 year old sexually active giel may be expected to reduce the efficacy of the oral contraceptive through liver enzyme induction, Therefore Ciproxin would be the most appropriate agent from the above list as it does not induce Cytochrome p40, jon Report An Error Mark this question = => Question Id : 196670 Question 20 of 30 A.19 year old girl is concemed after exposure to meningococcal meningitis. Her flatmate concentrated meningococcal meningitis and she now wants preventive treaiment She is healthy without any past medical history. She takes Logynon as a contraceptive agent and uses a salbutamol inhaler infrequently. What would be the drug of choice for her prophylaxis? a) Augmentin Y © &) Ciprofloxacin ©) Clarithromycin © Doxycycline ©) Bifempicin newer [FEIRIRIRNN) O00: Users expan Question Explanation: Rifampicin is a reasonable choice as prophylaxis against meningococcal infection but in this 19 year old sexually active giel may be expected to reduce the efficacy of the oral contraceptive through liver enzyme induction, Therefore Ciproxin would be the most appropriate agent from the above list as it does not induce Cytochrome p40, jon Report An Error ‘Mark this question ez Question Td : 201445 Question 21 of 30 A Thai woman aged 24 years presents with a left hemiparesis of two weeks duration, HIV antibody test was positive, CT scan of the head showed multiple ring-enhanced lesions. The most likely cause of her weakness is a) Amoebie brain abscesses b) Cerebral toxeplasmosis ©) Herpes simplex encephalitis & Primary brain lymphoma €) Progressive rmilifocal lencoencephalopathy Question Explanation: Cerebral toxoplasmosis is the most likely diagnosis. Multiple ring enhanced lesions are commonly seen in patients with cerebral toxoplasmosis, though solitary ring enhanced lesions are scen in 25% of patients on CT scan, MRI scan is more sensitive in identifying small iesions than CT scan, Amoebic brein abscesses is not the most lcely cause in this patient. Miutiple ring enhanced lesions are not seen in patients with herpes simplex encephalitis. Primary brain Iyinphoma causes a significant mass effect with surrounding oedema. Progressive rmulifocal lencoencephalopethy causes multifocal white matter lesions without any mass effect or surzouncing ordama, ‘Mark this question ez Question Td : 201445 Question 21 of 30 A Thei woman aged 24 years presents with a left hemiparesic oftwo weeks duration. HIV antibody test wae positive. CT esan of the head showed multiple ring-enkanced lesions. The mos! likely cause ofher weakness is a) Amoebie brain abscesses Y © bd) Cerebral toxoplasmosis ©) Herpes simplex encephalitis 4) Primery brain lymphoma «) Progressive multifocal leucoencephalopathy Question Explanation: Cerebral toxoplasmosis is the most likely diagnosis. Multiple ring enhanced lesions are commonly seen in patients with cerebral toxoplasmosis, though solitary ring enhanced lesions are scen in 25% of patients on CT scan, MRI scan is more sensitive in identifying small iesions than CT scan, Amoebic brein abscesses is not the most lcely cause in this patient. Miutiple ring enhanced lesions are not seen in patients with herpes simplex encephalitis. Primary brain Iyinphoma causes a significant mass effect with surrounding oedema. Progressive rmulifocal lencoencephalopethy causes multifocal white matter lesions without any mass effect or surzouncing ordama, ‘Mark this question —& => Question Id: 201475) Question 22 of 30 (Cutaneous lesion that is associated with HIV infection is which one of the following? 8) Leucoplakia ) Lichen planus ©) Lichen sclerosus ©) Plasmna cell balanitis ®) Psoriasis Question Explanatio Troceurs in 2-4% of healthy people. E-pre-existing psoriasis flares up for no apparent reason or middle-aged people develop psoriasis for the first time, one should exclude underlying HIV infection in those patients Leucoplekia is a pre-cancerous lesion wthereas oral hairy leucoplakia ie a sign of immunodeficiency in an HIV positive patient. Lichen planus, icken scleroaus and plasmaa cell balanitis are not associated with HIV infection, ‘Mark this question —& => Question Id: 201475) Question 22 of 30 ‘Cutaneous lesion that is associated with HIV infection is which one of the following? a) Leucoplakia b) Lichen plarus ©) Lichen seleroms Plasma cell balanitis ¥ © 2) Psoriasis Question Explanatio Troceurs in 2-4% of healthy people. E-pre-existing psoriasis flares up for no apparent reason or middle-aged people develop psoriasis for the first time, one should exclude underlying HIV infection in those patients Leucoplekia is a pre-cancerous lesion wthereas oral hairy leucoplakia ie a sign of immunodeficiency in an HIV positive patient. Lichen planus, icken scleroaus and plasmaa cell balanitis are not associated with HIV infection, ‘Marke this question e& => i 201505 Question 23 of 30 An HIV positive Affican woman aged 35 years has weakness of both legs and urine incontinence. CSF showed increased protein and neutrophils with normal glucose. The mostliely cause of her weakness is a) CMS polyradiculomyelopathy ) Guillan-Barre smdrome ) Hepes vires encephaltis 4) HIV encephalopathy €) Toxoplasma Question Explanation: Syinptoms are suggestive of polyradicular myelopathy (weakness of legs with involvement of sphincters), Increased neutrophils are found in CMV polyradiculomyelopathy but not in Guillain-Barre syndrome. HIV encephalopathy usually causes confusion and inemory loss. It does not involve sphincters. Gullain Barre syndrome causes polyradiculopathy. explaining all her symptoms, but with, nonnal call counts end raised protein in the CSE. Herpos sineplest encephalitis causes fever, headache, confision and deteriorating level of consciousness ‘Marke this question e& => i 201505 Question 23 of 30 An HIV positive Affican woman aged 35 years has weakness of both legs and urine incontinence. CSF showed increased protein snd neutrophils with normal glucose. The mest likely cause of her wealeness is Y © a) CMS polyradculomyelopathy ) Guillain-Barre syndrome ) Hespes virus encephaltis 4) HIV encephalopathy «) Toxoplasma Question Explanation: Syinptoms are suggestive of polyradicular myelopathy (weakness of legs with involvement of sphincters), Increased neutrophils are found in CMV polyradiculomyelopathy but not in Guillain-Barre syndrome. HIV encephalopathy usually causes confusion and inemory loss. It does not involve sphincters. Gullain Barre syndrome causes polyradiculopathy. explaining all her symptoms, but with, nonnal call counts end raised protein in the CSE. Herpos sineplest encephalitis causes fever, headache, confision and deteriorating level of consciousness ‘Marke this question & => Question Ta = 203382 Question 24 of 30 Correct statement regarding Herpes simplex encephalitis is which one of the following? a) Brain magnetic resonance imaging (MRI) is characteristicelly normal ) Cold sores or genital herpes are usually present ©) Fits are uncommon 4) Temporal lobe involvement is commen €) Viral identiication by polymerase chain reaction (PCR) on cerebrospinal duid (CSE) is non-specific Answer ( Expionaion | Other User's Explanation Report An Error Question Explanation: ‘MRI brain aonmally shows changes in the temporal lobes. Presenting features include fever, headache, vomiting, reduced, consciousness and seimures There may be dysphasia, hallucinations and peculiar behaviour. There are usvally no skin manifestations of Herpes simplex infections. The virus is rarely isclated from CSF but may be detected by PCR, ‘Marke this question & => Question Ta = 203382 Question 24 of 30 Correct statement regarding Herpes simplex: encephalitis is which one of the following? 2) Brain magnetic resonance imaging (MRI) is characteristically normal ') Cold sores or genital herpes are usually present ©) Fits are uncommon YW © 4 Temporal lobe involvement is commen €) Viral identiication by polymerase chain reaction (PCR) on cerebrospinal duid (CSE) is non-specific Answer ( Expionaion | Other User's Explanation Report An Error Question Explanation: ‘MRI brain aonmally shows changes in the temporal lobes. Presenting features include fever, headache, vomiting, reduced, consciousness and seimures There may be dysphasia, hallucinations and peculiar behaviour. There are usvally no skin manifestations of Herpes simplex infections. The virus is rarely isclated from CSF but may be detected by PCR, 3i2/2014 3:10:34 AM ‘Mark this question & => Question Td : 203432 Question 25 of 30 A 30 year old HIV positive male has Pneumocystis carini pneumonia, His blood gases reveal a PO2 of SSmamHg (75-100) whilst bbresthing 28% oxygen Whish one of the following is indicated in his treatment? a) Atovaquone ') Clindaragein c) Leucovirn 4) Pentamnidine ¢) Trimethoprim-sulphamethozazole Question Explanation: This patient has severe PCP as suggested by the hypoxia (pO2 less than 70). He should be treated with high percentage oxygen and the drug of cheice is high dose IV cotrimoxazole (trimethoprim-sulphamethcxezole). allergic to co-trimoxazcle, IV pentamidine or Clindamycin are appropriate. TV leucovirin and oral atovaquone are further options but are not first line therapies. Prednisolone has been shown to reduce mortality substantially in patients with a PO2< €0mmiTg 3i2/2014 3:10:34 AM ‘Mark this question & => Question Td : 203432 Question 25 of 30 A 30 year old HIV positive male has Pneumocystis carini pneumonia, His blood gases reveal a PO2 of SSmamHg (75-100) whilst bbresthing 28% oxygen Whish one of the following is indicated in his treatment? a) Atovaquone ') Clindaragein c) Leucovirn 4) Pentamnidine VW © ¢) Trimethoprim-sulphamethozazole Question Explanation: This patient has severe PCP as suggested by the hypoxia (pO2 less than 70). He should be treated with high percentage oxygen and the drug of cheice is high dose IV cotrimoxazole (trimethoprim-sulphamethcxezole). allergic to co-trimoxazcle, IV pentamidine or Clindamycin are appropriate. TV leucovirin and oral atovaquone are further options but are not first line therapies. Prednisolone has been shown to reduce mortality substantially in patients with a PO2< €0mmiTg Mare this question = => Question Td : 206246 Question 26 of 30 A. 36 year old man has cellulitis of his sight leg. Oa exam he is milly confused and febrile (40. 1°C) with 2 pulse is 120'min and BP 80/5SmamnHg. He is treated with intravenous Benzylpericilin and fluclozacillin, Group A streptococcus was isolated from two sets of blocd cultures. There was no significant clinical improvement after 24 hours. Which antibiotic should be added? @) Ciprofloxacin 6) Clindemscin ©) Gentamicin 6) Bifempicin ©) Vancomycin Question Explanatis The patient has a severe celuliis with features of streptococcal texic shock syndrome (TSS). Streptococcal TSS is mediated via streptococcal exotoxins “Although clindamycin is @ bacteriostatic antibiotic, it acts by switching off protein synthesis within bacterie; this in tum willlead to decreased exotoxin expression, thereby removing the mediators of TSS. Mare this question = => Question Td : 206246 Question 26 of 30 A 36 year old man has cellulitis of his right leg. On exam he is mildly confused and febrile (40.1°C) with a pulse is 120'min and BP 80/5Smnlg. He is treated with intravenous Benzylpericilin and fuclozacili. Group A streptococcus was isolated irom two sets of blocd cultures. There was no significant clinical improvement after 24 hours. Which antibiotic should be added? a) Ciprofloxacin ¥ © b) Clindemycin ¢) Gentamicin 6 Rifampicin ©) Vancomycin Question Explanatis The patient has a severe celuliis with features of streptococcal texic shock syndrome (TSS). Streptococcal TSS is mediated via streptococcal exotoxins “Although clindamycin is @ bacteriostatic antibiotic, it acts by switching off protein synthesis within bacterie; this in tum willlead to decreased exotoxin expression, thereby removing the mediators of TSS. ‘Mark this question >_> Question Td : 206346 Question 27 of 30 A woman hes a 36 hour history of herpes zoster in the T4 dermatome, She complains of severe pain in the skin supplied by T4, The most appropricte management is which one of the following? a) Aciclovir b) Carbamazepine ©) Famciclovir 4) Nothing ©) Prednisolone Answer | Explanation Other User's Explanation Report An Ertor Question Explanatio: Carbamazepine is a useful management option in the treatment of post- herpetic neuralgia Early use of stercids in herpes zoster may also reduce the amount of analgesia required and the lenath of illness. Aciclovir and famcislovir may be used to treat herpes zoster. They both reduce time ofhealing and resolution of ascociated pain, Aciclovir is now a generic medication and therefore will be cheaper than famciclovir and is the most cost effective of the treatments listed, ‘Mark this question >_> Question Td : 206346 Question 27 of 30 A woman has a 36 hour history of herpes zoster in the T4 dermatome. She complains of severe pain in the skin supplied by T4, The ‘most appropriete management is which one of the following? Y © a) Aciclovir b) Carbamazepine ) Fareciclovir 4) Nothing ©) Prednisolone Answer | Explanation Other User's Explanation Report An Ertor Question Explanatio: Carbamazepine is a useful management option in the treatment of post- herpetic neuralgia Early use of stercids in herpes zoster may also reduce the amount of analgesia required and the lenath of illness. Aciclovir and famcislovir may be used to treat herpes zoster. They both reduce time ofhealing and resolution of ascociated pain, Aciclovir is now a generic medication and therefore will be cheaper than famciclovir and is the most cost effective of the treatments listed, “Mark this question <—>=> Question 28 of 30 44.49 year old chronic alcoholic has a three day history of fever, night sweats and a cough productive of purulent sputum, There is no past history of respiratory disease. Percussion ncte is dull over the right apex and there was bronchial breathing in this area on auscutation, CXR shows right upper lobar consolidation His WBC count is 23 x109/L with newirophils 18.3 x109/L (15-7 x109). ‘The likely diagnosis is ) Aspiration pneumonia b) Klebsiella pneumonia ©) Legionella pneumonia 4) Mycoplasma pneumonia €) Primary tuberculosis Question Explanation: Community-acquired Klebsiella pneumonias a disease of debilitated middle-aged and older men with alcoholism. Mortality rates are as high as 50% regardless of treatment. Klebsiella pneumonia characteristically affects one of the upper lobes of the lung, although infection of the lower lobes is not uncommon. There is an increased tendency toward abscess formation. Aspiration pneumonia typically affects in right lower lobe in person with impaired swallowing Legionnaires’ disease is associated with contaminated air conditioning and water delivery systems. Mycoplasma pneumoniae infections have an insidious onset with malaise myalgia, sore throat and headache, Clough is characteritically dry. Chest x-ray (CXR) changes are usually patchy and involve the lower or middle lobes. “Mark this question <—>=> Question 28 of 30 44.49 year old chronic alcoholic has a three day history of fever, night sweats and a cough productive of purulent sputum, There is no past history of respiratory disease. Percussion ncte is dull over the right apex and there was bronchial breathing in this area on auscutation, CXR shows right upper lobar consolidation His WBC count is 23 x109/L with newirophils 18.3 x109/L (15-7 x109). ‘The likely diagnosis is ) Aspiration pneumonia Y © b) Rlebsiela pneumonia ©) Legionella pneumonia 4) Mycoplasma pneumonia €) Primary tuberculosis Question Explanation: Community-acquired Klebsiella pneumonias a disease of debilitated middle-aged and older men with alcoholism. Mortality rates are as high as 50% regardless of treatment. Klebsiella pneumonia characteristically affects one of the upper lobes of the lung, although infection of the lower lobes is not uncommon. There is an increased tendency toward abscess formation. Aspiration pneumonia typically affects in right lower lobe in person with impaired swallowing Legionnaires’ disease is associated with contaminated air conditioning and water delivery systems. Mycoplasma pneumoniae infections have an insidious onset with malaise myalgia, sore throat and headache, Clough is characteritically dry. Chest x-ray (CXR) changes are usually patchy and involve the lower or middle lobes. ‘Marle this question & => Question Ta : 205614 Question 29 of 30 A 19 year old homesemal male develops progressive pneumonia not responding to antbiotics. Methenamine silver staining of the sputum showed small circular cyst and Giemsa staining shows sinall punctuate nuclei of the trophozoites and intracystic Sporozoite ‘The most likely organism is a) Cryptococcus neoformans ) Leishtnania donovani ©) Pneumocystis carinii 6) Toxoplasma gordi ©) Trypanosoma crazi Question Explanation: ‘The organism is Pneumocystis carinil. The organism may be identified on microscopy after (a) methenamine silver staining for the cyst phase of the organism (b) Giemsa staining that demonstrates the smal, punctate nuclei of trophozoites and intracystic sporosoites, or (©) fiuorescence-tagged monoclonal aatibody. ‘Marle this question & => Question Ta : 205614 Question 29 of 30 A 19 year old homocesmal male develops progressive pneumonia not recponding to antibiotice. Methenamine cllver staining of the sputum showed small circular cyst and Giemsa staining shows sinall punctuate nuciei of the trophozoites and intracystic Sporozoite The most likely organism is a) Cryptococcus neoformans ) Leishtnania donovani ¥ © 0) Pneumocystis carinii 6) Toxaplatma gondii ©) Trypanosoma cruzi Question Explanation: ‘The organism is Pneumocystis carinil. The organism may be identified on microscopy after (a) methenamine silver staining for the cyst phase of the organism (b) Giemsa staining that demonstrates the smal, punctate nuclei of trophozoites and intracystic sporosoites, or (©) fiuorescence-tagged monoclonal aatibody. ‘Marke this question & Question Td: 206665 Question 30 of 30, Correct statement regerding herpes simplex: encephalitis is which one of the following? a) Is associated with a polymorphonuclear pleocytosis in the CSE b) Produces a diffuse, evenly distributed inflammation of cerebral tissues c) Produces atypical EEG pattem with lateralized periodic discharges at 2 Hr 4) Should be treated with acyclovir as soon as the diagnosis is condimed by urgent CSF viral antibody itres #) Shows a peak incidence in the auturnn Question Explanation: ‘This EEG pattern ic seen but ic not diagnostic. “Winer is the peak incidence. A lymphocytosis is characteristic in the cerebrospinal uid (CSE) ‘Terepcral lobe location is typical not diffuse Tnomediate treatment is required on clinical suspicion, ‘Marke this question & Question Td: 206665 Question 30 of 30, Correct statement regerding herpes simplex: encephalitis is which one of the following? a) Is associated with a polymorphonuclear pleocytosis in the CSE b) Produces a diffuse, evenly distributed inflammation of cerebral tissues Y © ¢) Produces atypical EEG pattem with lateralized periodic discharges at 2 Hz 4) Should be treated with acyclovir as soon as the diagnosis is condimed by urgent CSF viral antibody itres #) Shows a peak incidence in the auturnn Question Explanation: ‘This EEG pattern ic seen but ic not diagnostic. “Winer is the peak incidence. A lymphocytosis is characteristic in the cerebrospinal uid (CSE) ‘Terepcral lobe location is typical not diffuse Tnomediate treatment is required on clinical suspicion, ‘Mark this question => Question Td : 42 Question 1 of 30 “Which offthe following vimses docs not have fecel oral transmission? a) Hepatitis A ) Ebstein Barr Virus ©) Norwalk virus ) Polio virus ©) Hepatitis E Question Explanation: ‘Viruses that are transmitted via fecal oral route include: hepatitis A virus, hepatitis E virus, norwalle, polio, and numerons others. The Ebstcin Barr virus (EBV) causes mononucleosis and is spread vie person-to-person through the oropharyngeal route vie seliva and possibly spread via blood transfusion. jon Report An Error ‘Mark this question => Question Td : 42 Question 1 of 30 “Which of the following viruses doos not have fecel oral trancmission? a) Hepatitis A JV © b) Ebstein Bar Virus c) Norwalk virus ) Polio virus ©) Hepatitis E Question Explanation: ‘Viruses that are transmitted via fecal oral route include: hepatitis A virus, hepatitis E virus, norwalle, polio, and numerons others. The Ebstcin Barr virus (EBV) causes mononucleosis and is spread vie person-to-person through the oropharyngeal route vie seliva and possibly spread via blood transfusion. jon Report An Error ‘Marke this question & => Question Td : 42721 Question 2 of 30 Echoviruses have been implicated as eticlogy of which one of the following diseases? a) Guilain-Barré syncrome &) Reye's syndrome ©) Orchitis, 6) Aseptic meningitis @) Hemolytic uremic syndrome Answer [Boviataton | Other User's Explanation Report An Error Question Explanatic More than 90% of com unity-acquired cases of viral and acute aseptic meningitis are caused by group B cozsackieviruses or echoviruses. Group B coxsackievirnses or serotypes 2-5 and echovinus serotypes 4, 6, 9, 11, 16 and 30 are found most commonly. Echoviuses can also cause encephaliis vesicular rashes and herpangna ‘Marke this question & => Question Td : 42721 Question 2 of 30 Echovinses have been implicated as etiology of which one ofthe following diseases? 8) Guilain-Barré syncrome ) Reye's eyndeome ©) Orchitis Y © & Aseptic meningitis ©) Hemolytic uremic syndrome Answer [Boviataton | Other User's Explanation Report An Error Question Explanatic More than 90% of com unity-acquired cases of viral and acute aseptic meningitis are caused by group B cozsackieviruses or echoviruses. Group B coxsackievirnses or serotypes 2-5 and echovinus serotypes 4, 6, 9, 11, 16 and 30 are found most commonly. Echoviuses can also cause encephaliis vesicular rashes and herpangna ‘Mark this question & => Question Td : 48803 Question 3 of 30 An 18 year old gil was raped and she was given a prophylactic drug to prevent genotrhea and incubating syphilis. The most appropriate treatment of choice is which of the following? a) Doxyeycine ) Ceftriaened ©) Metronidazole 6) Azithromycin Question Explanation: Treatment of suspected gonorthea is with ceftriaxone, which can be given 250mg IM. Chlamydia is usualy treated with either doxycycline or azithromycin. Metroridazcle is a treatment for bacterial vaginosis and trichomonis, Syphilis is treated with penicillin usually, Report An Error ‘Mark this question & => Question Td : 48803 Question 3 of 30 An 18 year old gil was raped and she was given a prophylactic drug to prevent genotrhea and incubating syphilis. The most appropriate treatment of choice is which of the following? a) Doxyeyciine SY © b) Ceftriaxoned ©) Metronidazole 9) Acithrorayein Question Explanation: Treatment of suspected gonorthea is with ceftriaxone, which can be given 250mg IM. Chlamydia is usualy treated with either doxycycline or azithromycin. Metroridazcle is a treatment for bacterial vaginosis and trichomonis, Syphilis is treated with penicillin usually, Report An Error ‘Marke this question <= => Question Ta : 60462 Question 4 of 30 A 24-year-old premant women has been exposed to cats. Her infantis bora with hydrocephalus, cerebral calcification, and choroideetinitis. There is a "white" reflez present on findoscopis exam and ceizires occur quite offer, What is the organisa responsible for this disorder? 2) Mycoplasma pneumoniae ) Toxoplasma gondii ¢) Cytomegalovirus 6) Treponema Pallidum ©) Entameba histolytica Question Explanatic Toxoplasmosis can cause severe damage in the fetus iftrophozoites invade its reticuloendothelial system and muscles. Acute and chronic chorioretinitis can occur in the infant, resulting in blindness. Cerebral damage is common and consists of inflammatory foci with areas of necrosis thet are scattered throughout the brain, "Pscudocysts" are present within the areas of necrosis and can eventually result in calcification, ‘Thus, avoidance of cat liter and cats during pregnancy is recommended to prevent congenital toxoplasmosis infection. Mycoplasma Pneumoniae may cause outbreaks of acute respiratory disease, particularly in enclosed communities. Mycoplasmas are small, pleomorphic orgarisms which have not been documented to cause congenttal deformities or cerebral damage. Cytomegalovirus infection is common in immunocompromised indviduels. This virus can cause blindness by camage to the retina, It can also cause esophagitis and pneumonitis. Congenital transmission can be seen, but cerebral damage is rare, Amebiasis is caused by the protozoan parasite Entameba histolytica Infection is usually transtnitted by water. Diagnosis is most often made by finding cysts in the stool, Liver abscesses and invasion to the submucosa of the large bowel can also be seen. Eye Report An Error infection is rare. Syphilis is not associated to cats ‘Marke this question <= => Question Ta : 60462 Question 4 of 30 A 24-year-old premant women has been exposed to cats. Her infantis bora with hydrocephalus, cerebral calcification, and choroideetinitis. There is a "white" reflez present on findoscopis exam and ceizires occur quite offer, What is the organisa responsible for this disorder? 2) Mycoplasma pneumoniae Y © b) Toxoplasma gondii ¢) Cytomegalovirus 6) Treponema Pallidum ©) Entameba histolytica Question Explanatic Toxoplasmosis can cause severe damage in the fetus iftrophozoites invade its reticuloendothelial system and muscles. Acute and chronic chorioretinitis can occur in the infant, resulting in blindness. Cerebral damage is common and consists of inflammatory foci with areas of necrosis thet are scattered throughout the brain, "Pscudocysts" are present within the areas of necrosis and can eventually result in calcification, ‘Thus, avoidance of cat liter and cats during pregnancy is recommended to prevent congenital toxoplasmosis infection. Mycoplasma Pneumoniae may cause outbreaks of acute respiratory disease, particularly in enclosed communities. Mycoplasmas are small, pleomorphic orgarisms which have not been documented to cause congenttal deformities or cerebral damage. Cytomegalovirus infection is common in immunocompromised indviduels. This virus can cause blindness by camage to the retina, It can also cause esophagitis and pneumonitis. Congenital transmission can be seen, but cerebral damage is rare, Amebiasis is caused by the protozoan parasite Entameba histolytica Infection is usually transtnitted by water. Diagnosis is most often made by finding cysts in the stool, Liver abscesses and invasion to the submucosa of the large bowel can also be seen. Eye Report An Error infection is rare. Syphilis is not associated to cats ‘Mark this question = => Question Ti : 62190 Question 5 of 30 A 25-year-old firefighter is brought to the hospital with third-degree bums. He subsequently develops several infectons. Which one of the following represents the most likely of these infectious agents? 4) Hemophitus Infuenzac b) Neisseria Meningitides ©) Staphylococcus Aureus 6) Proteus spp. €) None of the above Anewor [UBIFIRISWAR) otner sors Explanation Report An Error Question Explanation: Infections agent sach as Pseadomonas aemiginosa, Klehisella, Escherichia coi, and Staphylococens aureus are commonly found in immunocompromised patients secondary to lesions of the skin barrier. These types of patients include patients with buene and those with penetrating trauma. Infections by Proteus spp. are often found in patients with complement deficiencies. ‘Mark this question = => Question Ti : 62190 Question 5 of 30 A 25-year-old firefighter is brought to the hospital with third-degree bums. He subsequently develops severalinfectons. Which one of the following represents the most likely of these infectious agents? 8) Hemophitus Induenzac ) Neisseria Meningitides Y © ©) Staphylococcus Aureus 6) Proteus spp. 6) None of the above Anewor [UBIFIRISWAR) otner sors Explanation Report An Error Question Explanation: Infections agent sach as Pseadomonas aemiginosa, Klehisella, Escherichia coi, and Staphylococens aureus are commonly found in immunocompromised patients secondary to lesions of the skin barrier. These types of patients include patients with buene and those with penetrating trauma. Infections by Proteus spp. are often found in patients with complement deficiencies. ‘Marke this question & => Question Ta : 64470 Question 6 of 30 Athunter develops an inflamed papule on one finger. The patient develops severe ilhiess with atypical pneumonia several days later (On physical exam his lymph nodes of the axilla of the affected arm are enlarged, Reduced breath sounds and occasional rales are heard, Splenomegaly is noted. According to him, he has handled rabbits recently. The most likely diagnosis is a) Actinomycosis b) Brucelosis ) Plague 9) Tuleremia Answer | Explanation Other User's Explanation Report An Error Question Explanation: This is tularemia, the causative organism of which is Francisela tularensis. The classic clue in test questions is exposure to rabbits, especially in the wild Tatrested cases tend to last 3 ta 4 weeks before resolving, Streptomycin is the antibiotic of choice gentamicin aad chloramphenicol can alternatively be used. Deaths occurin about 6% of untreated cases and are very rare in treated cases. ‘Marke this question & => Question Ta : 64470 Question 6 of 30 A hunter develops an inflamed papule on one finger. The patient develops severe illness with atypical pneumoria several days later. ‘On physical exam his lymph nodes of the auilla of the affected arm are enlarged. Reduced breath sounds and occasional rales are heard. Splenomegaly is noted. According to him, he has handled rabbits recently. The most likely diagnosis is a) Actinomycosis b) Erucellosis c) Plague Y © 4) Tularemia Answer | Explanation Other User's Explanation Report An Error Question Explanation: This is tularemia, the causative organism of which is Francisela tularensis. The classic clue in test questions is exposure to rabbits, especially in the wild Tatrested cases tend to last 3 ta 4 weeks before resolving, Streptomycin is the antibiotic of choice gentamicin aad chloramphenicol can alternatively be used. Deaths occurin about 6% of untreated cases and are very rare in treated cases. ‘Marke this question => Question Ti : 64643 Question 7 of 30 A 25 year old man is diagnosed with gonorrhea He is treated with ceftriaxone. A few days later he retums and states that he still has symptoms of dysuria and discharge. His urine microscopy shows no organisms. The most licely cause of his symptoms is which of the following? a) Chlamydia b) Herpes ©) Latent gonorrhea ©) Syphylis Question Explanation: Tris a standard guideline to always treat a patient with Gonorchea for both Gonerrhea and Chlamydia. Chlamydia is the most prevalent STD. Itis most common among peogle in ther late teens and early twenties and can coexist with gonomhea. Treatment options include for Gonorrhea: Ceftriaxone 125 mg IM single dose, for CHamydia Doxycycline 100 mg Bid for 7 days or Azithromycin 1 gram orally as a single dose. In patient's that are pregnant, avoid doxycycline. The patient should alzo be advised to have his sexual partner treated, ‘Marke this question => Question Ti : 64643 Question 7 of 30 A.25 year old man is diamosed with goncerhea He is treated with ceflriaxone. A few days later he retums and states that he still has symptoms of dysuria and discharge. His urine microscopy shows no organisms. The most licely cause of his symptoms is which of the following? ¥ © a) Chlamydia b) Herpes ©) Latent gonomhea © Syphylis Question Explanation: Tris a standard guideline to always treat a patient with Gonorchea for both Gonerrhea and Chlamydia. Chlamydia is the most prevalent STD. Itis most common among peogle in ther late teens and early twenties and can coexist with gonomhea. Treatment options include for Gonorrhea: Ceftriaxone 125 mg IM single dose, for CHamydia Doxycycline 100 mg Bid for 7 days or Azithromycin 1 gram orally as a single dose. In patient's that are pregnant, avoid doxycycline. The patient should alzo be advised to have his sexual partner treated, ‘Marke this question & => Question Td : 65797 Question 8 of 30 “Which of the following factors is NOT associated with prognosis in patients with hempes encephaltis? a) Age greater than 30 ') Age less than 30 c) Sex 4) Treatment with acyclovir ©) State of consciousness at initiation of therapy Anewor [JENSEN] ter usorsExplanation Report An Error Question Explanation: The patient's sexcis not associated with prognosis. Older patients do worse than younger; treatment with acyclovir improves prognosis, the beiter the paticrt’s state of consciousness at initiation of therapy, the beiter the prognosis, ‘Marke this question & => Question Td : 65797 Question 8 of 30 “Which of the following factors is NOT associated with prognosis in patients with hespes encephalitis? a) Age greater than 30 ') Age less than 30 VO Sex 4) Treatment with acyclovir ©) State of consciousness at initiation of therapy Anewor [JENSEN] ter usorsExplanation Report An Error Question Explanation: The patient's sexcis not associated with prognosis. Older patients do worse than younger; treatment with acyclovir improves prognosis, the beiter the paticrt’s state of consciousness at initiation of therapy, the beiter the prognosis, Mark this question & => Question Td : 78788 Question 9 of 30 “Which ofthe followingis an AIDS patient most likely to develop? a) Hodgkin's lymphoma ) Rheumatic heart disease ©) Pneumocystis carinii pneumonia © Systemic Lupus erythematosus ©) Hepatitis A Question Explanation: n Report An Error Peamocystis carinii pneumonia is an opportunistic infection in immunocompromised individuals, Ttis the most common infection in patients with AIDS. Indinduals usually present with shortness of breath and hypoxemia, Hodgkin's lymphoma is present in young aduits and is one of the curable cancers. Ibis not an AIDS- defining ilness. Rheumatic heart disease is common in developing countries andl is secondary to streptococcus group A. Systemic Lupus erythematosus is an autoimmune disease which can affect any organ in the body. Hepatiis A is transmitted primarily via the fecal-oral route and it is a self-limiting infection. Mark this question & => Question Td : 78788 Question 9 of 30 “Which ofthe followingis an AIDS patient most likely to develop? a) Hodgkin's lymphoma ) Rheumatic heart disease Y © © Pneumocystis carinii pneumonia © Systemic Lupus erythematosus ©) Hepatitis A Question Explanation: n Report An Error Peamocystis carinii pneumonia is an opportunistic infection in immunocompromised individuals, Ttis the most common infection in patients with AIDS. Indinduals usually present with shortness of breath and hypoxemia, Hodgkin's lymphoma is present in young aduits and is one of the curable cancers. Ibis not an AIDS- defining ilness. Rheumatic heart disease is common in developing countries andl is secondary to streptococcus group A. Systemic Lupus erythematosus is an autoimmune disease which can affect any organ in the body. Hepatiis A is transmitted primarily via the fecal-oral route and it is a self-limiting infection. Mark this question = Question Id : 90406 Question 10 of 30 A 27 year old male noticed inguinal buboes on his extemal geritalia, He visits his physician and a diagnosis of lymphogranuloma venerum is made. ‘The most likely bacteria to co-infect with the cause of the lymphogranuloma venerum is which one of the following? 2) Treaplasma wreelytcum ) Herpes simplex virus-L ) Herpes simplex virus-2 4) Neisseria gonorthea €) Neisseria meningitidis Answer | Bolanation Other User's Explanation Report An Error Question Explanation: Neisseria gonococcal infections are common co-infections associated with Chlamydia trachomatis infections, Mark this question = Question Id : 90406 Question 10 of 30 A.27 year cld male noticed inguinal buboes on his external genitalia, He visits his physician and a diagnosis of Iymmphogranulorna venerum is made. Tae most likely bacteria to co-infect with the cause of the lymphogranuloma venerum is which one of the following? 2) Treaplasma urealytcum +) Hespes simplex virus-L ) Herpes simplex virus-2 YW © 4) Neisseria gononthea ) Neisseria meningitidis Answer | Bolanation Other User's Explanation Report An Error Question Explanation: Neisseria gonococcal infections are common co-infections associated with Chlamydia trachomatis infections, ‘Marke this question <=> Question Td : 92791 Question 11 of 30 Rabies produces acute illness in approximately 30,000 persons antuelly Which of the following is FALSE concerning infection with the rabies virus? a) Neurovirulence is caused, in part, by envelope glycoproteins, which bind te acetylcholine receptors 'b) Infection in domestic animals typically represents a "spillover! from sylvatic reservoirs ofinfection, c) In the Unted States, cat rabies has become more amportant than dog rabies as a source of infection 4) Following introduction of the live virus onto a mmcous membrane or the epidermis, viral replication occurs within striated muscle cells atthe site of inoculation. €) Viremnia is essential for naturally acquired disease to occur Answer | Berianaton Other User's Explanation Report An Error Question Explanation: ‘Two forms of rabies exist: urban (domestic dogs, cats) and sylvatic (skunks, raccoons, bats, etc.). Infection in demeste animals resuits primarily as a spill over' from the sylvatic reservoirs of infection, In most areas of the wosld, the dog is the most important vector ofrabies for humans. However, in the United States, most cases now originate from cats. The initial step in infection isthe introduction of the live virus ether through the epidermis or outo a mucous membrane. Subsequently the virus replicates in the striated muscle cells at the site of the inoculation, exposing the neuromuscular andor neurotendinal spindles. Ultimately, the virus spreads centripetally up the nerve to the central nervous system. Viremaia is not thought to play a ole in acquired infections. Envelope glycoproteins are lenoblilce structures that cover the surface of the virion. Thece glycopecteins bind to acetylcholine receptors, contributing io the neurovirulence of the rabies virus. ‘Marke this question <=> Question Td : 92791 Question 11 of 30 Rabies produces acute illness in approximately 30,000 persons antuelly Which of the following is FALSE concerning infection with the rabies virus? a) Neurovirulence is caused, in part, by envelope glycoproteins, which bind te acetylcholine receptors 'b) Infection in domestic animals typically represents a "spillover! from sylvatic reservoirs ofinfection, c) In the Unted States, cat rabies has become more amportant than dog rabies as a source of infection 4) Following introduction of the live virus onto a mmcous membrane or the epidermis, viral replication occurs within striated muscle cells atthe site of inoculation. Y © 2) Viremia is essential for naturally acquired disease to occur. Answer | Berianaton Other User's Explanation Report An Error Question Explanation: ‘Two forms of rabies exist: urban (domestic dogs, cats) and sylvatic (skunks, raccoons, bats, etc.). Infection in demeste animals resuits primarily as a spill over' from the sylvatic reservoirs of infection, In most areas of the wosld, the dog is the most important vector ofrabies for humans. However, in the United States, most cases now originate from cats. The initial step in infection isthe introduction of the live virus ether through the epidermis or outo a mucous membrane. Subsequently the virus replicates in the striated muscle cells at the site of the inoculation, exposing the neuromuscular andor neurotendinal spindles. Ultimately, the virus spreads centripetally up the nerve to the central nervous system. Viremaia is not thought to play a ole in acquired infections. Envelope glycoproteins are lenoblilce structures that cover the surface of the virion. Thece glycopecteins bind to acetylcholine receptors, contributing io the neurovirulence of the rabies virus. ‘Marke this question << => Question Td : 93117 Question 12 of 30 A.28 years old woman has a CDA T lymphocyte count of 500 cells/microliter. The LEAST licely clinical finding in this patient is a) Hairy leukoplakia ) Kaposi's sarcoma ©) Vaginal candidiasis 3) CMV retinitis, 6) Tuberculosis Answer | Explanation Other User's Explanation Report An Error Question Explanatio Hairy levicoplakia (A), Kaposi's sarcoma (B), vaginal candidiasis (C), and tuberculosis (E) are seen in AIDS patients even when CD T-cell counts are as high ar 500 cellsimicroliter. However, CMV retinitis (D) is typically seen only when CD4 T-cell counts drop tp as low as 50 celishiicroiter. ‘Marke this question << => Question Td : 93117 Question 12 of 30 A.28 years old woman has a CDA T lymphocyte count af 500 cellsfnicroliter. The LEAST icely clinical finding in this patient is 2) Haiy leukoplakia t) Kaposi's carcoma ©) Vaginal candidiasis Y © & CMV retinitis 6) Tuberculosis Answer | Explanation Other User's Explanation Report An Error Question Explanatio Hairy levicoplakia (A), Kaposi's sarcoma (B), vaginal candidiasis (C), and tuberculosis (E) are seen in AIDS patients even when CD T-cell counts are as high ar 500 cellsimicroliter. However, CMV retinitis (D) is typically seen only when CD4 T-cell counts drop tp as low as 50 celishiicroiter. “Mark this question €&>=> Question Id : 96630 Question 13 of 30 A 37 year old veterinarian presents with fever, headache, abdominal pain, myalgias, and jaundice. His symptoms resolved on their ownin one week and then relapsed. He was found to have renal feilure and hepatomegaly was observed on examination, Laboratory investigations show thrombocytopenia ard an eleveted creatirine phosphokinase level. The most licely diaanosis is a) Leptospirosis +) Hepatitis A ©) Lyme disease © Listeriosis ©) Tularemia Anowor [FEIRIRIRHGNY) Other Users Explanation Report An Exar Question Explanation: Leptospirosis is an infectious disease caused by spirochetes, Infection is obtained by exposure to the urine of dogs. Aseptic meningitis, renal damage, liver failure, and fever are some of the manifestations. This discase is self limited with relapse. Diagnosis is made by a fourfold rise in agglutinating antibody titers. Treatment is doxycycline Hepatitis A can also present the same way except for the occurrence of relapse and renal failure Lyme disease is also caused by exposure to a spirochete-Borrelia burgdorferi which is transmitted by a deer tick. A rash is more common. Renal failure and liver damage is rare. Listeriosis is caused by ingestion of unpasteurized cheese and other dairy products. It can cause purulent meningitis in pregnancy and immuaocompromiced hosts ‘Tuleremia causes cutaneous ulcers and regional adenopathy. It is a zoonotic infection occurring secondary to exposure to rabbits, “Mark this question €&>=> Question Id : 96630 Question 13 of 30 A 37 year old veterinarian presents with fever, headache, abdominal pain, myalgias, and jaundice. His symptoms resolved on their ownin one week and then relapsed. He was found to have renal feilure and hepatomegaly was observed on examination, Laboratory investigations show thrombocytopenia ard an eleveted creatirine phosphokinase level. The most lice'y diaanosis is YM © & Leptospirosis +) Hepatitis A ©) Lyme disease © Listeriosis ©) Tularemia Anowor [FEIRIRIRHGNY) Other Users Explanation Report An Exar Question Explanation: Leptospirosis is an infectious disease caused by spirochetes, Infection is obtained by exposure to the urine of dogs. Aseptic meningitis, renal damage, liver failure, and fever are some of the manifestations. This discase is self limited with relapse. Diagnosis is made by a fourfold rise in agglutinating antibody titers. Treatment is doxycycline Hepatitis A can also present the same way except for the occurrence of relapse and renal failure Lyme disease is also caused by exposure to a spirochete-Borrelia burgdorferi which is transmitted by a deer tick. A rash is more common. Renal failure and liver damage is rare. Listeriosis is caused by ingestion of unpasteurized cheese and other dairy products. It can cause purulent meningitis in pregnancy and immuaocompromiced hosts ‘Tuleremia causes cutaneous ulcers and regional adenopathy. It is a zoonotic infection occurring secondary to exposure to rabbits, “Marke this question & => Question Td : 109509 Question 14 of 30 ‘The best indicator of the poorest prognosis in an HIV infected patient is which one of the following? a) Penmocysiis carini pneumonia 6) CDM countless than 100 c) Plasma HIV RMA (wiral load) ) Presence of Mycobacteria avium interacelllare ©) Age greater than 50 Answer (Bertataton Other User's Explanation Report An Error Question Explanatiot Polymerase chain reaction measurement of plasma HIV RNA (viral load) is actuelly a far more powerful prognostic indicator of mortality for AIDS than the CD4 count. PCP is the mest common epporturistic infection in HIV individuals. It alone does not predist ahigh mortality, especially with the development of prophylaxis. CD4 lymphocytes counts are usefil in detecmining the rate of progression of HIV infection, but the viral load measurement is better. MATis very debilitating, but it alone does not predict mortality Tn combination with other opportmnistic infections, an HIV inclividual does have a poor prognosis. In HIV age does not in itself predict a higher mortality. The degree of iumunosuppression and the viral load predicts the prognosis. “Marke this question & => Question Td : 109509 Question 14 of 30 ‘The best indicator of the poorest prognosis in an HIV infected patient is which one of the following? a) Pneumocystis carint paeumonia b) CD4 countless than 100 Y © c) Plasma HIV RNA (viral load) @) Presence of Mycobactena avium interacelhdare ©) Age greater than 50 Answer (Bertataton Other User's Explanation Report An Error Question Explanatiot Polymerase chain reaction measurement of plasma HIV RNA (viral load) is actuelly a far more powerful prognostic indicator of mortality for AIDS than the CD4 count. PCP is the mest common epporturistic infection in HIV individuals. It alone does not predist ahigh mortality, especially with the development of prophylaxis. CD4 lymphocytes counts are usefil in detecmining the rate of progression of HIV infection, but the viral load measurement is better. MATis very debilitating, but it alone does not predict mortality Tn combination with other opportmnistic infections, an HIV inclividual does have a poor prognosis. In HIV age does not in itself predict a higher mortality. The degree of iumunosuppression and the viral load predicts the prognosis. Mark this question = => Question Td : 113172 Question 15 of 30 A patient presents with abdominal pain, rash, and lympadenopaty. A rapid plasmin reagent (RPR) testis sent and found to be postive, The antibictic that is the treatment of choice is which one of the following? a) Vancomycin b) Tetracycline ©) Pericilin ) Exythromycin ©) Ciprofloxacin Question Explanation: ‘A postive RPR is consistent with a diagnosis of syphilis The treatment of choice is penicilin The other antibiotics are not the first line treatment Mark this question = => Question Td : 113172 Question 15 of 30 A patient presents with abdominal pain, rash, and lympadenopaty. A rapid plasmin reagent (RPR) testis sent and found to be postive, The antibictic that is the treatment of choice is which one of the following? 2) Vancomycia b) Tetracycline ¥ © &Pericilin ¢) Erythromycin ©) Ciprofloxacin Question Explanation: ‘A postive RPR is consistent with a diagnosis of syphilis The treatment of choice is penicilin The other antibiotics are not the first line treatment ‘Mark this question & => Question Id : 141260 Question 16 of 30 A.28 year old previously healthy man has severe sore throct, fever fatigue and headache for 1 week. His temperature is 39°C (102.2°F) orally. Anteriar cervical nodes are palpable and he has petectiae aver his trunk and oral mucosae but no pharyngecl esndates along with hepatosplenomegaly. A blood smear is shown below. Identity of the predomination white blood cel is on Oy CS iirrerstiyecicaleiedon a) Basophl b) CD8+ lpmphocyte ¢) CD15+ lymphocyte 4) CD21+ lymphocyte ©) Eosinophil Question Explanation: This is a case of infectious mononucleosis caused by the Epstein-Barr virus. This virus infecis B lymphocytes by attaching to the CD21 receptor, but the cells thet will predominate in a blood film will be reactive T cells which would be CD8+ T lymphocytes Basophil would be a cel that would be increased in cases of allergic response. CD16+ lymphocytes would be NK cells. which are not inducible, and therefore would not be increased in number regardless of infection. CD21+ lymphocytes are the cells infected and destroyed by the virus. Eosinophils are cells increased in allergic or parasitic responses, Report An Error ‘Mark this question & => Question Id : 141260 Question 16 of 30 A.28 year old previously healthy man has severe sore throct, fever fatigue and headache for 1 week. His temperature is 39°C (102.2°F) orally. Anteriar cervical nodes are palpable and he has petectiae aver his trunk and oral mucosae but no pharyngecl esndates along with hepatosplenomegaly. A blood smear is shown below. Identity of the predomination white blood cel is on Oy CS iirrerstiyecicaleiedon a) Basophl Y¥ © b) CD8+ lymphocyte ©) CD16+ Iymphocyte 4) CD21+ lymphocyte e) Eosinophil Question Explanation: This is a case of infectious mononucleosis caused by the Epstein-Barr virus. This virus infecis B lymphocytes by attaching to the CD21 receptor, but the cells thet will predominate in a blood film will be reactive T cells which would be CD8+ T lymphocytes Basophil would be a cel that would be increased in cases of allergic response. CD16+ lymphocytes would be NK cells. which are not inducible, and therefore would not be increased in number regardless of infection. CD21+ lymphocytes are the cells infected and destroyed by the virus. Eosinophils are cells increased in allergic or parasitic responses, Report An Error Mark thie question & => ‘Question Id : 141313 Question 17 of 30 A.55 year old man suffering from influenza deteriorates and develops high fever and shaking chills. His diagnosis of pneumonia is confirmed and a spurum sample is postive for gram positive diplococei. No known drug allergy is present. What anrbiotis is most appropriate treat hin? @) Cofotaxime £) Chloramphenicol ©) Erythromycin ©) Penicitin 6) Vancomycin Question Explanation: Penicilin remains the first line drug of choice for pneumococcal pneumonia, except in patients with penicilin allergy and in the relatively few areas in which pneumococcal strains with high level penicilin resistance exist, Alternative therapies inchide erythromycin and vancomycin in case of pericilin allergies ‘The third generation cephalosporin cefotaxime is not ususlly used for pneumococcal pneumonia, Chloramphenicel is not usually used for pneumococcal pneumonia, Erythromycin is a good alternative therapy for pneumococcal pneumonia buts usually used only when. a penicillin allergy is present Vancomycin is not the first Ine therapy, butt is a good altemative in patients allergic to penicilin or ‘when high level penicilin resistance is present. Mark thie question & => ‘Question Id : 141313 Question 17 of 30 A.55 year old man suffering from influenza deteriorates and develops high fever and shaking chills. His diagnosis of pneumonia is confirmed and a spurum sample is postive for gram positive diplococei. No known drug allergy is present. What anrbiotis is most appropriate treat hin? @) Cofotaxime £) Chloramphenicol ©) Erythromycin Y © Penicillin 6) Vancomycin Question Explanation: Penicilin remains the first line drug of choice for pneumococcal pneumonia, except in patients with penicilin allergy and in the relatively few areas in which pneumococcal strains with high level penicilin resistance exist, Alternative therapies inchide erythromycin and vancomycin in case of pericilin allergies ‘The third generation cephalosporin cefotaxime is not ususlly used for pneumococcal pneumonia, Chloramphenicel is not usually used for pneumococcal pneumonia, Erythromycin is a good alternative therapy for pneumococcal pneumonia buts usually used only when. a penicillin allergy is present Vancomycin is not the first Ine therapy, butt is a good altemative in patients allergic to penicilin or ‘when high level penicilin resistance is present. Mark this question & => Question Td : 144413 Question 18 of 30 A 37 year old teacher takes an appointment with her physicien because of. history of swollen, painful hands for 4 days. Her wrists and metacarpophalangeal joints are boggy and inflamed bilaterally, and the pain and stiffhess are worse in the morning. By the time her appointment occurs, the condition appears to be improving. The physician should ask which one of the following? a) "Do you have a familial history of autoimmune disease?" b) "Do you often feat soft cheeses and/or deli foods?" ) "Have any of your students had faciel rashes?” A) "Have you had potential exposure te ticks?" ) "Have you had a recent urinary tract or eye infection” Answer | Explanation Other User's Explanation Report An Error Question Explanation: Parvovirus B19 causes erythema infectiosum, or Fifth disease. Adults typically do not get the facial rash ("slapped cheek’) typical of children but have arthralgias and arthritis. The symmetrical distribution of involved joints is similar to that in sheurratoid arthritis. The onset is an aduit ic typically 3 te 4 weeks after exposure. Parvovirus infection: may persist in immunocupprecced patients, reculting in red blood cell aplasia, Asking about a Earilid history of autoknmune disease is appropriate if hheumatoid artis is suspected. Although the present affected joints are similar to those affected with rheumatoid arthritis, this patient is somewhat younger than the typical RA patient, Asking about eating soft cheeses andlor deli foods is appropriate if you suspect listeriosis, caused by the gram positive rod Listeria moncoytogenes. Meringitis and bacteremia are common clinical manifectations. Elderly people, neonates, pregnant women, and people taking steroids have the highest isk for infection, Asking about potential exposure to ticks is appropriate it you suspect Lyme disease, caused by Borrelia burgdorfer. Incidence is highest in the summer and fal. The white footed mouse and white tailed deer are zoonotic reservoirs. Within 3 to 32 days aber a tick bite the patient develops fever. lymphadenopathy, meringicemae, end the characterictio rach (erythema agran:). The rach enlarges and resolver over 3 to d weeks Sequelae include arturiis carditis, and neurologic abnormalities. Since this teacher noticed no symptoms other than the anthitis Lyme disease is probably not the most likely diagnosis. Asking about a recent urinary tract for eye infection is appropriate if you suspect Reiter syncrome. This is seronegatve, asymmetric arthropathy predominantly affecting the lower extremites. Itmaybe triggered by aC. trachomatis infection. In addition to the artariti, patients may have urethritis (which is uevally due to Chlamyelia) conjunctivitis mucocutaneous disease such as balanitis, oral uicerations or keratoderma. Approximately 80% of patients are HLA. B27 positive. Mark this question & => Question Td : 144413 Question 18 of 30 A 37 year old teacher takes an appointment with her physicien because of. history of swollen, painful hands for 4 days. Her wrists and metacarpophalangeal joints are boggy and inflamed bilaterally, and the pain and stiffhess are worse in the morning. By the time her appointment occurs, the condition appears to be improving. The physician should ask which one of the following? a) "Do you have a familial history of autoimmune disease?" b) "Do you often feat soft cheeses and/or deli foods?" ¥ © c) "Have any of your students had faciel rashes?” A) "Have you had potential exposure te ticks?" ) "Have you had a recent urinary tract or eye infection” Answer | Explanation Other User's Explanation Report An Error Question Explanation: Parvovirus B19 causes erythema infectiosum, or Fifth disease. Adults typically do not get the facial rash ("slapped cheek’) typical of children but have arthralgias and arthritis. The symmetrical distribution of involved joints is similar to that in sheurratoid arthritis. The onset is an aduit ic typically 3 te 4 weeks after exposure. Parvovirus infection: may persist in immunocupprecced patients, reculting in red blood cell aplasia, Asking about a Earilid history of autoknmune disease is appropriate if hheumatoid artis is suspected. Although the present affected joints are similar to those affected with rheumatoid arthritis, this patient is somewhat younger than the typical RA patient, Asking about eating soft cheeses andlor deli foods is appropriate if you suspect listeriosis, caused by the gram positive rod Listeria moncoytogenes. Meringitis and bacteremia are common clinical manifectations. Elderly people, neonates, pregnant women, and people taking steroids have the highest isk for infection, Asking about potential exposure to ticks is appropriate it you suspect Lyme disease, caused by Borrelia burgdorfer. Incidence is highest in the summer and fal. The white footed mouse and white tailed deer are zoonotic reservoirs. Within 3 to 32 days aber a tick bite the patient develops fever. lymphadenopathy, meringicemae, end the characterictio rach (erythema agran:). The rach enlarges and resolver over 3 to d weeks Sequelae include arturiis carditis, and neurologic abnormalities. Since this teacher noticed no symptoms other than the anthitis Lyme disease is probably not the most likely diagnosis. Asking about a recent urinary tract for eye infection is appropriate if you suspect Reiter syncrome. This is seronegatve, asymmetric arthropathy predominantly affecting the lower extremites. Itmaybe triggered by aC. trachomatis infection. In addition to the artariti, patients may have urethritis (which is uevally due to Chlamyelia) conjunctivitis mucocutaneous disease such as balanitis, oral uicerations or keratoderma. Approximately 80% of patients are HLA. B27 positive. Mare this question = => Question Id: 144687 Question 19 of 30 More frequently associated condition with klebsiela pneumoniae than with pseudomenas aeruginosa is which one of the following? 2) Artificial ventilation +) Cystic fibrosis ©) Diabetes mellitus 4) Green colored sputum ©) Upper lobe cavitation Question Explanation: Klebsiella pneumoniae is a well recognized cause of commurity acquired lobar pneumonia associated with cavitetion.Ttis found typically in alcoholic men over 40 years of age with underlying diabetes or obstructive lng disease, Klebsiella pneumoniae mimics Streptococcus pneumoniae as a pulmonary pathogen except that Klebsiella has a greater tendency to progress to hung abscess and empyema, Pseudomonas aeruginosa is usually associated with patients on ventilators, particularly in intensive care units Tnmunocompetent patients usually have bilateral bronchopneumonia without cavitary lesions. Artificial ventlation is classically associated with P, aeruginosa infection. The orgarism thrives in a wet envirorment, such as respirators, cleaning solutions, sinfectants, sinks, vegetables, flowers, endoscopes, and physiotherapy pools. P, aeruginosa is a yew important pathogen. Mucoid strans ofthis organism infect the airways n patents with cystic fibrosis, leading to acute exacerbations and chronic progression of fung damage. Both organisms cause disease in association with dicbetes. Klebsiella pneumoniae produces pulmonary disease and P. aeruginosa causes necrotic kin ulcers in diabetics. Green colored sputum is associated with P. aeruginosa, because more than half of the clinical isolates produce the blue green pigment pyocyanin, which is helpfil in identifying the organism, Mare this question = => Question Id: 144687 Question 19 of 30 More frequently associated condition with klebsiela pneumoniae than with pseudomenas aeruginosa is which one of the following? 2) Atiicial ventilation +) Cystic fibrosis 6) Diabetes mellitus 4) Green colored sputum SM © ©) Upper lobe cavitation Question Explanation: Klebsiella pneumoniae is a well recognized cause of commurity acquired lobar pneumonia associated with cavitetion.Ttis found typically in alcoholic men over 40 years of age with underlying diabetes or obstructive lng disease, Klebsiella pneumoniae mimics Streptococcus pneumoniae as a pulmonary pathogen except that Klebsiella has a greater tendency to progress to hung abscess and empyema, Pseudomonas aeruginosa is usually associated with patients on ventilators, particularly in intensive care units Tnmunocompetent patients usually have bilateral bronchopneumonia without cavitary lesions. Artificial ventlation is classically associated with P, aeruginosa infection. The orgarism thrives in a wet envirorment, such as respirators, cleaning solutions, sinfectants, sinks, vegetables, flowers, endoscopes, and physiotherapy pools. P, aeruginosa is a yew important pathogen. Mucoid strans ofthis organism infect the airways n patents with cystic fibrosis, leading to acute exacerbations and chronic progression of fung damage. Both organisms cause disease in association with dicbetes. Klebsiella pneumoniae produces pulmonary disease and P. aeruginosa causes necrotic kin ulcers in diabetics. Green colored sputum is associated with P. aeruginosa, because more than half of the clinical isolates produce the blue green pigment pyocyanin, which is helpfil in identifying the organism, Mark this question & => Question Td : 145561 Question 20 of 30 A.25 year old sexually active man develops epicidymtis and orchitis. Needle biopsy demonstrates a prominent leukocytic infiltrate ‘with numerous neutrophils. Infectious agent that isthe most likely cause of this man's infection is 2) Acid fast bacillus ) Gram negative coccus ©) Gram negative, oxidase negative bacillus 6) Gram negative, oxidase positive bacillus ©) Spirochete CO Question Explanation: Acute epididymitis and orchiti with prominent neutrophils in a sexmaly active male are most icely due to infection with Neisseria gonorthoeae or Chlamydia trachomatis, N. gonerhoeae can produce ancnspeoific pattom of acute inflammation (nonspecific epididymitis and orchtis) or can be sufficiently severe as to cause frank abscess within the epididymis. Ofthe listed possibilities, 1 gonorthoeae is a gram-negative coccus. Chlamydia would be described as a non-Gram stairing intracellular pathogen. Treat N. gonorthoeae with ceftriaxone ‘An acid fast bacilue describes Mycobacteriuma tuberculosis, which can cause tuberculosis ofthe epididyreis and tectec, characterized ‘by granuloma formation. A gram negetive, oxidase negative bacillus describes Escherichia coli, which is an important cause of nonspeciic epididymitis and orchitis in children with congenital genitourinary abnormalities and in older men. A gram negative, oxidase positive bacillus describes Pseudomonas so. which has been implicated as an iraportant cause of nonspecific epicidymitis and lorchitis in older men. A spirochete describes Treponema pallidum, the causative agent of syphilis which can cause testicular involvement with gumma formation, endarteritis, and/or a prominent plasma cell intrate Mark this question & => Question Td : 145561 Question 20 of 30 ‘A.25 year old semualy active man develops epididymis and orchitis. Needle biopsy demonstrates a prominent leukocytic infiltrate ‘with mimerous neutrophils. Infectious agent that the most kely cause ofthis man’s infection is 2) Acid fast bacillus ¥ © b) Gram negative coccus ©) Gram negative, oxidase negative bacillus 4) Gram negative, oxidase positive bacilus ©) Spirochete CO Question Explanation: Acute epididymitis and orchiti with prominent neutrophils in a sexmaly active male are most icely due to infection with Neisseria gonorthoeae or Chlamydia trachomatis, N. gonerhoeae can produce ancnspeoific pattom of acute inflammation (nonspecific epididymitis and orchtis) or can be sufficiently severe as to cause frank abscess within the epididymis. Ofthe listed possibilities, 1 gonorthoeae is a gram-negative coccus. Chlamydia would be described as a non-Gram stairing intracellular pathogen. Treat N. gonorthoeae with ceftriaxone ‘An acid fast bacilue describes Mycobacteriuma tuberculosis, which can cause tuberculosis ofthe epididyreis and tectec, characterized ‘by granuloma formation. A gram negetive, oxidase negative bacillus describes Escherichia coli, which is an important cause of nonspeciic epididymitis and orchitis in children with congenital genitourinary abnormalities and in older men. A gram negative, oxidase positive bacillus describes Pseudomonas so. which has been implicated as an iraportant cause of nonspecific epicidymitis and lorchitis in older men. A spirochete describes Treponema pallidum, the causative agent of syphilis which can cause testicular involvement with gumma formation, endarteritis, and/or a prominent plasma cell intrate Mark this question e& => Question Td : 149695 Question 21 of 30 A. 38 year old black male presents with enorezia, malzise, fatigue, dark urine, and upper abdominal discomfort. He admits to komocesmality, but denies bloed traneficions, alochol intake, of intravenous drug abuse. On examination he has temperature of 100.2°F, scleral icterus, and jaundice. Hepatomegaly is present, LFTs show: total bilirubin 12.4%, SGOT 980 units SGPT 120.units. Serologic findings are as follows: anti-hepatitis A IgM negative HBsAg positive anti-HBc IgM postive HBeAg positive anti-HBsAg negative hepatitis C negative. The first change in the serologic findings when this patient erters the window period is a) He will become HEc.Ag positive b) He will become HBe IgG positive ) He wil become HBeAg negative 4) He wil become HBsAb positive ) He wil become HBsAg negative Question Explanation: ‘This patient has acute hepatitis B. The "window penod” refers to that period in infection when neither hepatitis B surface angen (HBsAg) nor ts antibody (HBsAb) can be detected in the serum of the patient. Iris an immunologically me diated phenomenon caused by the precipitation of antigen antibedy complexes in their zone of equivalent concentrations and, thereby, their removal from the circulation Because of this, he first thing that will happen in the window period is thar the serum will become negative for the surface antigen (HBsAg), as that antigen is precipiteted out ofthe serum by developing levels of ts specific antbody (HBsAb) HBr Ag is not typically measured in the serum. Levels ofthe c-core antibody HBc TzG and HBeAg do not have a relationship to the window period, Levels of HBsAb willnot be detectable until there is antibody excess, and the patientis Isaving the window period. Report An Error Mark this question e& => Question Td : 149695 Question 21 of 30 A. 38 year old black male presents with enorezia, malzise, fatigue, dark urine, and upper abdominal discomfort. He admits to komocesmality, but denies bloed traneficions, alochol intake, of intravenous drug abuse. On examination he has temperature of 100.2°F, scleral icterus, and jaundice. Hepatomegaly is present, LFTs show: total bilirubin 12.4%, SGOT 980 units SGPT 120.units. Serologic findings are as follows: anti-hepatitis A IgM negative HBsAg positive anti-HBc IgM postive HBeAg positive anti-HBsAg negative hepatitis C negative. The first change in the serologic findings when this patient erters the window period is a) He will become HEc.Ag positive b) He will become HBe IgG positive ) He wil become HBeAg negative 4) He wil become HBsAb positive Y © ¢) He wil become HBsAg negative Question Explanation: ‘This patient has acute hepatitis B. The "window penod” refers to that period in infection when neither hepatitis B surface angen (HBsAg) nor ts antibody (HBsAb) can be detected in the serum of the patient. Iris an immunologically me diated phenomenon caused by the precipitation of antigen antibedy complexes in their zone of equivalent concentrations and, thereby, their removal from the circulation Because of this, he first thing that will happen in the window period is thar the serum will become negative for the surface antigen (HBsAg), as that antigen is precipiteted out ofthe serum by developing levels of ts specific antbody (HBsAb) HBr Ag is not typically measured in the serum. Levels ofthe c-core antibody HBc TzG and HBeAg do not have a relationship to the window period, Levels of HBsAb willnot be detectable until there is antibody excess, and the patientis Isaving the window period. Report An Error Mark this question & => Question Id: 150805 Question 22 of 30 A. 24 year old woman has fever weightloss, lack of appetite and periocte difficulty breathing over the past few months. An HIV test is positive. The physician decides to do a confinmetory test, What test should be ordered by the physician? 8) Enzyme-linked immmuncsorbent assay +) Fluorescence-activated cell sorting 6) Radial iraramodifiacion 6) Radioallergosorbent test 6) Western blot Question Explanation: ‘The Western blotis the most appropriate test for confirmation of HIV infection. It identifies several different antibodies against HIV. Tis an expensive and time consuming test that is quite insensitive tut the presence of pasitine ELISA. results at well ac positive “Vester blot provides extremely high diagnostic ascuracy. The test ic deemed positive ifthe pationt has atleast two antibodies against components of the HIV vision, namely gp 120, gp41 or p24. The initial HITY test tis patient had wes most lcely an enzyrne- linked immunosorbent assay (ELISA). ELISA is usually used to detect the presence of anti p24 antibody in the patient. The testis extremely senstive, but for that reason suffers from a high pescentage of false-positive resuits and must always be followed by a “Wester blet. Fluorescence activated cell sorting is a technique used to separate and count specific numbers and types of cells in a sample. An example of this would be to count the number of B cells and T cells in a specific blood sample. Radial immunodiffusion is atest sometimes used for quantitation of imnunogobulin levels in patients. This is used to determine the 1gG, IgM, and IgA levels in the patient's serum, This test cannot be used to measure levels of IgD or IgB; because these two immunoglobulins are at such low levels, a more sersitve test such 2s EIA (radioimmunoassay) or EIA. enzyme-linked immunoassay) must be used The radioallergorotbent test ic used to determine the patient's preseat level of epecific IgE that reacte with a specific allergen that has been applied to a disk or glass bead Mark this question & => Question Id: 150805 Question 22 of 30 A. 24 year old woman has fever weightloss, lack of appetite and periocte difficulty breathing over the past few months. An HIV test is positive. The physician decides to do a confinmetory test, What test should be ordered by the physician? 8) Enzyme-linked immmuncsorbent assay +) Fluorescence-activated cell sorting 6) Radial iraramodifiacion 6) Radioallergosorbent test Y © &) Westem blot Question Explanation: ‘The Western blotis the most appropriate test for confirmation of HIV infection. It identifies several different antibodies against HIV. Tis an expensive and time consuming test that is quite insensitive tut the presence of pasitine ELISA. results at well ac positive “Vester blot provides extremely high diagnostic ascuracy. The test ic deemed positive ifthe pationt has atleast two antibodies against components of the HIV vision, namely gp 120, gp41 or p24. The initial HITY test tis patient had wes most lcely an enzyrne- linked immunosorbent assay (ELISA). ELISA is usually used to detect the presence of anti p24 antibody in the patient. The testis extremely senstive, but for that reason suffers from a high pescentage of false-positive resuits and must always be followed by a “Wester blet. Fluorescence activated cell sorting is a technique used to separate and count specific numbers and types of cells in a sample. An example of this would be to count the number of B cells and T cells in a specific blood sample. Radial immunodiffusion is atest sometimes used for quantitation of imnunogobulin levels in patients. This is used to determine the 1gG, IgM, and IgA levels in the patient's serum, This test cannot be used to measure levels of IgD or IgB; because these two immunoglobulins are at such low levels, a more sersitve test such 2s EIA (radioimmunoassay) or EIA. enzyme-linked immunoassay) must be used The radioallergorotbent test ic used to determine the patient's preseat level of epecific IgE that reacte with a specific allergen that has been applied to a disk or glass bead 3i2/2014 3:28:02 AM ‘Marke this question & => Question Td : 201600 Question 23 of 30 A 22 year old Affican man has a history of fever, right sweats, shortness of breath and weight loss for 2 months. CXR shows moderately severe left pleural effusion, HIV test was positive. Tue likely cause of pleural effusion is 8) Hodgkin's lymphoma ') Pleural tuberculosis c) Pneumocystis jiroveci pneumonia (PCP) 4) Pulmonary aspergillosis ©) Pulmonary Kaposi's sarcoma Question Explanation: Pleural tuberculosis is the most licely cause in an HITV positive Aftican man with a two month history of weight loss. Hiis pleural effusion is due to pleural tuberculosis (extra pulmonary tuberculosis), Hodgkin's Iymphoma can cause pleural effusion due to pleural involvement but itis often associated with mediastinal mass. His chest radiograph only showed pleural effusion. Non-Hodgkin’s Iymnphoma (not Hodgkin's lymphoma) is commonly associated with these patients. PCP does not cause pleural effusion. It typically causes bilateral reticular shadows from the hila without any hilar lymph node enlargement or pleural effusion, Pulmonary aspergillosis shows infiltrative lesions but it does not typicelly cause pleural effusion, Pulmonary Kaposi's sarcoma can canse pleural effusion by involving the pleura, but it often causes coarse irregular nodular lesions in the lungs. 3i2/2014 3:28:02 AM ‘Marke this question & => Question Td : 201600 Question 23 of 30 A 22 year old Affican man has a history of fever, right sweats, shortness of breath and weight loss for 2 months. CXR shows moderately severe left pleural effusion, HIV test was positive. Tue likely cause of pleural effusion is 8) Hodgkin's lymphoma Y © b) Pleural tuberculosis ) Paeumnocystis jroveci pneumonia (PCP) 4) Pulmonary aspergillosis «) Pulmonary Kaposi's sarcoma Question Explanation: Pleural tuberculosis is the most licely cause in an HITV positive Aftican man with a two month history of weight loss. Hiis pleural effusion is due to pleural tuberculosis (extra pulmonary tuberculosis), Hodgkin's Iymphoma can cause pleural effusion due to pleural involvement but itis often associated with mediastinal mass. His chest radiograph only showed pleural effusion. Non-Hodgkin’s Iymnphoma (not Hodgkin's lymphoma) is commonly associated with these patients. PCP does not cause pleural effusion. It typically causes bilateral reticular shadows from the hila without any hilar lymph node enlargement or pleural effusion, Pulmonary aspergillosis shows infiltrative lesions but it does not typicelly cause pleural effusion, Pulmonary Kaposi's sarcoma can canse pleural effusion by involving the pleura, but it often causes coarse irregular nodular lesions in the lungs. ‘Mack this question & => Question Td : 203463 Question 24 of 30 A.worman aged 22 years is referred with a one week history af fever, headache and fatigue. She was a "mail ordet" bride who had receatly moved to the UK from Thailand to live with her aew kusband, Oa the basis ofher travel history, which disease can be excluded from the folowing list of differentials? 8) Cerebral toxoplasmosic +b) HIV seroconversion illness ©) Japanese B encephaltic 4) Tubertiosis ©) Yellow fever Avewor [UEQENGHEN) other Usare Explanation Repost An Erne Question Explanatio1 ‘Yellow fever only occurs in tropical South America and in sub-Saharan Aftica, Japanese B encephalitis ha: a high prevalence in southeast Asia, All ofthe other diseases listed are widespread globally. ‘Mack this question & => Question Td : 203463 Question 24 of 30 A.worman aged 22 years is referred with a one week history af fever, headache and fatigue. She was a "mail ordet" bride who had receatly moved to the UK from Thailand to live with her aew kusband, Oa the basis ofher travel history, which disease can be excluded from the folowing list of differentials? 8) Cerebral toxoplasmosic +b) HIV seroconversion illness ©) Japanese B encephaltic 4) Tubertiosis Y © e) Yellow fever Avewor [UEQENGHEN) other Usare Explanation Repost An Erne Question Explanatio1 ‘Yellow fever only occurs in tropical South America and in sub-Saharan Aftica, Japanese B encephalitis ha: a high prevalence in southeast Asia, All ofthe other diseases listed are widespread globally. 3i2/2014 3:28:30 AM ‘Mark this question & => Question Td : 206334 Question 25 of 30 An asymptomatic 23 year old HIV positive male presents to the Infectious Diseases clinic His labs show "viral load’ of 250,000 copies/ml and CD4 count of 200:109/ml (500-1600). The most appropriate treatment strategy for this paticntis which one ofthe following? a) Start antiretroviral therapy ») Start antiretrovirals when count is 150 ¢) Start antiretrovirals when count is 100 ©) Start antiretrovirals together with prophylactic antibiotic therapy +) Start prophylactic antibiotic therapy Question Explanation: Both CD4 cell counts and plasma HIV RIVA levels are independent predictors of clinical outcome. The most appropriate treatment strategy for this young man would be the introduction of anti retroviral therapy. Generaly, antiretrovrial therapy would be initiated in asymptomatic patients with a CD4 court between 200 and 250, A HIV RNA level >20,0001 copiestml merits the introcuction of antiretroviral therapy respective of CD4 count 3i2/2014 3:28:30 AM ‘Mark this question & => Question Td : 206334 Question 25 of 30 An asymptomatic 23 year old HIV positive male presents to the Infectious Diseases clinic His labs show "viral load’ of 250,000 copies/ml and CD4 count of 200:109/ml (500-1600). The most appropriate treatment strategy for this paticntis which one ofthe following? Y © a) Start antiretroviral therapy ») Start antiretrovirals when countis 150 «) Start antiretrovirals when countis 100 ©) Start antiretrovirals together with prophylactic antibiotic therapy =) Start prophylactic antibiotic therapy Question Explanation: Both CD4 cell counts and plasma HIV RIVA levels are independent predictors of clinical outcome. The most appropriate treatment strategy for this young man would be the introduction of anti retroviral therapy. Generaly, antiretrovrial therapy would be initiated in asymptomatic patients with a CD4 court between 200 and 250, A HIV RNA level >20,0001 copiestml merits the introcuction of antiretroviral therapy respective of CD4 count ‘Mark this question & => Question Td : 206492 Question 26 of 30 A.44 year old lady has a progressive, ascending motor weakness over several days. Lamber puncture is performed with normal ‘opening pressure and yields clear, colorless CSF with normal glucose, increased protein, and cell court of Simicroliter, all lymphocytes. She gracnally recovers over the nest month Which condition likely preceded her illness? a) Ketoatidosis 6) Staphylococcus aureus septicaemia ©) Systemic lupus erythemarosus ©) Viral pneumonia ©) Vitamin B12 deficiency Question Explanation: She has Guillain-Barre syndrome often preceded by an episode of infection sush as viral (cytomegalovirus [CMV] or bacterial (Campylobacter), jon Report An Error ‘Mark this question & => Question Td : 206492 Question 26 of 30 8.44 year old lady has a progressive, ascending motor weakness over several days. Lumbar puncture is performed with normal opening pressure and vields clear, colorless CSF with normal alucose, increased protein, and cell count of Sfmnicroliter, all lymphocytes. She gradually recovers over the aext month Which condition likely preceded her illness? a) Ketoacidosis +) Staphylococens aurens septicaemia c) Systemic lupus erythematosus Y © 6 Viral pneumonia e) Vitamin B12 deficiency Question Explanation: She has Guillain-Barre syndrome often preceded by an episode of infection sush as viral (cytomegalovirus [CMV] or bacterial (Campylobacter), jon Report An Error <& => Question Td id Question 27 of 30 Mark this question A 19 year old boy has a urethral discharge one week after having casual unprotected sexx Gram stain shows numerous neutrophils, come cf whish contain gram negative intracellular diplococci. He is treated with Cefiratone, 250 mg at an intramuscular injection. 5 days later, he re-altends with persisting discharge. The most likely cause of this discharge is which one of the following? a) Chlamydia trachomatis b) Fenicillin-resistant Neisseria gonorrhoea c) Re-infection with Neisseria gonorrhoea o) Ureaplasma urealyticum e) Urethral stricture Answer (Birisnaton) Other User's Explanation Report An Error Question Explanatio1 ‘This patient has been adequately treated for gonorrhoea and a persistent discharge would be unusual unless, as is often the case, there is a co-infection. The patent is likely to have a non-specific urethritis cue to Chlamydlia trachomatis, recpiring treatment with ether dozycycline or erythromycin for 7-14 days <& => Question Td id Question 27 of 30 Mark this question A 19 year old boy has a urethral discharge one week after having casual unprotected sexx Gram stain shows numerous neutrophils, come cf whish contain gram negative intracellular diplococci. He is treated with Cefiratone, 250 mg at an intramuscular injection. 5 days later, he re-altends with persisting discharge. The most likely cause of this discharge is which one of the following? ¥ © a) Chlamydia trachomatis b) Fenicillin-resistant Neisseria gonorthoca ¢) Re-infection with Neisseria gonomhoea oD) Ureaplasma wealyticun ) Urethral stricture Answer (Birisnaton) Other User's Explanation Report An Error Question Explanatio1 ‘This patient has been adequately treated for gonorrhoea and a persistent discharge would be unusual unless, as is often the case, there is a co-infection. The patent is likely to have a non-specific urethritis cue to Chlamydlia trachomatis, recpiring treatment with ether dozycycline or erythromycin for 7-14 days ‘Mark this question —& => Question Id : 212688 Question 28 of 30 An 84 year old diabetic woman, on dialysis, develops blateral orbital and facial pain, headache, and sudden loss of vision in the sight ee. The right eyelids are red and swollen and she has complete loss of vision and motion on the right eye The mucosa of the nasal passages is swollen and acorotic, with a black discoloration. MRI shows soft ticsue swelling ofthe nasal mucosa, simusos, and orbital tissues. Temporal atery biopsy is negative for giant cell artertis. Biopsy of the nasal and sinuses mucosa demonstrates thrombosed vessels and multiple broad aonseptate hyphae with right angle branches. The most appropriate treatment is 8) Amphotericin B and surgical debridement 6) Corticosteroids ) Heparin anticoagulation 4) Metronidazole and drainage of the paranasal sinuses ¢) Surgical decompression of both orbits Ancwer [UEARNBNEH) oer Ue. Question Explanation: The chnical pictwe is that of rhino orbital mucormycosis, a devastating fungal infection that is seen in debiltated patients and in Gabetics The biopsy ofthe nasal mucosa confirms the clitical diagnosis. Because, mortality is very high, the appropriate treatraent is amphoterion B end surgcal debridement, Corticosteroids (B) would have been the correct treatment for giant cell temporal arteritis. Anticoagulation (C) will not reverse the small vessel thrombosis that is typical of this disease. The fimai cannot be treated with metronidazole, and the infection will not subside with sins drainage (D), Surgical decompression (B) is the correct treatment for retro-orbital cellulitis ‘Explanation Report An Error ‘Mark this question —& => Question Id : 212688 Question 28 of 30 An 84 year old diabetic woman, on dialysis, develops blateral orbital and facial pain, headache, and sudden loss of vision in the sight eye. The right eyetids are red and swollen and she has complete loss of vision and motion on the right eye. The mucosa of the nasal passages is swollen and acorotic, with a black discoloration. MRI shows soft ticsue swelling ofthe nasal mucosa, simusos, and orbital tissues. Temporal atery biopsy is negative for giant cell artertis. Biopsy of the nasal and sinuses mucosa demonstrates thrombosed vessels and multiple broad aonseptate hyphae with right angle branches. The most appropriate treatment is Y © 2) Amphotericin B and surgical debridement 6) Corticosteroids ) Heparin anticoagulation 4) Metronidazole and drainage of the paranasal sinuses ¢) Surgical decompression of both orbits Ancwer [UEARNBNEH) oer Ue. Question Explanation: The chnical pictwe is that of rhino orbital mucormycosis, a devastating fungal infection that is seen in debiltated patients and in Gabetics The biopsy ofthe nasal mucosa confirms the clitical diagnosis. Because, mortality is very high, the appropriate treatraent is amphoterion B end surgcal debridement, Corticosteroids (B) would have been the correct treatment for giant cell temporal arteritis. Anticoagulation (C) will not reverse the small vessel thrombosis that is typical of this disease. The fimai cannot be treated with metronidazole, and the infection will not subside with sins drainage (D), Surgical decompression (B) is the correct treatment for retro-orbital cellulitis ‘Explanation Report An Error Mare this question ee => Question Id : 216922 | Question 29 of 30 A 29-year-old man has been relatively healthy all hs life. He had asthma up to the age of 6 years, but no problems since then. At age 11 years, he had appendectomy. He denies any fever, chils, weightloss, etc. Examination and investigations are normal. A PPD skin test at 48 hours, is 11 mam in diameter. What ic the interpretation? 8) A PPD test cannot be interpreted in symptomatic patients 6) A PPD testis always equivecal ifthe patient does not have a cough ©) APPD testis always positive ifthe chest radiograph is normal & Equivocal ©) Negative f) Positive Question Explanation: Tn patients who have no tisk factors for tuberculosis, the cutoff for a positive PPD testis 15 mm. A finding of 11 mm is considered negative. In patients who have risk factors, the cutoff is lower. The cutoffis 10 mm in high-tisk patients (IV drug users, people living in poverty immigrants from high TB areas, healthcare workers, prisoners, nursing home residents, homeless people, and patients who have debilitating disease). The cutoff is 5 mm in very high-risk patients (HIV-postive, close contacts of active TB patients, chronic steroid use, immunocompromised patients, or organ transplant recipients). A FPD test can be interpreted! in asymptomatic patients. A PPD test can be either positive or negative in patients who are asymptomatic (.e., those who have no cough) or those who have a nottnal chest radiograph, A positive PPD signifies latent infection and is not an indicator of active TB. Tn this patient who has no risk Factors, only a PPD of 15 mm or more would be considered positive, therefore, equivocal and positive ace incorrect. Mare this question ee => Question Id : 216922 | Question 29 of 30 A 29-year-old man has been relatively healthy all hs life. He had asthma up to the age of 6 years, but no problems since then. At age 11 years, he had appendectomy. He denies any fever, chils, weightloss, etc. Examination and investigations are normal. A PPD skin test at 48 hours, is 11 mam in diameter. What ic the interpretation? 8) A PPD test cannot be interpreted in symptomatic patients 6) A PPD testis always equivecal ifthe patient does not have a cough ©) APPD testis always positive ifthe chest radiograph is normal & Equivocal M © & Negate f) Positive Question Explanation: Tn patients who have no tisk factors for tuberculosis, the cutoff for a positive PPD testis 15 mm. A finding of 11 mm is considered negative. In patients who have risk factors, the cutoff is lower. The cutoffis 10 mm in high-tisk patients (IV drug users, people living in poverty immigrants from high TB areas, healthcare workers, prisoners, nursing home residents, homeless people, and patients who have debilitating disease). The cutoff is 5 mm in very high-risk patients (HIV-postive, close contacts of active TB patients, chronic steroid use, immunocompromised patients, or organ transplant recipients). A FPD test can be interpreted! in asymptomatic patients. A PPD test can be either positive or negative in patients who are asymptomatic (.e., those who have no cough) or those who have a nottnal chest radiograph, A positive PPD signifies latent infection and is not an indicator of active TB. Tn this patient who has no risk Factors, only a PPD of 15 mm or more would be considered positive, therefore, equivocal and positive ace incorrect. Merk this question & Question Td : 217097 Question 30 of 30 A 25-year-old man has fever of 38.7 °C (101.6 °F) with chills and tenderness over the liver. No organomegaly is noted, Ubrasound reveals an abscess in the tight lobe, ALP and aminotransferases are elevated. What is the most appropriate next step? a) Begin therapy with levofloxacin b) Begin therapy with metronidazole c) Begin therapy with praziquantel A) Percutancous drainage of abscess ) Surgical drainage and injection of ethyl alcohol Answer | Birionaton | Other User's Explanation Report An Error Question Explanation: This patient has an amebic abscess of the liver. Itis commonly seen in men who have "a Mexico connection” or in homosesnal men whe have a high frequency of intestinal carriage of Entamoeba histolytica, the ameba responsible for this abscess Presentation and imaging diagnosis are similar to pyogenic liver abscess, however, treatment is done with metronidazole and seldom requires dranage. Definitive diagnosis is byway of serology and not through culturing the organism in the pus (one cannot grow the ameba from the pus). Therefore. do not drain the abscess and await culture results Levofloxacin ic not used in the treatment of amebie abscess. Praziquantel is used to treat Schistosomiasis, Hydatid cyst of the iver is caused by infestation with the tapeworm echinococcus. These cysts have fluid that is under pressure and needling may cause rupture with anaphylazis or intrapertoneal seeding ‘Therefore, these cysts are treated surgically with drainage of the fluid andl injection with ethyl alcohol or 20% steuile saline ifthe Hid is clear. Merk this question & Question Td : 217097 Question 30 of 30 A 25-year-old man has fever of 38.7 °C (101.6 °F) with chills and tenderness over the liver. No organomegaly is noted, Ubrasound eveale an abscess in the right lobe, ALP and aminotransferaces are elevated. What is the most appropriate next step? a) Begin therapy with levofloxacin JV © b) Begin therapy with metronidazole c) Begin therapy with praziquantel ) Percutaneous drainage of abscess e) Surgical drainage and injection of ethyl alcohol Answer | Birionaton | Other User's Explanation Report An Error Question Explanation: This patient has an amebic abscess of the liver. Itis commonly seen in men who have "a Mexico connection” or in homosesnal men whe have a high frequency of intestinal carriage of Entamoeba histolytica, the ameba responsible for this abscess Presentation and imaging diagnosis are similar to pyogenic liver abscess, however, treatment is done with metronidazole and seldom requires dranage. Definitive diagnosis is byway of serology and not through culturing the organism in the pus (one cannot grow the ameba from the pus). Therefore. do not drain the abscess and await culture results Levofloxacin ic not used in the treatment of amebie abscess. Praziquantel is used to treat Schistosomiasis, Hydatid cyst of the iver is caused by infestation with the tapeworm echinococcus. These cysts have fluid that is under pressure and needling may cause rupture with anaphylazis or intrapertoneal seeding ‘Therefore, these cysts are treated surgically with drainage of the fluid andl injection with ethyl alcohol or 20% steuile saline ifthe Hid is clear. 2115, ‘Marke this question => Question Td Question 1 of 30 The group at greatest risk for developing HIV/AIDS in the United States today is a) The elderly. 'b) Neonates ©) Recent immigrants 4) Individuals with congenitally abnonnal immune systems, ©) Adolescents Avewor (RESINGHR) other Users Explanation Report An srr Question Explanatio1 ‘Studies demonstrate that teenagers are least licely to practice "safe sex" or be informed about the modes of infection with HIV. None ofthe other groups are expected to be at any unusual risk for acquiring this infection, 2115, ‘Marke this question => Question Td Question 1 of 30 The group at greatest risk for developing HIV/AIDS in the United States today is a) The elderly. 'b) Neonates ©) Recent immigrants 4) Individuals with congenitally abnonnal immune systems, Y © 6) Adolescents Avewor (RESINGHR) other Users Explanation Report An srr Question Explanatio1 ‘Studies demonstrate that teenagers are least licely to practice "safe sex" or be informed about the modes of infection with HIV. None ofthe other groups are expected to be at any unusual risk for acquiring this infection, '3i2/2014 4:25:20 AM ‘Marke this question & => Question Ta : 62137 Question2 of 30 A 25-year-old male presents to the medical clinic with a symmetric erythematous rash localized to the grcin, scrotum, and penis. His pregnant wife is concemed that the rash may be dangerous for the pregnancy. She has no current complaints of rash, The most likely cause of the patient's rash is 8) Gonomhea. ) Contact dermatitis, ©) Psoriasis 4) Asymptomatic yeast infection in his wie €) Infestation Question Explanation: ‘Vaginal candiciasis is extremely common in pregnancy and may be asymptomatic, yet be transmitted to the sesual partner and cause atypical genitel rash The symptoms are not characteristic of gonorthea, psoriasis, cr infestation. Without history af exposure, there is no basis for the diagnosis of contact dermattis '3i2/2014 4:25:20 AM ‘Marke this question & => Question Ta : 62137 Question2 of 30 A 25-year-old male presents to the medical clinic with a symmetric erythematoue rash localized to the grein, ecrotum, and penis. His pregnant wife is concemed that the rash may be dangerous for the pregnancy. She has no current complaints ofrash, The most likely cause of the patient's rash is 8) Gonomhea. ) Contact dermatitis, ©) Fsoriasis SY © & Asymptomatic yeast infection in his wile. 6) Infestation Question Explanation: ‘Vaginal candiciasis is extremely common in pregnancy and may be asymptomatic, yet be transmitted to the sesual partner and cause atypical genitel rash The symptoms are not characteristic of gonorthea, psoriasis, cr infestation. Without history af exposure, there is no basis for the diagnosis of contact dermattis Mark this question & => Question Ti : 78650 Question 3 of 30 Cheracteristic cerebrospinal fuid findings in cryptococcal meningitis include all ofthe following EXCEPT @) A reduced CSF ghicose level ) An elevated CSE protein level ¢) A postive cryptococcal antibody test. 9) An increased umber of lymphocytes. ©) A postive India ink test Question Explanation: There is no commonly performed test for cryptococcal antibody in the CSF. There is a latex agglutination test for csyptococcel antigen which is second only to culture in sensitivity and specificity for the diagnosis of cryptococcal meningitis. Typical biochemical changes of the spinal fiid inciude an elevated protein and a modestly reduced ghicose level. Most patients (excent for some patients with AIDS) wil have a lymphocytic pleocytosis. The India ini testis less sensitive than cryptococcal antigen testing, but is very helpful when performed by a laboratory experienced in the interpretation of the test n Report An Error Mark this question & => Question Ti : 78650 Question 3 of 30 Cheracteristic cerebrospinal fluid findings in cryptococcal meningitis include all of the following EXCEPT 2) A reduced CSF ghucose level. b) An elevated CSF protein level. ¥ © ©) A postive cryptococcal antibody test. ¢) Anincreased number of lymphocytes, €) A postive India ink test Question Explanation: There is no commonly performed test for cryptococcal antibody in the CSF. There is a latex agglutination test for csyptococcel antigen which is second only to culture in sensitivity and specificity for the diagnosis of cryptococcal meningitis. Typical biochemical changes of the spinal fiid inciude an elevated protein and a modestly reduced ghicose level. Most patients (excent for some patients with AIDS) wil have a lymphocytic pleocytosis. The India ini testis less sensitive than cryptococcal antigen testing, but is very helpful when performed by a laboratory experienced in the interpretation of the test n Report An Error ‘Mark this question & => Question Td : 78661 Question 4 of 30 A 36-year-old AIDS patient presents with fevers up to 104°E, anemia, anorema, and weightloss over the course of three weeks. His alkaline phosphatase levelis elevated and he is dehydrated from diasrhea, What is the most licely diagnosis? a) Pneumocystic carinii pneumonia ) Disseminated mycobacterium avium complex c) Cytomegelovirus infection 4) Mycobacteria tberculosis c) Lymphoma Anowor (UEXQNGRER) ctner User's Explon Question Explanation: This patient presents with classic symptoms of disseminated MAT infection, which hes a very poor prognosis. Elevated temperatures, retroperitoneal adenopathy, hilar adenopathy, poor appetite, and a T-cell count below 100 in an immunocompromised patient are common, Pneumocystis carinii pneumonia would present with puimonery symptoms initially, not diarthea and elevated alkaline phosphatase level Cytomegalovinis infection can present as a pneumonia, colitis, and retinitis. Tt is also common in transplant recipients. M. tuberculosis can also present intially with dissemination in an AIDS patient, but the isolated elevation of alkaline phosphatase is umusval, An elevated LDH level would be present in lymphoma as well as CIS manifestations Report An Error ‘Mark this question & => Question Td : 78661 Question 4 of 30 A. 36-year-cld AIDS patient presents with fevers up to 104°F, anemia, anorenta, and weightloss over the course of three weeks. His alkaline phosphatase levelis elevated and he is dehydrated from diasrhea, What is the most licely diagnosis? a) Pneumocystic carinii pneumonia ¥ © b) Disseminated mycobacterium avium complex c) Cytomegalovirus infection 4) Mycobacteria tberculosis ¢) Lymphoma Anowor (UEXQNGRER) ctner User's Explon Question Explanation: This patient presents with classic symptoms of disseminated MAT infection, which hes a very poor prognosis. Elevated temperatures, retroperitoneal adenopathy, hilar adenopathy, poor appetite, and a T-cell count below 100 in an immunocompromised patient are common, Pneumocystis carinii pneumonia would present with puimonery symptoms initially, not diarthea and elevated alkaline phosphatase level Cytomegalovinis infection can present as a pneumonia, colitis, and retinitis. Tt is also common in transplant recipients. M. tuberculosis can also present intially with dissemination in an AIDS patient, but the isolated elevation of alkaline phosphatase is umusval, An elevated LDH level would be present in lymphoma as well as CIS manifestations Report An Error ‘Mark this question => Question Td : 81096 Question 5 of 30 A.21 year old sexually active woman presents with fever, lower cbdominal pain, and vaginal discharge. On pysical examination, she hhad cervical motion tendemess and bilateral adnexal tendemess. Lab data reveals an elevated white count and ESR. Her physician suspects pelvic inflammatory disease. Which of the following is NOT a complication of this disease? a) Infetilty. ') tibo-overian abscess. 6) Ectopic pregnancy. 6) Perforation of the bowel 6) Periadnexal adhesions Question Explanatic This patient has pelvic inflammatory disease which is caused mainly by Chlamydia trachomatis and Neisseria gonortheae. Itis characterized by vasious conditions, including cervicitis, encomnetnitis, and parametric. Aclvanced cases can result in pesitoriti, The ovaries tend to resist infection, but sometimes become involved. Tubal infertility (A) resubs from chronic scar tissue thet results in blockage ofthe fallopian tubes from the chronic, non-treated pelvic inflammatory disease. Chlamydia trachomatis infection (B) is the ‘most common cause of pelvic inflammatory disease. An ectopic pregnancy (C) that hemorrhages and perforates can result in death. A person is at an increased risk for an ectopic pregnancy if she has scarred fallopian tubes from pelvic inflammatory disease. Periadnexal achesions (B) can result in chronic pelvic pain or lower abdominal pain, These adhesions can be removed by laparoscopy. ‘Mark this question => Question Td : 81096 Question 5 of 30 A.21 year old sexually active woman presents with fever, lower cbdominal pain, and vaginal discharge. On pysical examination, she hhad cervical motion tendemess and bilateral adnexal tendemess. Lab data reveals an elevated white count and ESR. Her physician suspects pelvic inflammatory disease. Which of the following is NOT a complication of this disease? a) Infetilty. ') tibo-overian abscess. 6) Ectopic pregnancy. Y © 6) Perforation of the bowel 6) Periadnexal adhesions Question Explanatic This patient has pelvic inflammatory disease which is caused mainly by Chlamydia trachomatis and Neisseria gonortheae. Itis characterized by vasious conditions, including cervicitis, encomnetnitis, and parametric. Aclvanced cases can result in pesitoriti, The ovaries tend to resist infection, but sometimes become involved. Tubal infertility (A) resubs from chronic scar tissue thet results in blockage ofthe fallopian tubes from the chronic, non-treated pelvic inflammatory disease. Chlamydia trachomatis infection (B) is the ‘most common cause of pelvic inflammatory disease. An ectopic pregnancy (C) that hemorrhages and perforates can result in death. A person is at an increased risk for an ectopic pregnancy if she has scarred fallopian tubes from pelvic inflammatory disease. Periadnexal achesions (B) can result in chronic pelvic pain or lower abdominal pain, These adhesions can be removed by laparoscopy. ‘Marke this question & => Question Ta : 81442 Question 6 of 30 “Which of the following is caused by infection with Gardenerella spp.? 2) lymphogranuloma venereuin, ) Chancroid. ¢) Syphilis. 6) Bacterial vaginosis, ©) Granloma ingninale newer [UENSIHEN) ner Users Explanation Report An Error Question Explanation: Infection with Calymmatsbacterium grarulomatis cautes painfal beefy-red granulometous lesions ), Ih ic a sexually transmitted disease. H. ducreyi causes chancroid (B), which presents with a painful ulcer and secondary inguinal adenopathy Treponema pallidum causes eyphils (C). Gardenerella spp. is a factor in bacterial vaginosis (D). Chlamydia trachomatis subtypes L1-L3 may cause lymphogranuloma venereum (A), which may produce a vesicular lesion fellowed by inguinal lymphadenitis and, eventually, ‘bubo inguinale, ‘Marke this question & => Question Ta : 81442 Question 6 of 30 “hich ofthe following is caused by infection with Gardenerela tpp.? 2) lymphogranuloma venereum. ) Chancesid. ¢) Syphilis. oY © & Bacterial vaginosis. ©) Gramuloma inguinale newer [UENSIHEN) ner Users Explanation Report An Error Question Explanation: Infection with Calymmatsbacterium grarulomatis cautes painfal beefy-red granulometous lesions ), Ih ic a sexually transmitted disease. H. ducreyi causes chancroid (B), which presents with a painful ulcer and secondary inguinal adenopathy Treponema pallidum causes eyphils (C). Gardenerella spp. is a factor in bacterial vaginosis (D). Chlamydia trachomatis subtypes L1-L3 may cause lymphogranuloma venereum (A), which may produce a vesicular lesion fellowed by inguinal lymphadenitis and, eventually, ‘bubo inguinale, ‘Mark this question = => Question Td : 94192 Question 7 of 30 A 24 year old man exhibits a syncrome offever, headache, myalgia, and aching joints 3 days after a summer camping trip. He then develops a generalized maculopapular eruption thet initially involves the palms and soles and then becomes hemorthagic. Splencmegaly is present, A protracted 3 week course of the illness occurs in spite of antibiotic therapy and is followed by gradual resolution of the ilness. The mest iely diagnosis is a) Lyme arthritis +) Syctemic inpuc erythematocus in exacerbation ©) Rocky Mountain spotted fever 4) Subacute immune complex disease ®) Gonococcemia Question Explanation: Fever, headache, arid myalgia must be recognized as the classic trad of Rocky Mustain spotted fever in all patients who might have ‘been bitten by a tick, as commonly occurs in cempers. This patient exhibits a classic picture ofthe diseace, which often has a virulent 3 week course even when treated with tetracycline or chloramphenicol. A 10% mortality rate has been noted for this ilness. Lyme disease is also the resut of tick transmission, but is otherwise urlike Rocky Mountain spotted fiver. Gonococremia is associated with nomocrticuale pain and a popular rach; eystemic lupus erythematocus and immune complex disease do not produce the clinical picture seen in this patient ‘Mark this question = => Question Td : 94192 Question 7 of 30 A 24 year old man exhibits a syndrome of fever, headache, myalgia, and aching joints 3 days after a summer camping trip. He then develops a generalized maculopapular eruption that initially invoives the pakns and soles and then becomes hemocthagic Splenomegaly is present. A protracted 3 week course of the illness occurs in spite of antibiotic therapy and is followed by gradual resolution of the illness. The most lileely diagnosis is 2) Lyme arthritis b) Systemic lupus erythematosus in exacerbation ¥ © c) Rocky Mountain spotted fever Subacute immune complex disease ®) Gonococcemia Question Explanation: Fever, headache, arid myalgia must be recognized as the classic trad of Rocky Mustain spotted fever in all patients who might have ‘been bitten by a tick, as commonly occurs in cempers. This patient exhibits a classic picture ofthe diseace, which often has a virulent 3 week course even when treated with tetracycline or chloramphenicol. A 10% mortality rate has been noted for this ilness. Lyme disease is also the resut of tick transmission, but is otherwise urlike Rocky Mountain spotted fiver. Gonococremia is associated with nomocrticuale pain and a popular rach; eystemic lupus erythematocus and immune complex disease do not produce the clinical picture seen in this patient ‘Mark this question & => Question Td : 94514 Question 8 of 30 A.37 year oldman comes to you because he has experienced 6 weeks of gradvally increasing fevers, dry cough and shortness of breath, He has lost 5.4 kg (12 1b) since his last routine visit 6 months ago. On physical examination, there are several large, nontender, anterior and posterior cervical lymph nodes. His medical history inchudes infections monomucleosis, hepetits B, impetigo, prostatitis and tinea versicolor. Inhis history which factor is most suggestive of pneumonia related to a sexually transmitted disease? a) Impetigo b) Hepaiitis B ©) Infectious mononucleosis a) Prostatitis ¢) Tinea versicolor Answer | Explanation | Other User's Explanation _Report An Error Question Explanation: ‘This man's past medical history includes multiple prior infections, inchidng mononucleosis, hepatitis B, prostattis, and tinea versicolor. Furthermore, he has had a subacute illness with weightloss, fevers, and a nonprocuctive cough. The history of hepatitis B and prostatitis in a young man with these symptoms should always suggest the possibiity of HIV infection transmitted through homosexual activity. Of the listed infections, hepaitis B is the infection most likely to be transmitted sexually ‘Mark this question & => Question Td : 94514 Question 8 of 30 A.37 year oldman comes to you because he has experienced 6 weeks of gradvally increasing fevers, dry cough and shortness of breath, He has lost 5.4 kg (12 1b) since his last routine visit 6 months ago. On physical examination, there are several large, nontender, anterior and posterior cervical lymph nodes. His medical history inchudes infections monomucleosis, hepetits B, impetigo, prostatitis and tinea versicolor. Inhis history which factor is most suggestive of pneumonia related to a sexually transmitted disease? a) Impetigo v © b) Hepatitis B ©) Infectious mononucleosis a) Prostatitis ¢) Tinea versicolor Answer | Explanation | Other User's Explanation _Report An Error Question Explanation: ‘This man's past medical history includes multiple prior infections, inchidng mononucleosis, hepatitis B, prostattis, and tinea versicolor. Furthermore, he has had a subacute illness with weightloss, fevers, and a nonprocuctive cough. The history of hepatitis B and prostatitis in a young man with these symptoms should always suggest the possibiity of HIV infection transmitted through homosexual activity. Of the listed infections, hepaitis B is the infection most likely to be transmitted sexually Tae ths question @&c=> [Question Ta 97466 |] Question9 of 30 A person was bit in the hand by another person, He presents to the emergency room four hours after the bite. The wound is one om Jong, the wound edges have tninimnal erythema, and there is no purulent drainage. What should be the treatment of choice? a) Oral penicilin, b) Broad-spectrum IV antibiotics. c) Oral penicilin and wound closure 4) Broad-spectrum IV antibiotics and wound closure ¢) Wound closure. Anower [JERENEERI) otnor User's Exp Question Explanation: Human bites can be severe injuries secondary to the virulence of human mouth flora, principally Viconella, Treatment should consist of irigation and broad spectrum IV antibotes. The wound should be left open. Wound closure increases the chences of a severe irfection developing, as the wound is mable to drain. Oral antiiotics are usually aot sufficient for the treatment of human bites, athough they may be effective against animal bites. jon Report An Error Tae ths question @&c=> [Question Ta 97466 |] Question9 of 30 A person was bitin the hand by another person, He presents to the emergency room four hours after the bite, The wound ie one om Jeng, the wound edges have minimal erythema, and there is no purulent drainage. What should be the treztinent of choice? a) Oral penicilin, WV © b) Broad-spectrum IV antibiotics o) Oral penicilin and wound closure. 4) Broad-spectrum IV antibiotics and wound closure. ¢) Wound closure. Anower [JERENEERI) otnor User's Exp Question Explanation: Human bites can be severe injuries secondary to the virulence of human mouth flora, principally Viconella, Treatment should consist of irigation and broad spectrum IV antibotes. The wound should be left open. Wound closure increases the chences of a severe irfection developing, as the wound is mable to drain. Oral antiiotics are usually aot sufficient for the treatment of human bites, athough they may be effective against animal bites. jon Report An Error Mark this question & => Question Td ; 103156 Question 10 of 30 ‘The most appropriate treatment for a 46 year old man with Staphylococcus aureus associated osteomyelitis is 2) Gentamicin ') Methictin ©) Penicillin 4) Clindamycin ©) Streptomycin Anower (UBINRESIEH) otner veers Explanation Report An Error Question Explanation: Clindamaycia is usefal for the treatment of chronic osteomyeliie becauce it can achiave levels in bone that are 99% of those in serum. ‘The other bacteriocidal antibiotics cannot. Mark this question & => Question Td ; 103156 Question 10 of 30 ‘The most appropriate treatment for a 46 year old man with Staphylococcus aureus associated osteomyelitis is 2) Gentamicin ') Methictin ©) Penicillin Y © 4) Clindamycin ©) Streptomycin Anower (UBINRESIEH) otner veers Explanation Report An Error Question Explanation: Clindamaycia is usefal for the treatment of chronic osteomyeliie becauce it can achiave levels in bone that are 99% of those in serum. ‘The other bacteriocidal antibiotics cannot. ‘Mark this question & => Question Td : 109530 Question 11 of 30 ‘The most likely cell ype infected by the agent sesponsible for the majority of cases of heterophil positive infectious mononucleosis is which one of the following? 8) CD4 Ttymphocyte 6) CD8 T-lymphocyte ©) Macrephage 6) Neutrophil ©) E lymphocyte Question Explanati Epstein Barr virus or EBY is the ikely causative agent in the majority of cases of heterophil positive infectious mononucleosis. It infects B cells. T lymphocytes which attack infected B cells are inhibited by a soluble factor (vIL-I0} thatis encoded by the EBV genome, ‘Mark this question & => Question Td : 109530 Question 11 of 30 ‘The most likely cell ype infected by the agent sesponsible for the majority of cases of heterophil positive infectious mononucleosis is which one of the following? 8) CD4 Ttymphocyte 6) CD8 T-lymphocyte ©) Macrephage 6) Neutrophil S @ ©) B lymphocyte Question Explanati Epstein Barr virus or EBY is the ikely causative agent in the majority of cases of heterophil positive infectious mononucleosis. It infects B cells. T lymphocytes which attack infected B cells are inhibited by a soluble factor (vIL-I0} thatis encoded by the EBV genome, ‘Mark this question —& => Question Td : 120139 Question 12 of 30 A 25 year old man who is HIV positive develops a space occupying lesion. The most frequent space occupying lesion in patients with AIDS is which one of the following? a) CNS lymphoma ) Toxoplasmosis Cryptoccal meningitis Kaposi's sarcoma °) d ©) Non Hodgkin's lymphoma Anowor [JEXQINGHEN) other Users Explanation Report An Eror Question Explanation: ‘Toxoplasmosis represeats the most frequent space-occupying lesion in patients with AIDS. Ibis followed in frequency by non Hodgkin's lymphoma, which is the second most frequently diagnosed space-occupying lesion. ‘Mark this question & => Question Td : 120139 Question 12 of 30 ‘A.25 year old man who is HIV positive develops a space occupying lesion. The most Eequeat space occupying lesion in patients with AIDS is which one of the following? a) CNS lymphoma ¥ © b) Toxoplasmosis, Cryptoccal meningitis Kaposi's sarcoma °) d ©) Non Hodgkin's lymphoma Anowor [JEXQINGHEN) other Users Explanation Report An Eror Question Explanation: ‘Toxoplasmosis represeats the most frequent space-occupying lesion in patients with AIDS. Ibis followed in frequency by non Hodgkin's lymphoma, which is the second most frequently diagnosed space-occupying lesion. ‘Marle this question <=> Question 13 of 30 A 22 year old studenthas a 1 week history of mild lower abdominal pain that is dull in nature relieved slightly by acetaminophen and worsened by intercourse. Pelvic examination seveals a red, swollen cervix without motion tendemess, The taucosa is friable. KOH mountis negative, and wet mourt does notreveal clue cells. Gram stain of the exudate reveals gram negative cocci, The most important feature in the pathogenesis of this onanism is a) Beta hemoyysin ) Bxotostin ©) IgA protease 4) Intracellular habitat ©) Pil Answer (Explanation | Other User's Explanation Report An Error Question Explanatio ‘The symptoms suggest infection with Neisseria gonorthoea or Chlamydia trachomatis. Of the two, Neisseria is the only one that would produce a neutrophilic discharge The fastest and most reliable assay specific For diagnosis of gonorrhea is the use of commercial DNA probes, with results available in 2 to + hours. The most impostant mechanism of pathogenesis for Neisseria gonorrhoeae is the possession of pil, In this infection the pil promote adherence to the mucosa and protection from phagocytosis, and undergo entigenic variation, Beta hemelysins are produced by Streptococcus pyogenes. S. agalactiae urotropic Escherichia coli, and many other organisms, but are net invelved in Neisserial pathogenesis, Exotozins are produced by a variety of pathogens, but none of these is an agent of cervicits and pelvic inflammatory disease. IgA protease is produced by both Neisseria gonomhoeae and N. meningitides. Although it is an important mechanism of pathogenesis, allowing colonization of the mucosa, itis not the most important feature, Intracellular habitat is not an attnbute of Neisseria, These are extracellular organisms, which are in the process of being killed when they are found packed into neutrophils ‘Marle this question <=> Question 13 of 30 A 22 year old studenthas a 1 week history of mild lower abdominal pain that is dull in nature relieved slightly by acetaminophen and worsened by intercourse. Pelvic examination seveals a red, swollen cervix without motion tendemess, The taucosa is friable. KOH mountis negative, and wet mourt does notreveal clue cells. Gram stain of the exudate reveals gram negative cocci, The most important feature in the pathogenesis of this onanism is a) Beta hemoyysin ) Bxotostin ©) IgA protease 4) Intracellular habitat YO Pit Answer (Explanation | Other User's Explanation Report An Error Question Explanatio ‘The symptoms suggest infection with Neisseria gonorthoea or Chlamydia trachomatis. Of the two, Neisseria is the only one that would produce a neutrophilic discharge The fastest and most reliable assay specific For diagnosis of gonorrhea is the use of commercial DNA probes, with results available in 2 to + hours. The most impostant mechanism of pathogenesis for Neisseria gonorrhoeae is the possession of pil, In this infection the pil promote adherence to the mucosa and protection from phagocytosis, and undergo entigenic variation, Beta hemelysins are produced by Streptococcus pyogenes. S. agalactiae urotropic Escherichia coli, and many other organisms, but are net invelved in Neisserial pathogenesis, Exotozins are produced by a variety of pathogens, but none of these is an agent of cervicits and pelvic inflammatory disease. IgA protease is produced by both Neisseria gonomhoeae and N. meningitides. Although it is an important mechanism of pathogenesis, allowing colonization of the mucosa, itis not the most important feature, Intracellular habitat is not an attnbute of Neisseria, These are extracellular organisms, which are in the process of being killed when they are found packed into neutrophils Mark this question ==> Question 1d: 144423 Question 14 of 30 A Brazilian immigrant undergoes an employment physical exemmation. CXR shows a patchy, bilateral pneumonia and a hing mass, and he is referred to a specialist. Biopsy shows fungal orgenisms with a few mubiple budding yeast forms. The most lely diagnosis is a) Blestomycosis ) Coctidioi¢omycosis ©) Histoplasmosis 4) Paracoccidioidomycosis ©) Sporotrichosis Anower [MERISRSRRNY other Users Explanation Roport An Exar Question Explanation: Latin America and "pilot's wheel” budding yeast are clues for paracoccidioidomycosis. This disease is caused by Paracoccidicides brasiliensis, a dimorphic fungus that is found es a multiple budding yeastin tissues. Blastomycosis results in large budding yeasts in the sputum. [tis found in the great river beds of the eastem half of the United Stetes, with North and South Carolina being especialy tighly endemic. It causes acute and chronic lung infections as well as disseminated disease and is a special problem in the immmocompromised. Coccidioidomycosis is a fingus of the U.S. desert southwest. Ttalso causes acute and chronic pulmonary infections, disseminated infections, and cutaneous lesions, but the tissue form would be spherules with endospores. Hisioplasmosis is endemic along the Mississippi, Missouri, and Ohio river valleys of the United States. It causes acute and chronic pulmonary infections and disseminated infections in the immunocompromised, The tissue form is intracellular, unencapsulated yeast. Sporctrichosis causes skin lesions in rose gardeners that may be single cutaneous mycetomas or follow lymphatic drainages. The tissue form is cigar shaped yeast Mark this question ==> Question 1d: 144423 Question 14 of 30 ‘A Brazillian immigrant undergoes aa employment physical exammnatioa, CXR shows a patchy, bilateral pneumonia and a hing mass, and he is referred to a specialist. Biopsy shows fungal orgenisms with a few mubiple budding yeast forms. The most lely diagnosis is a) Blastomycosis b) Coccidioidomycosis ©) Histoplasmosis Y © 4) Paracoccidioidemycosis ¢) Sporotrichosis Anower [MERISRSRRNY other Users Explanation Roport An Exar Question Explanation: Latin America and "pilot's wheel” budding yeast are clues for paracoccidioidomycosis. This disease is caused by Paracoccidicides brasiliensis, a dimorphic fungus that is found es a multiple budding yeastin tissues. Blastomycosis results in large budding yeasts in the sputum. [tis found in the great river beds of the eastem half of the United Stetes, with North and South Carolina being especialy tighly endemic. It causes acute and chronic lung infections as well as disseminated disease and is a special problem in the immmocompromised. Coccidioidomycosis is a fingus of the U.S. desert southwest. Ttalso causes acute and chronic pulmonary infections, disseminated infections, and cutaneous lesions, but the tissue form would be spherules with endospores. Hisioplasmosis is endemic along the Mississippi, Missouri, and Ohio river valleys of the United States. It causes acute and chronic pulmonary infections and disseminated infections in the immunocompromised, The tissue form is intracellular, unencapsulated yeast. Sporctrichosis causes skin lesions in rose gardeners that may be single cutaneous mycetomas or follow lymphatic drainages. The tissue form is cigar shaped yeast ‘Mark this question & => Question Ta : 145479 Question 15 of 30 A 35 vear oldman returns after a trip to Calcutta He presents with malaise, fatigue, and loss of appetite. 3 months after initial agnosis, the serological test results reveal: HBs.tb, positive; HBeAb, positive, HBeAg, negative, HBsAg, negate, HBsAb, negative. The absence of IBsAg and HBsAb form the serum of this patient indicates which of the following? 2) The patient has been suczessfully vaccinated against Hepatitis B ) The patientis ancrgic to Hepatitis B ) The patientis immune to Hepatitis B 4) The patient is making an excess of HBsAb e) The patient is making an excess of HBsAg ) The patient is making equivalent amounts of HBsAb and HBsAg Avower [FEQIRNSNY) otter veorsExplanation Report An Error Question Explanation: ‘The patient has clearly retumed ftom Calcutta with an active infection of Hepatiti B, since his serum is positive for antibody against the core antigen of Hepatitis B. This is the standard diagnostic for this infection. The absence of HBsAg and HBsAb from the serum reflects that the patient has entered the "window petiod," a good prognostic indicator, because it demonstrates that antibodies to the surface antigen are being made in enough concentration to precipitate the corresponding antigen out of the circulation. This is the in vive demonstration ofthe equivalence zone in the antibodyfantigen precipitation titration, Successful vaccination against Hepatitis B would be reflected by a positive titer of HBsAb only. The presence of antibody against the c core antigen reflects the existence of an active infection. Aneray to Hepattis B is aot correct, because a person who is anergic is selectively unresponsive or tolerant to the antigen in question. Clearly, the presence of antibodies to, several of these viral antigens in the serum demonstrates that the person is not anergic. “Although the person is beginning to moutt a protective immune response (autibody 10 EBsAg) it has not yet reached a protective level Fhe patient were making en excess of HBsAb. here would be antigen excess. Thus the patient would be positive for HBsAb and negative for HBsAg. Ifthe patent were making an excess of HBsAg the patient would be positive for HEs/Ag and negative for HBsAb ‘Mark this question & => Question Ta : 145479 Question 15 of 30 A 35 vear oldman returns after a trip to Calcutta He presents with malaise, fatigue, and loss of appetite. 3 months after initial agnosis, the serological test reeulte reveal: HBsAb, postive; HBe.Ab, positive, HBeAg, negative; HBsAg, aegative, EBsAb, negative. The absence of HBsAg and HBsAb form the serum of this patient indicates which of the following? 2) The patient has been suczessfully vaccinated against Hepatitis B ') The patientis anergic to Hepatitis B ) The patientis immune to Hepatitis B <) The patients making an excess of HEsAb e) The patient is making an excess of HBsAg ¥ © 8 The patient is making equivalent amounts of HBsAb and HBsAg Avower [UEQIRNNY) otrerveors Explanation Report An Error Question Explanation: ‘The patient has clearly retumed ftom Calcutta with an active infection of Hepatiti B, since his serum is positive for antibody against the core antigen of Hepatitis B. This is the standard diagnostic for this infection. The absence of HBsAg and HBsAb from the serum reflects that the patient has entered the "window petiod," a good prognostic indicator, because it demonstrates that antibodies to the surface antigen are being made in enough concentration to precipitate the corresponding antigen out of the circulation. This is the in vive demonstration ofthe equivalence zone in the antibodyfantigen precipitation titration, Successful vaccination against Hepatitis B would be reflected by a positive titer of HBsAb only. The presence of antibody against the c core antigen reflects the existence of an active infection. Aneray to Hepattis B is aot correct, because a person who is anergic is selectively unresponsive or tolerant to the antigen in question. Clearly, the presence of antibodies to, several of these viral antigens in the serum demonstrates that the person is not anergic. “Although the person is beginning to moutt a protective immune response (autibody 10 EBsAg) it has not yet reached a protective level Fhe patient were making en excess of HBsAb. here would be antigen excess. Thus the patient would be positive for HBsAb and negative for HBsAg. Ifthe patent were making an excess of HBsAg the patient would be positive for HEs/Ag and negative for HBsAb Mark this question & => Question Ta : 146768 Question 16 of 30 A-medical student is taking an international health elective in Mexico, He develops profise watery diarrhea, estimating thet he is losing 5 liters of uid a day. He is severely dehydrated and is transported to a local hospital The diarthea continues even though the patient is not cating, Stool analysis reveals no blood and few leukocytes. Flesks of mucus are fouad in the sample. What tozin has the most simiat mechanism of action to the toxin produced by this organism? 4) Alpha toxin of Staphglococeus aureus +) Diphtheria toxin ©) Bxotoxin A of Pseudomonas aeruginosa 4) Labile toxin of Escherichia coli e) Pertussis toxin Answer (Explanation) Other User's Explanation Report An Error Question Explanation: This is cholera, caused by Vibrio cholerae. The voluminous "rice- wetet" stools described in the question result from the action of the cholera enterotoxin. A Chelera enterotoxin acts by activating adenylate cyclase by ADP rbosyiation of GTP-binding protein in small intestinal crypt cells, turring on massive secretion of isotonic fluid. A Enterotonigenic. Escherichia coli produce a toxin very similar te choleta toxin, the heat-lebile toxin. The alpta toxin of Staphylococcus aureus is a pore forming toxin that punches holes in cellular membranes. Diphthena toxin acts by nibosylating elongation factor-2 and thereby mbbiting eukarvotic protein synthesis. Is binding component targets heart. nerve. and epithelial cells Exotoxin A of Pseudomones aeruginosa acts very similar to the diphtheria toxin, inhibiting elongation factor-2, but targeting liver cells, Pertussis tovin causes edema by ADP sibosylaiing Gi, the inhibitory subunit of G protein, thus increasing cyclic AMP. Mark this question & => Question Ta : 146768 Question 16 of 30 A-medical student is taking an international health elective in Mexico, He develops profise watery diarrhea, estimating thet he is losing 5 liters of uid a day. He is severely dehydrated and is transported to a local hospital The diarthea continues even though the patient is not cating, Stool analysis reveals no blood and few leukocytes. Flesks of mucus are fouad in the sample. What tozin has the most simiat mechanism of action to the toxin produced by this organism? 4) Alpha toxin of Staphglococeus aureus +) Diphtheria toxin ©) Bxotoxin A of Pseudomonas aeruginosa Y © 4) Labile toxin of Escherichia coli e) Pertussis toxin Answer (Explanation) Other User's Explanation Report An Error Question Explanation: This is cholera, caused by Vibrio cholerae. The voluminous "rice- wetet" stools described in the question result from the action of the cholera enterotoxin. A Chelera enterotoxin acts by activating adenylate cyclase by ADP rbosyiation of GTP-binding protein in small intestinal crypt cells, turring on massive secretion of isotonic fluid. A Enterotonigenic. Escherichia coli produce a toxin very similar te choleta toxin, the heat-lebile toxin. The alpta toxin of Staphylococcus aureus is a pore forming toxin that punches holes in cellular membranes. Diphthena toxin acts by nibosylating elongation factor-2 and thereby mbbiting eukarvotic protein synthesis. Is binding component targets heart. nerve. and epithelial cells Exotoxin A of Pseudomones aeruginosa acts very similar to the diphtheria toxin, inhibiting elongation factor-2, but targeting liver cells, Pertussis tovin causes edema by ADP sibosylaiing Gi, the inhibitory subunit of G protein, thus increasing cyclic AMP. Mark this question = => Question Id : 149468 Question 17 of 30 Examination of a 39 year old woman with 3 days of progressive joint pain in her ankles, knees, and wtists shows a diffuse petechial rash over her trunk and extensor surfaces. Her aridles and knees are swollen, red, and tender with decreased range of motion, and there ic tenderness cver the tendon sheaths of her hands and ferearme. Blood cultures are negative. Toint uid aspiration reveals @ ‘white cell count of 22,000/mmn3 with no visible organisms. A feabure of the infectious agent that allows it to produce recurrent infections is which one ofthe following? a) Ihic an intracellular pathogen bb) Its resistant to ceftriaxone ) Ihie resistant to complement-mediated lysis 4) Tis capsule is not immunogenic €) Tis pili underge antigenic and phase variation Question Explanation This isa classic presentation of Neisseria gonorrhoeae arthritis, Patients present with petechial rash, tenderness over the tendon sheaths (tenosyncvitis), ankles and knees are the most commonly involved joirts and are found on physical exam as swollen, red, and tender wih decreased range of motion Patients are continuously susceptble to reinfection because of antigenic variation and phase variation of the pii, N. gonotthoeae is not an intracellular pethogen, although it may be found intracellularly in neutrophils after it has been phagocytized. Ceftriaxone is the drug of choice for N. gonorrhoeae. Gonococai are especially susceptible to complement mediated lysis, not resistant to it Ni gonorrhoeae has an insignificant capsule that does not play a major role in the pathogenesis, but its capsule is immanogenic. Streptococcus pyogenes is the best known example of ¢ nonimmunogeric capsule, made of hyaluronic acid, but virtually all other capsules are immunogenic.

Вам также может понравиться